Download as pdf or txt
Download as pdf or txt
You are on page 1of 242

Scanned by CamScanner

Scanned by CamScanner
Scanned by CamScanner
Scanned by CamScanner
Scanned by CamScanner
Scanned by CamScanner
Scanned by CamScanner
Scanned by CamScanner
Scanned by CamScanner
Scanned by CamScanner
Scanned by CamScanner
Scanned by CamScanner
Scanned by CamScanner
Scanned by CamScanner
Scanned by CamScanner
Scanned by CamScanner
Scanned by CamScanner
Scanned by CamScanner
Scanned by CamScanner
Scanned by CamScanner
Scanned by CamScanner
Scanned by CamScanner
Scanned by CamScanner
Scanned by CamScanner
Scanned by CamScanner
Scanned by CamScanner
Scanned by CamScanner
Scanned by CamScanner
Scanned by CamScanner
Scanned by CamScanner
Scanned by CamScanner
Scanned by CamScanner
Scanned by CamScanner
Scanned by CamScanner
Scanned by CamScanner
Scanned by CamScanner
Scanned by CamScanner
Scanned by CamScanner
Scanned by CamScanner
Scanned by CamScanner
Scanned by CamScanner
Scanned by CamScanner
Scanned by CamScanner
Scanned by CamScanner
Scanned by CamScanner
Scanned by CamScanner
Scanned by CamScanner
Scanned by CamScanner
Scanned by CamScanner
Scanned by CamScanner
Scanned by CamScanner
Scanned by CamScanner
Scanned by CamScanner
Scanned by CamScanner
Scanned by CamScanner
Scanned by CamScanner
Scanned by CamScanner
Scanned by CamScanner
Scanned by CamScanner
Scanned by CamScanner
Scanned by CamScanner
Scanned by CamScanner
Scanned by CamScanner
Scanned by CamScanner
Scanned by CamScanner
Scanned by CamScanner
Scanned by CamScanner
Scanned by CamScanner
Scanned by CamScanner
Scanned by CamScanner
Scanned by CamScanner
Scanned by CamScanner
Scanned by CamScanner
Scanned by CamScanner
Scanned by CamScanner
Scanned by CamScanner
Scanned by CamScanner
Scanned by CamScanner
Scanned by CamScanner
Scanned by CamScanner
Scanned by CamScanner
Scanned by CamScanner
Scanned by CamScanner
Scanned by CamScanner
Scanned by CamScanner
Scanned by CamScanner
Scanned by CamScanner
Scanned by CamScanner
Scanned by CamScanner
Scanned by CamScanner
Scanned by CamScanner
Scanned by CamScanner
Scanned by CamScanner
Scanned by CamScanner
Scanned by CamScanner
Scanned by CamScanner
Scanned by CamScanner
Scanned by CamScanner
Scanned by CamScanner
Scanned by CamScanner
Scanned by CamScanner
Scanned by CamScanner
Scanned by CamScanner
Scanned by CamScanner
Scanned by CamScanner
Scanned by CamScanner
Scanned by CamScanner
Scanned by CamScanner
Scanned by CamScanner
Scanned by CamScanner
Scanned by CamScanner
Scanned by CamScanner
Scanned by CamScanner
Scanned by CamScanner
Scanned by CamScanner
Scanned by CamScanner
Scanned by CamScanner
Scanned by CamScanner
800+

Type text here

Scanned by CamScanner
GENERAL INTELLIGENCEAND REASONING
Solved Paper 2018 (Set-1)
1. In the following questions, select the related word (a) 40 (b) 70
pair from the given alternatives. (c) 80 (d) 60
Car : Tyre : : ? : ?
(a) House : Room (b) Labour : Salary 8. In the following question, select the related number
(c) Camera : Photo (d) House : People from the given alternatives.
11 : 121 : : 15 : ?
2. In the following question, select the related word (a) 289 (b) 343
from the given alternatives. (c) 225 (d) 217
Triangle : Figure : : Car : ?
(a) Vehicle (b) Seat 9. In the following question, select the related number
(c) Road (d) White from the given alternatives.
49 : 56 : : 81 : ?
3. In the following questions, select the related word (a) 92 (b) 88
from the given alternatives. (c) 76 (d) 84
Hour : Minute : : Kilometer : ?
(a) Meter (b) Gram 10. In the following question, select the odd word from
(c) Litre (d) Weight the given alternatives.
(a) Hand (b) Legs
4. In the following questions, select the related letters (c) Lungs (d) Ear
from the given alternatives.
FRIL : OLUI : : TRAM : ? 11. In the following question, select the odd word from
(a) PKUW (b) BDFW the given alternatives.
(c) DPFV (d) PDUW (a) Lizard (b) Rabbit
(c) Snake (d) Crocodile
5. In the following questions, select the related letters
from the given alternatives. 12. In the following question, select the odd word from
ROCK : VSGO : : BELT : ? the given alternatives.
(a) AKLM (b) FIPX (a) Car (b) Bus
(c) FCXT (d) ERPX (c) Truck (d) Transport

6. In the following question, select the related letters 13. In the following question, select the odd letters
from the given alternatives. from the given alternatives.
LS : HO : : TQ : ? (a) IFC (b) LIF
(a) FM (b) JG (c) SPM (d) GDZ
(c) CL (d) TF
14. In the following question, select the odd letters
7. In the following question, select the related number from the given alternatives.
from the given alternatives. (a) LO (b) HS
12 : 50 : : 18 : ? (c) IR (d) CY

Engineer’s platform
15. In the following question, select the odd number (c) LZD (d) NOM
from the given alternatives.
(a) XSNI (b) OJEY 23. A series is given with one term missing. Select the
(c) UPKF (d) EZUP correct alternative from the given ones that will
16. In the following question, select the odd number complete the series.
from the given alternatives. RSK, TVM, VYO, XBQ, ?
(a) 10 – 101 (b) 12 – 145 (a) RGT (b) YFA
(c) 24 – 577 (d) 26 – 675 (c) LQT (d) ZES

17. In the following question, select the odd number 24. A series is given with one term missing. Select the
from the given alternatives. correct alternative from the given ones that will
(a) 23 (b) 37 complete the series.
(c) 41 (d) 51 KPM, PVR, UBW, ZHB, ?
(a) LQV (b) NGT
18. In the following question, select the odd number (c) ENG (d) HIR
pair from the given alternatives.
(a) 11 – 26 (b) 16 – 33 25. In the following question, select the missing number
(c) 13 – 28 (d) 18 – 33 from the given alternatives.
35, 39, 48, 64, 89, ?
19. According to dictionary, which of the following word (a) 169 (b) 120
will come at second position? (c) 125 (d) 134
1. Easily 2. Earmark
3. Ear 4. Ease 26. In the following question, select the missing number
5. Earthy from the given alternatives.
(a) Earmark (b) Earthly 41, 13, 54, 67, 121, ?
(c) Ease (d) Easily (a) 143 (b) 188
(c) 194 (d) 168
20. From the given alternatives, according to dictionary,
which word will come at LAST position? 27. In the following question, select the missing number
(a) Opaque (b) Ointment from the given alternatives.
(c) Orderly (d) Ordinary 9, 23, 50, 103, 208,?
(a) 323 (b) 411
21. Arrange the given words in the sequence in which (c) 417 (d) 431
they occur in the dictionay.
1. Outlook 2. Outlet 28. Five people are standing in a ticket booking line. A, C
3. Outburst 4. Ornament and d are standing together, D is between A and C, A is
5. Outrageous immediately behing B and E is immediately behind C.
(a) 43215 (b) 34215 Who is standing second last in the line?
(c) 25341 (d) 52341 (a) C (b) E
(c) D (d) A
22. A series is given with one term missing. Select the
correct alternative from the given ones that will 29. At a family gathering C explained her relation with D
complete the series. by saying that D is the grand daughter of C’s mother’s
LMK, FGE, ZAY, TUS, ? mother. If C’s mother has no siblings, then how is D
(a) BMQ (b) SRB related to C?
Engineer’s platform
(a) Daughter (b) Wife (a) 6 (b) 14
(c) Sister (d) Mother (c) 1 (d)17

30. From the given alternative words select the word 37. Which of the following terms follows the trend of
which cannot be formed using the letters of the given the given list?
word. AABAAAAA, ABAAAAAA, BAAAAAAA, AAAAAAAB,
(a) chain (b) acute AAAAAABA, _______________ .
(c) stain (d)chain (a) AAAAABAA (b) AAAABAAA
(c) AAABAAAA (d)AABAAAAA
31. If CHATEAU is coded as DIBUFBV , then how will FOX
be coded as? 38. A plane takes off and flies 30 km South. Then it
(a) ULC (b) ENW turns East and flies 10 km, then it turns to its left and
(c) GPY (d) GPZ flies 25 km, then it turns and flies 10 km towards West.
Where is the plane now with respect to its starting
32. In a certain code language , 2579 means ‘nice bow position?
and arrow’, 3457 means ‘healthy and nice fruit’ and (a) 5 km South (b) 5 km North
‘8721 means’ bow to nice king’. Find the code for ‘bow’. (c) 55 km South (d) 55 km North
(a) 5 (b) 2
(c) 7 (d) 9 39. Two motorcycle riders start from the same point.
Rider A goes 15 km North then turns to his right and
33. In a certain code language,’+’ represents ‘-‘, ‘-‘ rides for another 8 km. Rider B goes 9 km West, then
represents ‘x’, ‘x’ represents ‘+’ and ‘+’ represents ‘+’. turns North and rides for 4 km and then turns to his
Find out the answer to the following question. right and rides 17 km. Where is rider A with respect to
100 × 5 + 15 − 12 ÷ 6 =? rider B?
(a) 513 (b) 71 (a) 11 km South (b) 11 km North
(c) -31 (d) -154 (c) 19 km North (d) 19 km South

34. If 74 % 36 = 2, 29 % 61 = 4 and 19 % 26 = 2 then find 40. In The question two statements are given, followed
the value of 48 % 25 = ? by two conclusions, I and II. You have to consider the
(a) 23 (b) 73 statements to be true even if it seems to be at variance
(c) 5 (d) 2 from commonly known facts. You have to decide which
of the given conclusions, if any, follows from the given
35. if A $ B means A is daughter of B, A # B means A statements.
Is brother of B and If A * B means A is father of Statement I: No students are scholars
B, then what does P $ Q * R # S mean? Statement II: No students are teachers
(a) P is mother of S Conclusion I: Some teachers are scholars
(b) P is daughter of S Conclusion II: All scholars are teachers
(c) P is sister of S (a) Only conclusion I follows
(d) P is mother’s mother of S (b) Only conclusion II follows
(c) Both conclusions I and II follows
36. Select the missing number from the given responses (d) Neither conclusion I nor conclusion II follows
4 10 ?
1 7 8 41. In the question three statements are given, followed
4 5 9 by three conclusions, I, II and III. You have to consider
the statements to be true even if it seems to be at
Engineer’s platform
variance from commonly known facts. You have to rectangle represents post-graduates. Which set of
decide which of the given conclusions, if any, follows letters represents men who are not cyclists?
from the given statements.
Statement I: All cotton is cloth
Statement II: All cotton is shirts
Statement III: Some cotton is woven
Conclusion I: Some woven is shirts
Conclusion II: Some cloth is woven
Conclusion III: All cloth is shirts
(a) FGBK (b) CDIJ
(a) Only conclusion I follows
(c) JCKB (d) JIHG
(b) Only conclusion II follows
(c) Only conclusion I and II follow
45. Which of the following Venn diagrams represents
(d) All conclusions, I, II and III follows
the relationship between Mothers, Nurses and
Engineers?
42. Which of the following cube in the answer figure
cannot be made based on the unfolded cube in the
question figure?

43. Which of the following answer figure patterns can


be combined to make the question figure?

46. Which answer figure will complete the pattern in


the question figure?

44. In the following figure, square represents lawyers,


triangle represents cyclists, circle represents men and

Engineer’s platform
47. From the given answer figures, select the one in
which the question figure is hidden/embedded.

50. A word is represented by only one set of numbers as


given in any one of the alternatives. The sets of
numbers given in the alternatives are represented by
two classes of alphabets as shown in the given two
matrices. The columns and rows of Matrix-I are
numbered from 0 to 4 and that of Matrix-II are
numbered from 5 to 9. A letter firm these matrices can
be represented first by its row and next by its column,
for example ‘P’ can be represented by 79, 59 etc. and ‘T’
can be represented by 31, 20 etc. Similarly, you have to
identify the set for the word ‘TURF’.
Matrix I
0 1 2 3 4
48. A piece of paper is folded and punched as shown
0 D L H J B
below in the question figures. From the given answer
1 D K L K J
figures, indicate how it will appear when opened. 2 I H J L F
3 C I G H M
4 L D C J E

Matrix II
5 6 7 8 9
5 Z U Q W P
6 S X X Z W
7 R W Q N P
8 V Y Y N X
9 R Z T S U

(a) 67, 99, 75, 23 (b) 89, 56, 41, 34


(c) 87, 97, 42, 12 (d) 97, 56, 75, 24

49. If a mirror is placed on the line MN, then which of ANSWERS:


the answer figures is the right image of the given
figure? 1. (a) 2. (a) 3. (a) 4. (d) 5. (b) 6. (b) 7. (c)
8. (c) 9. (b) 10. (c) 11. (b) 12. (d) 13. (d) 14. (d)
15. (b) 16. (d) 17. (d) 18. (a) 19. (a) 20. (a) 21. (a)
22. (d) 23. (d) 24. (c) 25. (c) 26. (b) 27. (c) 28. (a)
29. (c) 30. (b) 31. (c) 32. (b) 33. (d) 34. (c) 35. (c)
36. (b) 37. (a) 38. (a) 39. (b) 40. (d) 41. (c) 42. (*)
43. (a) 44. (b) 45. (d) 46. (c) 47. (a) 48. (b) 49. (b)
50. (d)
Engineer’s platform
EXPLANATIONS: - back is also 12. While there is no such connection
between the letters of CY.
1. Car: wheel → The wheel is an essential part of the
car. Home: Rooms → Room is the essential part of the 15. On reading the English alphabet from Z to A
house. (backwards), we XSNI = [X (+5) = S, S (+5) = N, N (+5) = I]
[(+5) = 5 letters in front of distance) This type of
2. Triangle: Shape: There are many types of shapes, The connection also in UPKF and EZUP Get. While there is
triangular shape is called the triangle. Cars: Vehicles: no such connection between letters of OJEY.
There are many types of vehicles. A vehicle made with
four wheels, windows and doors is called a car. 16. → 10 − 101 → (10 × 10) + 1 = 101,

3. Hour: Minutes, → 60 minutes is equal to 1 hour. 12 − 145 → (12 × 12) + 1 = 145,


Kilometers: meter, → 1000 meters is equal to 1
kilometer. 24 − 577 → (24 × 24) + 1 = 577,

7. → 12:50 → 12 × 5 - 10 = 50, 26 − 675 → (26 × 26) + 1 = 675 = 677


Similarly, 18:80 → 18 x 5 - 10 = 80
17. 23, 37 and 41, these numbers are not divisible by
any other number. Whereas 51 is divided by 17. So the
8. → 11: 121 → 11 × 11 = 121,
correct answer is 51.
Similarly, 15: 225 → 15 x 15 = 225
18. (11-26) = (13-28) = (18-33) = -15 The difference of all
9. → 49:56 → 49 + 7 = 56, is equal. Whereas (16-33) = -17.
Similarly, 81:88 → 81 + 7 = 88.
19. → (1). Ear, (2). Earmark, (3). Earthy, (4). Ease, (5).
10. Hands, ears and legs are the outer organ of the Easily.
body, while the lungs are the inner organ of the
body. 20. → (1). Opaque, (2). Ointment, (3). Orderly,
(4). Ordinary.
11. Lizards, snakes and crocodiles are creeping
organisms in the ground, while rabbits walk son four 21. → (4). Ornament, (3). Outburst, (2). Outlet, (1).
feet. Outlook, (5). Outrageous.
23.
12. Cars, buses and trucks are used to transport or R(+2)=T T(+2)=V V(+2)=X X(+2)=Z Z
people from one place to another and this action is S(+3)=V V(+3)=Y Y(+3)=B B(+3)=E E
called transporting. So the correct answer is K(+2)=M M(+2)=O O(+2)=Q Q(+2)=Q S
Transport.
24. According to alphabetical series:
13. There is a difference of 3 characters between the K(+5)=P P(+5)=U U(+5)=Z Z(+5)=E E
letters of IFC, LIF and SPM. As such, IFC → C, D, E, F, G, P(+6)=V V(+6)=B B(+6)=H H(+6)=N N
H, I while there is no such connection between letters M(+5)=R R(+5)=W W(+5)=B B(+5)=G G
of GDZ.
25. 35 + 22 = 39,
14. The sequence of letters of the LO, HS and the IR, get 39 + 32 = 48,
the reading of English alphabet from the front, then 48 + 42 = 64,
reading from the back. As such, LO, → the sequence of 64 + 52 = 89,
Lon front is 12 position and the sequence of O on the 89 + 62 = 125,

Engineer’s platform
26. 41 female. Condition (2) → 𝐴 # 𝐵 = 𝐴 is brother of B.
Therefore R # S = R is the brother of S. So R is male.
13+41=54, Condition (3) → 𝐴 ∗ 𝐵 = 𝐴 is the father of B. So Q * R =
Q is the father of R. So S and R’s father will be Q. So P is
54+13=67, the sister of S.

67+54=121, 36.
4 10 4+10=14
121+67=188 1 7 1+7=8
4 5 4+5=9
27. → 9, 23, 50, 103, 208, 417
9 × 2 + 5 = 23, 37. B is moving from the back to the front in each term.
23 × 2 + 3 = 103, Thus the final term will be AAAAABAA.
103 × 2 + 2 = 208,
208 × 2 + 1 = 417 .

30. acute, The last letter ‘a’ is not present in the given
words.

32. 2579(nice bow and arrow) …(1)


3457(health and nice fruit) …(2)
8721(bow to nice king) …(3)
(1) and (2) on removing common
→ (5,7)(nice, and) …(4)
(2) and (3) on removing common
→ (7)(nice) 44. Post graduate = (A, J, I, H, G, K)
By comparing (4) and (5) Lawyer = (G, H, I, D, E, F)
→ 7 = nice and 5 = and Cyclist = (F, G, K, B)
(1) and (3) on removing common Men = (J, I, C, D)
→ (2)( nice, bow) ∴ Men - = (J, I, C, D) – (F, G, K, B)
By comparing (6) and (7) = JICD
→ 2 = bow ∴ Option (b) is correct.

33. (+) = (-), (-) = (×), (×) = (+), (+) = (+) 50. From the given option.
(100 x 5 + 15 – 12÷ 6 ) From option (d)
100
(100 ÷ 5 − 15 × 12 + 6) = ( 5
)- (15 x 12) + 6 97 → T, 56 → U, 75 → R and 24 → F
= 20 – 180 + 6 ∴ Option (d) is correct.
= -154

34. 74%36 = 2 → [(7 − 6) + (4 − 3) = 1 + 1 = 2],


48%25 → [(4 − 5) + (8 − 2) = −1 + 6 = (5)

35. Have given, Condition (1) → A $ B = A is the


daughter of B. So P $ Q = A is brother of Q. So P is

Engineer’s platform
Solved Paper 2018 (Set-2) 8. In the following question, select the related number
form the given alternatives.
1. In the following question, select the related word pair 42 : 105 : : 38 : ?
from the given alternatives. (a) 111 (b) 135
Heart : Organ : : ? : ? (c) 95 (d) 83

(a) Bus : Vehicle (b) Car : Road 9. In the following question, select the related number
(c) Ship : Sea (d) Train : Rail form the given alternatives.
24 : 39 : : 49 : ?
2. In the following question, select the related word (a) 68 (b) 64
from the given alternatives. (c) 69 (d) 79
Big : Small : : Solid : ?
(a) Metal (b) Liquid 10. In the following question, select the odd word from the
(c )Smoke (d) Cold given alternatives.
(a) Liver (b) Intestine
3. In the following question, select the related word (c) Organ (d) Lungs
from the given alternatives.
Calculator : Calculations : : Scale : ? 11. In the following question, select the odd word from the
(a) Inches (b) Centimeters given alternatives.
(c) Measure (d) Steel (a) Black (b) Yellow
(c) Rainbow (d) Pink
4. In the following question, select the related letters
from the given alternatives. 12. In the following question, select the odd word from the
MUFT : PXIW : : GONE : given alternatives.
(a) HMRL (b) JSPA (a) Arteries (b) Veins
(c) JQRH (d) HQRF (c) Hand (d) Bones

5. In the following question, select the related letters 13. In the following question, select the odd letters from
from the given alternatives. the given alternatives.
PINK : TMRO : SOLD : ? (a) RLF (b) MGA
(a) HQTX (b) WSPH (c) EYR (d) HBV
(c) HAQV (d) AQBH
14. In the following question, select the odd letters from
6. In the following question, select the related letters the given alternatives.
from the given alternatives. (a) IR (b) KP
SW : VA : LT : ? (c) GT (d) OV
(a) NY (b) LA
(c) OX (d) OL 15. In the following question, select the odd letters from
the given alternatives.
7. In the following question, select the related number (a) XTQMJ (b) FBYUR
from the given alternatives. (c) GCZVS (d) TPMIE
(a)16 : 259 (b)13 : 171
(c)14 : 199 (d)14 : 195 16. In the following question, select the odd number
from the given alternatives.

Engineer’s platform
(a) 4 -21 (b) 7 – 54 (a)ACNG (b)TCCX
(c) 6 – 41 (d) 3 – 16 (c)TBCW (d)FRIP

17. In the following question, select the odd number 24. A series is given with one term missing. Select the
from the given alternatives. correct alternatives from the given ones that will
(a) 11 – 17 (b) 23 – 31 complete the series.
(c) 43 – 59 (d) 41 – 47 TRO, PNK, LJG, ? , DBY
(a)FBC (b)FNB
18. In the following question, select the odd number (c)FRN (d)HFC
from the given alternatives.
(a)12 – 156 (b)13 – 182 25. In the following question, select the missing number
(c)14 – 210 (d)15 – 230 from the given series.
11, 16, 21, 13, 19, 25, 15, 22, 29, ? , 25
19. Arrange the given words in the sequence in which (a)18 (b)17
they occur in the dictionary. (c)19 (d)21
1. Shocking 2. Shiver
3. Shelter 4. Shorten 26. In the following question, select the missing number
5. Shudder from the given series.
(a)23145 (b)32145 2112, 2328, 2671, 3183, ? , 4912
(c)14523 (d)41523 (a)3826 (b)3742
(c)3912 (d)3992
20. According to dictionary, which of the following word
will come at LAST position? 27. In the following question, select the missing number
1. Operation 2. Orderly from the given series.
3. Openly 4. Opulent 21, 22, 34, 5, 71, 180, 543, ?
5. Oral (a)1864 (b)2234
(a)Oral (b)Orderly (c)2336 (d)1904
(c)Opulent (d)Operation
28. M is 2 years older than P. L is 2 years older than O. O’s
21. From the given alternatives, according to dictionary, age is the average of the ages of L and N. P’s age is the
which word will come at THIRD position? average of the ages of L and M and L’s age is the
(a)Passage (b)Pastime average of P and O. Who is the youngest?
(c)Patience (d)Pathetic (a)L (b)M
(c)N (d)O
22. A series is given with one term mission. Select the
correct alternatives from the given ones that will 29. X said to Y that your mother’s father-in-law’s mother-
complete the series. in-law’s of my father. How are X and Y related?
AGM, FLR, KQW, PVB, ? (a)X is sister’s husband of Y
(a)UAG (b)VLC (b)X is child of Y’s father’s sister
(c)CGN (d)PNO (c)Y is father’s brother of X
(d)Y is sister’s husband of X
23. A series is given one term missing. Select the correct
alternatives from the given ones that will complete the 30. From the given alternatives words select the word
series. which cannot be formed using the letters of the given
FROM, CNLI, ZJIE, WFFA, ? word.
Engineer’s platform
GEOGRAPHIC 37. Which of the following terms follows the trend of the
(a)cheap (b)rogue given list?
(c)price (d)graph CABABABAB, ACBABABAB, ABCABABAB, ABACBABAB,
ABABCABAB, -------------
31. If BANQUET is coded as ZYLOSCR, then how will (a)ABABABACB (b)ABABABCAB
NEW, be coded as? (c)ABABACBAB (d)ABABABABC
GEOGRAPHIC
(a)MVD (b)MDV 38. A ship starts from the port and sails 43 miles East the it
(c)LCU (d)OFX turns south and sails 17 miles, then it turns West and
sails 25 miles, then it turns to its right and sails 17 miles.
32. In a certain code language, 9124 mean’s “run Where is it with respect to the port from where it
around the block”. 2548 mean’s “don’t block the path” started?
and 4763 mean’s “chock a block full”. Find the order for (a)18 miles West (b)68 miles East
“the’. (c)18 miles East (d)68 miles West
(a)1 (b)9
(c)2 (d)4 39. Two planes start from the same strip. Plane P flies 15
miles west, and then turns left flies 22 miles and lands.
33. In a certain code language, ‘+’ represents ‘-‘, ‘-‘ In the meanwhile plane Q flies 7 miles south then flies
represents ‘+’ ‘x’, ‘x’ represents ‘+’ and ‘+’ represents 11 miles east, then turns to its right flies 15 miles and
‘+’. Find out the answer to the following question. lands. Where is plane Q with respect to plane P?
90 × 10 ÷ 25 – 5 + 50 = ? (a)26 miles East (b)26 miles West
(a) 81 (b)84 (c)4 miles West (d)4 miles West
(c) 100 (d) 48
40. In the question two statements are given, followed by
34. If 11 @ 7 = 2, 18 @ 0 = 9 and 22 @ 20 = 1, then find two conclusions, I and II. You have to consider the
the value of 16 @ 10 = ? statements to be true even if it seems to be at variance
(a)6 (b)160 from commonly known facts. You have to decide which
(c)26 (d)3 of the given conclusions, if any, follows from the given
statements.
35. If A*B means A is father of B, A+B means A is sister Statements I : All officers are mothers
of B if A % B means A is daughter of B, then what does C Statements II : Some doctors are mothers
* D % E + F mean? Statements III : All doctors are mothers
(a)C is F’s husband (b)C is F’s wife’s brother Statements IV : Some mothers are officers.
(c)C is father of F (d)C is F’s sister’s husband (a)Only conclusion I follows
(b)Only conclusion II follows
36. Select the missing number from the given (c)Both conclusions I and II follow
responses. (d)Neither conclusion I nor conclusion II follows

8 10 2 41. In the question three statements are given, followed by


9 ? 8 three conclusions, I, II and III. You have to consider the
statements to be true even if it seems to be at variance
17 19 10
from commonly known facts. You have to decide which
of the given conclusions, if any, follows from the given
(a)1 (b)9 statements.
(c)-1 (d)-9 Statement I : All desert is jungle.
Engineer’s platform
Statement II : All sand is desert.
Statement III : No trees are jungle.
Conclusion I : No trees are desert. (d)
Conclusion II : No sand is trees.
Conclusion III : All jungle is sand. 44. In the following figure, square represents Artists,
(a)Only conclusion I follows. triangle represents Military officers, circle represents
(b)Only conclusion II follows. collectors and rectangle represents Fathers. Which set of
(c)Only conclusion I and II follow. letters represents collectors who are either military
(d)None of the conclusion follow. officers or fathers?

42. Which of the following cube in the answer figure


cannot be made based on the unfolded cube in the
question figure?

(a) H (b) B
(c) G (d) F

45. Which of the following Venn diagrams represents the


relationship between Women, Mothers and Teacher ?

(a) (b)
(a) (b)

(c) (d)
(c) (d)

43. Which of the following answer figure patterns can


46. Which answer figure will complete the pattern in the
be combined to make the question figure?
question figure?

(a) (b)
(a)

(c) (d)
(b)
47. From the given answer figures, select the one in
which the question figure is hidden/embedded.

(c)
Engineer’s platform
from 5 to 9. A letter from these matrices can be
(a) (b) represented first by its row and next by its column, for
example ‘H’ can be represented by 21, 43 etc. and ‘R’
can be represented by 96, 87 etc. Similarly, you have to
(c) (d) identify the set for the word ‘POEM’.

Matrix I
48. A piece of paper is folded and punched as shown 0 1 2 3 4
below in the question figures. From the given answer 0 A A L A B
figures, indicate how it will appear when opened. 1 C A F B C
2 J H F B K
3 F J A M L
4 I B B H E

(a) (b) Matrix II


5 6 7 8 9
5 Q U Y N Z
(c) (d) 6 N R V O Z
7 P N Q W Q
8 X N R V V
9 X R V T O
49. If a mirror is placed on the line MN, then which of
the answer figures is the right image of the given (a)76, 69, 01, 33 (b)56, 88, 23, 41
figure? (c)89, 56, 44, 02 (c)75, 68, 44, 33

(a) (b)
ANSWERS:

1. (a) 2. (b) 3. (c) 4. (c) 5. (b) 6. (c) 7. (b)


(c) (d) 8. (c) 9. (b) 10. (c) 11. (c) 12. (c) 13. (c) 14. (d)
15. (d) 16. (d) 17. (b) 18. (d) 19. (b) 20. (b) 21. (d)
22. (a) 23. (c) 24. (d) 25. (b) 26. (c) 27. (d) 28. (c)
50.A word is represented by only one set of numbers as 29. (b) 30. (b) 31. (c) 32. (c) 33. (b) 34. (d) 35. (d)
given in any one of the alternatives. The sets of 36. (b) 37. (c) 38. (b) 39. (a) 40. (b) 41. (c) 42. (d)
numbers given in the alternatives are represented by 43. (a) 44. (d) 45. (d) 46. (d) 47. (c) 48. (a) 49. (a)
two classes of alphabets as shown in the given two 50. (d)
matrices. The columns and rows of Matrix-I are
numbered from 0 to 4 that of Matrix-II are numbered EXPLANATIONS: -

Engineer’s platform
1. Heart organ = Heart is a of organ, Bus ; vehicle = bus Similarly, 49:64 ↔ [49 − 64] = −15
is a kind of vehicle.

2. Big : Small = (antonyms word) Similarly, Solid : Liquid 10. The organ is the correct answer. In other options,
= (antonyms word) the names of internal parts of the organs are written.

3. calculation = [Device : work of the device] similarly, 11. The rainbow has seven colors. In other options have
scale : measure? [device : work of the device]. names of different colors.

𝑀(3) = 𝑃 12. The hand is the correct answer. In other options the
4. MUFT : PXIW =
𝑈(+3) = 𝑋
names of internal parts of the body are written.
𝐹(+3) = 𝐼
𝑇(+3) = 𝑊
𝐺(+3) = 𝐽 13. RLF⇒ [𝑅(−6) = 𝐿 ↔ 𝐿(−6) = 𝐹]
Similarly, GONE : JRQH= Similar difference is also in MGA and HBV.
𝑂(+3) = 𝑅
𝑁(+3) = 𝑄 EYR ⇒ [𝐸(−6) = 𝑌 ↔ 𝑌(−7) = 𝑅]
𝐸(+3) = 𝐻 This option is different.

𝑃(+4) = 𝑇 16. 4-21 ↔ [42 + 5 = 21]


5. PINK : TMRO =
𝐼(+4) = 𝑀 7-54 ↔ [72 + 5 = 54]
𝑁(+4) = 𝑅
6-41 ↔ [62 + 5 = 41]
𝐾(+4) = 𝑂
𝑆(+4) = 𝑊
Similarly, SOLD : WSPH=
𝑂(+4) = 𝑆
17. 11-17 ↔ [11 − 17 = −6]
𝐿(+4) = 𝑃
𝐷(+4) = 𝐻 23-31 ↔ [23 − 31 = −8]
41-47 ↔ [41 − 47 = −6]
43-59 ↔ [43 − 59 = −16]
6.
𝑆(+3) = 𝑉 This option is different.
SW↔ ↔ 𝑉𝐴
𝑊(+4) = 𝐴
𝐿(+3) = 𝑂 18. 12-156 ↔ [122 + 12 = 156]
Similarly, LT↔ ↔ 𝑶𝑿 13-182 ↔ [132 + 13 = 182]
𝑇(+4) = 𝑋 14-210 ↔ [142 + 14 = 210]

7. 19,363 ⇒ [192 + 2 = 363] 19. (3) Shelter, (2) Shiver, (1) Shocking, (4) Shorten, (5)
Similarly, 13,171 ⇒ [132 + 2 = 171] Shudder

42 38 20. Openly, Operation, Opulent, Oral, Orderly.


8. =
105 𝑥
105×38
⇒𝑥= 42
= 95 𝑎𝑛𝑠𝑤𝑒𝑟 21. Passage, pastime, Pathetic, Patience.

9. 24:39 ↔ [24 − 39] = −15 22.


Engineer’s platform
A(+5)=F F(+5)=K K(+5)=P P(+5)=U U
G(+5)=L L(+5)=Q Q(+5)=V V(+5)=A A 31.
M(+5)=R R(+5)=W W(+5)=B B(+5)=G G B A N Q U E T
UAG. (-2) (-2) (-2) (-2) (-2) (-2) (-2)
Z Y L O S C R
23.
F(-3)=C C(-3)=Z Z(-3)=W W(-3)=T T N E W
R(-4)=N N(-4)=J J(-4)=F F(-4)=B B (-2) (-2) (-2)
O(-3)=L L(-3)=I I(-3)=F F(-3)=C C L C U
M(-4)=I I(-4)=E E(-4)=A A(-4)=W W
TBCW. 32. 9124 = run around the block …(1)
2548 = don’t block the path …(2)
24. 4763 = chock a block full …(3)
T(-4)=P P(-4)=L L(-4)=H H(-4)=D D From (2) and (3)
R(-4)=N N(-4)=J J(-4)=F F(-4)=B B 4 = block
O(-4)=K K(-4)=G G(-4)=C C(-4)=Y Y From eq. (1) and (2)
HFC. 2 = the (answer).

25. 11,16,21,13,19,25,15,22,29,17,25 33.


[11,16,21,], [13,19,25], [15,22,29,], [17,25] (+) (−), ⇒ (×), (×) ⇒ (÷), (÷) ⇒ (+)
11(+2)=13 13(+2)=15 15(+2)=17 17 Now changing sign symbol, (90 × 10 ÷ 25 – 5 + 50)
16(+3)=19 19(+3)=22 22(+3)=25 25 34.
11−7
21(+4)=25 25(+4)=29 29 11@7 = 2 ⇒ [ = 2]
2
17. 18−0
18@0 = 9 ⇒ [ = 9]
2
22−20
26. 2112, 2328, 2671, 3183, 3912, 4912 22@20 = 1⇒ [ 2 = 1]
16−10
16@10 = = 3 (answer)
2
27. 21 ↔ 21 + 1=22
3 35.
22 ↔ 22 × 2 + 1.5 = 34.5 Condition (1) = A* B=A is the father of B. So C * D = C is the
4
34.5 ↔ 33 × 2 + 1.5 + 0.5 = 71 father of D.C is father.
5
71 ↔ 71 × 2 + 1.5 + 0.5 + 0.5 = 180 Condition (2) = A % B = A is the daughter of B.
6 Therefore D% E. = D is the daughter of E. E is mother.
180 ↔ 180 × 2 + 1.5 + 0.5 + 0.5 + 0.5 = 543 Condition (3) = A + B = A is the sister of B.
7
543 ↔ 543 × + 0.5 + 0.5 + 0.5 + 0.5 = 1904 So E + F. = E is the sister of F.
2
Therefore, C is the husband if F (E)’s sister.
29. X said to Y 36.
Mother of Y (wife of the father of Y), Mother’s 8 + 9 = 17, 10 + (9) = 19
father-in-law (Y’s grandfather), wife of father-in-law (Y’s 37.
grandmother), ↔ X’s father’s mother-in-law (husband The order of C is moving forward in each post.
of Y’s grandmother, son of X). So X is the son of Y’s 1
𝐴𝐵 𝐴𝐵 𝐴𝐵 𝐴𝐵,
father’s sister. 𝐶
2
𝐵 𝐴𝐵 𝐴𝐵 𝐴𝐵,
𝐴𝐶
30. Rogue, u is not in the given letters. So this word can 3
not be made.
𝐴𝐵 𝐴𝐵 𝐴𝐵,
𝐴𝐵𝐶

Engineer’s platform
4
𝐵𝐴𝐵 𝐴𝐵,
𝐴𝐵𝐴𝐶
5
𝐴𝐵 𝐴𝐵 𝐴𝐵,
𝐴𝐵𝐶
6
𝐴𝐵 𝐴𝐵 𝐵 𝐴𝐵
𝐴𝐶
38.

44. F = Collector and military officer.

45.

Ship begin to move 43mils from A to C, than 17mils


from C to D, then 25mils from D to E, than 17mils from
E to B.
⇒ AB = AC – BC
= AC – ED (BC = ED)
= 43 – 25 = 18 mils.
Hence ship is 18miles east from the starting point.

39.

P and Q begin to move from point O. P move 15mils


from O to A, than 22 miles from A to B. Q move 7 miles
from O to C, than 11 miles from C to D, than 15 mils D
to E.
⇒ BE = AO + CD = 15 + 11 = 26 mils.
Hence Q is 26 miles east of P.

40.

41.

Engineer’s platform
Solved Paper 2018 (Set-3) 8. In the following question, select the related number
1. In the following question, select the related word pair from the given alternatives.
from the given alternatives. 5 : 125 : : 6 : ?
Red : Danger :: ? : ? (a) 343 (b) 576
(a) White : War (b) Yellow : cold (c) 216 (d) 326
(c) Blue : Pain (d) Black : Sorrow
9. In the following question, select the related number
2. In the following question, select the related word pair from the given alternatives.
from the given alternatives. 46 : 69 :: ? : ?
Blood : Red :: Coal : ? (a) 52 : 78 (b) 48 : 74
(a) Black (b) Heat (c) 58 : 86 (d) 60 : 80
(c) Solid (d) Mine
10. In the following question, select the odd word from
3. In the following question, select the related word pair the given alternatives.
from the given alternatives. (a) Cataract (b) Rickets
Sad : Happy :: ? : ? (c) Iron (d) Goiter
(a) History : Old
(b) Modern : Ancient 11. In the following question, select the odd word from
(c) Economics : Mathematics the given alternatives.
(d) Love : Beautiful (a) Hindi (b) Telugu
(c) Oriya (d) Australia
4. In the following question, select the related letter
pair from the given alternatives. 12. In the following question, select the odd word from
MOL : TVS : : ? : ? the given alternatives.
(a) NAP : UHW (b) NAP : CHK (a) Slipper (b) Shoes
(c) NOT : UHW (d) NOT : HFG (c) Sandals (d) Shirt

5. In the following question, select the related letters 13. In the following question, select the odd letters
pair from the given alternatives. from the given alternatives.
FORK : ILUH :: LAPE : ? (a) DLTB (b) EMUC
(a) TBXO (b) XBTC (c) GPXF (d) IQYG
(c) OXSB (d) GYTC
14. In the following question, select the odd letters
6. In the following question, select the related letters from the given alternatives.
pair from the given alternatives. (a) CHLQU (b) HMQVZ
MILK : PMOO :: TIPS : ? (c) MRVZF (d) AFJOS
(a) WAMS (b) GMSW
(c) GFBG (d) WMSW 15. In the following question, select the odd letters
from the given alternatives.
7. In the following question, select the related number (a) LQV (b) FKP
from the given alternatives. (c) MRX (d) GLQ
51 : 66 :: 81 : ?
(a) 76 (b) 96 16. In the following question, select the odd number
(c) 98 (d) 82 from the given alternatives.
(a) 11 – 17 (b) 23 – 31
Engineer’s platform
(c) 43 – 59 (d) 41 – 47 23. A series is given with one term missing. Select the
correct alternative from the given once that will
17. In the following question, select the odd number complete the series.
from the given alternatives. FIN, NQV, VYD, ?, LOT
(a) 12 – 156 (b) 13 – 182 (a) HLS (b) DGL
(c) 14 – 210 (d) 15 – 230 (c) HGP (d) GLH

18. In the following question, select the odd number 24. A series is given with one term missing. Select the
pair from the given alternatives. correct alternative from the given ones that will
(a) 23 – 92 (b) 31 – 124 complete the series.
(c) 43 – 182 (d) 37 – 148 LYMT, NCOX, PGQB, RKSF, ?
(a) JKLT (b) TOUJ
19. Arrange the given words in the sequence in which (c) NMLP (d) LTMA
they occur in the dictionary.
1. Please 2. Plunge 25. In the following question, select the missing number
3. Plummet 4. Plush from the given series.
5. Pleasure 31, 32, 40, 67, 131, ?
(a) 51324 (b) 15324 (a) 242 (b) 256
(c) 43125 (d) 34125 (c) 322 (d) 194

20. Arrange the given words in the sequence in which 26. In the following , select the missing number from
they occur in the dictionary. the given series.
1. Primary 2. Previous 31, 44, 75, 119, 194, ?
3. Presence 4. Preside (a) 243 (b) 313
5. Prestige (c) 353 (d) 393
(a) 54321 (b) 34521
(c) 45123 (d) 54123 27. In the following question, select the missing number
from the given series.
21. Arrange the given words in the sequence in which 44, 22, 33, 82.5,288.75, ?
they occur in the dictionary. (a) 1299.375 (b) 1649.43
1. Sickly 2. Shrivel (c) 2024.5 (d) 2345.375
3. Shrewd 4. Sinful
5. Singer 28. Five people P, Q, R, S and T are standing in a queue.
(a) 32145 (b) 23145 R is standing between P and T. P is just behind Q and Q
(c) 32541 (d) 23541 is second in the queue. Who is second last in the
queue?
22. A series is given with one term missing. Select the (a) T (b) S
correct alternative from the given ones that will (c) R (d) P
complete the series.
MK, IF, EA, ? , WQ 29. H said to W that you are the son- in- law of my
(a) SR (b) LQ husband’s sister’s mother’s husband. How are H and W
(c) AV (d) FN related to each other?
(a) W is the son-in-law of H

Engineer’s platform
(b) W is the husband of the sister of H’s husband (c) 15 (d) 4
(c) W is the husband of the sister of H
(d) W is the brother of H’s husband’s mother 37. Which of the following terms follows the trend of
the given list?
30. From the given alternative words select the word ABABABABC, ABABABACB, ABABABCAB, ABABACBAB,
which cannot be formed using the letters of the given ABABCABAB, _______________ .
word. (a) ACBABABAB (b) ABCABABAB
(a) linear (b) raise (c) ABACBABAB (d)CABABABAB
(c) resin (d) shin
38. A salesman starts on his job and walks 3 km West,
31. If STERNLY is coded as TUFSOMZ, then how will COP then he turns South and walks 4 km, then he turns West
be coded as? and walks 5 km, and then he turns to his right and walks
(a) XLK (b) BNO 4 km. Where is he now with respect to his starting
(c) DPQ (d) EQR position?
(a) 8 km East (b) 2 km West
32. In a certain code language, 2357 means ‘please bear (c) 2 km East (d) 8 km West
with me’, 8159 means ‘much burden to bear’ and 7654
means ‘hunter with the bear’. Find the code for ‘with’. 39. Two buses start from the same depot. Bus A goes 27
(a) 7 (b) 2 km West, then turns to its right and goes 43 km. In the
(c) 3 (d) 5 meanwhile Bus B goes 19 km North, then turns West
and goes 61 km, then turns to its right and goes 24 km.
33. In a certain code language, ‘+’ represents ‘−‘, ‘−‘ Where is Bus B with respect to Bus A?
represents ‘×’, ‘×’ represents ‘+’ and ‘+’ represents ‘+’ (a) 34 km East (b) 88 km West
Find out the answer to the following question. (c) 88 km East (d) 34 km West
196 + 4 – 125 × 50 + 10 = ?
(a) −61 (b) 196 40. In the question two statements are given, followed
(c) 93 (d) -65 by two conclusions, I and II. You have to consider the
statements to be true even if it seems to be at variance
34. If 21# 11 = 5, 18 # 4 = 7 and 6 #2 =2, then find the from commonly known facts. You have to decide which
value of 13 # 5 = ? of the given conclusions, if any, follows from the given
(a) 4 (b) 8 statements.
(c) 18 (d) 65 Statement I: No green is toxic
Statement II: Some chemicals are green
35. If A $ B means A is son of B, A # B means A is sister Conclusion I: Some chemicals are toxic
of B and If A*B mans A is father of B, then what does X $ Conclusion II: Some green are chemicals
Y * Z # W mean? (a) Only conclusion I follows
(a) X is father of W (b) X is brother of W (b) Only conclusion II follows
(c) W is father of X (d) X is father’s father of W (c) Both conclusions I and II follows
(d) Neither conclusion I nor conclusion II follows
36. Select the missing number from the given
responses. 41. In the question three statements are given, followed
8 4 12 by three conclusions, I, II and III. You have to consider
7 8 ? the statements to be true even if it seems to be at
8 5 13 variance from commonly known facts. You have to
(a) 1 (b) -1
Engineer’s platform
decide which of the given conclusions, if any, follows 44. In the following figure, square represents Priests,
from the given statements. triangle represents singers, circle represents Therapists
Statement I: All sofas are chairs and rectangle represents Indians. Which set of letters
Statement II: Some chairs are lounges represents Indians who are not priests?
Statement III: All lounges are recliners
Conclusion I: Some chairs are sofas
Conclusion II: Some sofas are lounges
Conclusion III: Some recliners are chairs
(a) Only conclusions I and II follow
(b) Only conclusions II and III follow
(c) Only conclusions I and III follow
(d) All conclusions I, II and III follow
(a) FGBK
(b) AH
42. Which of the following cube in the answer figure
(c) EC
cannot be made based on the unfolded cube in the
(d) HFE
question figure?
45. Which of the following Venn diagrams represents
the relationship between Asia, Inidia and Canada?

46. Which answer figure will complete the pattern in


43. Which of the following answer figure patterns can
the question figure?
be combined to make the question figure?

47. From the given answer figures, select the one in


which the question figure is hidden/embedded.
Engineer’s platform
numbered from 0 to 4 and that of Matrix-II are
numbered from 5 to 9. A letter from these matrices can
be represented first by its row and next by its column,
for example ‘E’ can be represented by 32, 12 etc and ‘X’
can be represented by 87, 79 etc. Similarly, you have to
identify the set for the word ‘MYTH’.
Matrix I Matrix II
0 1 2 3 4 5 6 7 8 9
0 H C H E H 5 O P T V U
1 G B E J D 6 P Y V O Z
2 I M I H A 7 T S S V X
3 J B E D D 8 O O X Z V
48. A piece of paper is folded and punched as shown 4 F L C C M 9 R Z U U O
below in the question figures. From the given answer (a) 44, 89, 57, 23 (b) 20, 89, 66, 43
figures, indicate how it will appear when opened. (c) 21, 66, 75, 04 (d) 32, 75, 43, 66

49. If a mirror is placed on the line MN, then which of


the answer figures is the right image of the given
figures?

ANSWERS:

1. (d) 2. (a) 3. (b) 4. (a) 5. (c) 6. (d) 7. (b)


8. (c) 9. (a) 10. (c) 11. (d) 12. (d) 13. (c) 14. (c)
15. (c) 16. (c) 17. (d) 18. (c) 19. (a) 20. (b) 21. (a)
22. (c) 23. (b) 24. (b) 25. (b) 26. (b) 27. (a) 28. (c)
50. A word is represented by only one set of numbers as 29. (b) 30. (b) 31. (c) 32. (a) 33. (b) 34. (a) 35. (a)
given in any one of the alternatives. The sets of 36. (c) 37. (c) 38. (d) 39. (a) 40. (b) 41. (b) 42. (b)
numbers given in the alternatives are represented by 43. (a) 44. (b) 45. (b) 46. (a) 47. (d) 48. (c) 49. (c)
two classes of alphabets as shown in the given two 50. (c)
matrices. The columns and rows of Matrix-I are
Engineer’s platform
EXPLANATIONS: - 10. Iron is different from other options. In other options
the names of diseases are written.
1. Red : Danger :: Red color is a symbol of danger.
Black : Sorrow = Black is a symbol of sorrow. 11. Hindi, Telugu and Oriya are the language spoken in
India. While Australia is the name of a country.
2. Blood : Red = Blood is of red color.
Coal : Black = The color of coal is black. 12. The shirt is worn over the body. Slippers, shoes and
sandals are worn in the legs.
3. Sad : Happy. [antonym], similarly,
Modern : Ancient [ O antonym] 13. GPXF
D(+8) = L L(+8) = T T(+8)=B B
4. M(+7) = 7 E(+8) = M M(+8) = U U(+8)=C C
(2) MOL O(+7) = V TVS, (G(+9)=P) P(+8) = X X(+8)=F F
L(+7) = S (different)
I(+8) = Q Q(+8) = Y Y(+8)=G G
N(+7) = U
Similarly, NAP A(+7) = H UHW,
14. MRVZF
P(+7) = W
C(+5)=H H(+4)=L L(+5)=Q Q(+4)=U U
H(+5)=M M(+4)=Q Q(+5)=V V(+4)=Z Z
5. F(+3) = I M(+5)=R R(+4)=V (V(+4)=)Z Z(+4)=F f
FORK O(-3) = L ILUH, (Different)
R(+3) = U
K(-3) = H 15. MRX
L(+5)=Q Q(+5)=V V
L(+3) = O F(+5)=K K(+5)=P P
Similarly, LAP A(-3) = X OXSB, M(+5)=R (R(+6)=X)(Different) X
P(+3) = S G(+5)=L L(+5)=Q Q
E(-3) = B
16. 11 – 17 → [11 - 17 = -6]
6. M(+3) = P 41 – 47 → [41 – 47 = -6]
MILK I(+4) = M PMOO, 23 – 31 → [23 – 31 = -8]
L(+3) = O 43 – 59 → [43 – 59 = -16] (Answer)
L(+4) = O
17. 12 – 156 → [ 12 X 12 + 12 = 156]
T(+3) = W 13 – 182 → [ 13 X 13 + 13 = 182]
Similarly, TIPS I(+4) = M WMSW, 14 – 210 → [ 14 X 14 + 14 = 210]
P(+3) = S 15 – 230 → [ 15 X 15 + 5 = 230] (it is different from
S(+4) = W others) Answers.

7. 51 : 66 → [51 + 15 =66] 18. 23 – 92 → [23 X 4 = 92]


Similarly, 81 : 96 → [81 + 15 = 96] 31 – 124 → [31 X 4 = 124]
43 – 182 → [43 X 4 = 172] Answer.
8. 5 : 125 ? [53=125] 37 – 148 → [37 X 4 = 148]
Similarly, 6 : 216 ? [63=216]
19. (1) Please (5) Pleasure (3) Plummet (2) Plunge
Engineer’s platform
(4) Plush
8𝟐.5×3
82.5 ↔ [ 2
+ 8𝟐. 5 + 8𝟐. 5 = 28𝟖. 75]
20. (3) Presence (4) Preside (5) Prestige (2) Previous
(1) Primary 28𝟖.75×3
288.75 ↔ [ 2
+ 28𝟖. 75 + 28𝟖. 75 +

21. (3) Shrewd (2) Shrivel (1) Sickly (4) Sinful (5) Singer 28𝟖. 75 =
129𝟗. 375]
22. MK [M(-2) = K]
IF [I(-3) = F] 28. According to question the sequence of position we
EA [E(-4) = A] get is T, R, P, Q, S.
AV [A(-5) = V] Answer.
WQ [W(-6) = Q] 31.
S T E R N L Y
(+1) (+1) (+1) (+1) (+1) (+1) (+1)
23. DGL (Answer.)
T U F S O M Z
F(+3)=I I(+5)=N N FIN
N(+3)=Q Q(+5)=V V NQV
C O P
V(+3)=Y Y(+5)=D D VYD (DPQ) ans.
(+1) (+1) (+1)
D(+3)=G G(+5)=L L (DGL)
D P Q
L(+3)=O O(+5)=T T LOT

32. 2357= please bear with me …(1)


24.
8159= much burden to bear …(2)
L(+2)=N N(+2)=P P(+2)=R R(+2)=T T
Y(+4)=C C(+4)=G G(+4)=K K(+4)=O O 7654= hunter with the bear …(3)
M(+2)=O O(+2)=Q Q(+2)=S S(+2)=U U From equation (2) and (3)
T(+4)=X X(+4)=B B(+4)=F F(+4)=J J 5 → bear
LYMT NCOX PGQB RKSF TOUJ From equation (1) and (3)
(Answer) 7 → with (Answer)

25. 31 ↔ [31 + 13 = 32] 33. (+) → (-), (-) → (X), (X) → (+), (+) → (+)
32 ↔ [32 + 23 = 40] (196 ÷ 4 − 125 × 50 + 10)
40 ↔ [40 + 33 =67] On changing the symbol of the equation
67 ↔ [67 + 43 =131] 125
(196 ÷ 4 × 125 ÷ 50 − 10) = 196 + 4 × ( ) − 10
131 ↔ [131 + 53 =256] 50
256 (Answer.)
= 196 + (4 X 2.5) – 10
26. 31 = 196
44 ↔ [44 + 31 =75]
21−11
75 ↔ [75 + 44 =119] 34. 21#11 = 5 ↔[ = 5]
2
119 ↔ [119 + 75 = 194]
194 ↔ [194 + 119 = 313] 18−4
18#4 = 7 ↔ [ 2
= 7]
313 (Answer)

22×3
6−2
27. 22 ↔ [ = 33] 6#2 = 2 ↔ [ = 4]
2 2

33×3
33 ↔ [ 2
+ 33 = 8𝟐. 5]
Engineer’s platform
13 − 5
13#5 = 4 ↔ [ = 4] 𝐴𝑛𝑠𝑤𝑒𝑟
2
35. Condition (1) → A * B = A is the son of B. So Y * Z = Y
is Z’s father. Y is father.
Condition (2) → A $ B = A is the son of B So X $ Y = X is
the son of Y, X and Z is brother.
Condition (3) → A # B = A is the sister of B. Hence Z # W.
= Z is the sister of W. So X is the brother of W. A and B begin to move from point O. A move 27 km
from O to P, than 43 km from P to Q. B move 19 km
36. 8 + 4 = 12, 7 + 8 = 13 ans from O to R, than 61 km from R to S, than 24 km from S
to T.
37. The order of C is moving forward from each post. → TQ = SR - PO
= 61 – 27
= 34 km
Hence B is 34 km west of A.

38.

Salesman moves 3 km from A to B, than 4 km from B to


C, then 5 km from C to D, then 4 km from D to E.
→ EA = EB + BA
= DC + BA (EB = DC)
= 5 + 3 = 8 km
Hence salesman, is 8 km west of the starting point.

39.

Engineer’s platform
Solved Paper 2018 (Set-4) 8. In the following question, select the related number
from the given alternatives.
1. In the following question, select the related word pair 24 : 164 : : 32 : ?
from the alternatives. (a) 212 (b) 194
Badminton : Court : : ? : ? (c) 132 (d) 186
(a) Skating : Rink (b) Chess : Ground
(c) Swimming : Grass (d) Polo : Sky 9. In the following question, select the related number
pair from the given alternatives.
2. In the following question, select the related word 73 : 21 : : ? : ?
from the given alternatives. (a) 95 : 45 (b) 46 : 28
(a) Theatre (b) Cards (c) 37 : 18 (d) 19 : 10
(c) Pen (d) Acting
10. In the following question, select the odd word from
3. In the following question, select the related word the given alternatives.
from the given alternatives. (a) Forest (b) Tiger
Pentagon : Figure : : Truck : ? (c) Lion (d) Elephant
(a) Heavy (b) Vehicle
(c) Engine (d) Goods 11. In the following question, select the odd word from
the given alternatives.
4. In the following question, select the related letters (a) School (b) Worker
from the given alternatives. (c) Driver (d) Waiter
RATS : XGZY : : RIPE : ?
(a) KVMA (b) XPUL 12. In the following question, select the odd word from
(c) XOVK (d) XNVG the given alternatives.
(a) Write (b) Read
5. In the following question, select the related letters (c) Listen (d) Colour
from the given alternatives.
FEAR : PCCH : : VANS : ? 13. In the following question, select the odd letters
(a) QPYX (b) SHPX from the given alternatives.
(c) PXYB (d) QSZA (a) CHM (b) JOS
(c) SXC (d) VAF
6. In the following question, select the related letters
from the given alternatives. 14. In the following question, select the odd letters
LAMP : PXQM : : COKE : ? from the given alternatives.
(a) GMQA (b) GLAT (a) KQW (b) BHN
(c) GLOB (d) GNPC (c) RXF (d) DJP

7. In the following question, select the related number 15. In the following question, select the odd letters
from the given alternatives. from the given alternatives.
43 : 7 : : 61 : ? (a) FBX (b) PLH
(a) 6 (b) 7 (c) VRN (d) GCZ
(c) 9 (d) 8
16. In the following question, select the odd number
from the given alternatives.
(a) 17 – 289 (b) 26 – 676
Engineer’s platform
(c) 27 – 729 (d) 36 – 46 (c) EJ (d) DK

17. In the following question, select the odd number 24. A series is given with one term missing. Select the
pair from the given alternatives. correct alternative from the given onces that will
(a) 13 – 16 (b) 19 – 22 complete the series.
(c) 21 – 24 (d) 33 – 38 F, J, O, S, ?, B, G
(a) Y (b) X
18. In the following question, select the odd number (c) Z (d) C
pair from the given alternatives.
(a) 14 – 21 (b) 19 – 28.5 25. In the following question, select the missing number
(c) 17 – 25.5 (d) 36 – 46 from the given series.
14, 16, 30, 45, 62, 132, ?, ?
19. Arrange the given words in the sequence in which (a) 124,396 (b) 126,393
they occur in the dictionary. (c) 82.243 (d) 106,343
1. Loathe 2. Living
3. Locate 4. Local 26. In the following question, select the missing number
5. Literate from the given series.
(a) 52143 (b) 52134 6, 25,62, 123, ?, 341
(c) 51234 (d) 51243 (a) 194 (b) 156
(c) 214 (d) 278
20. According to dictionary, which of the following word
will come at FIRST position? 27. In the following question, select the missing number
1. Narrate 2. Narrow from the given series.
3. Naked 4. Naïve 53, 19, 72, 91, ?, 254
5. Nasty (a) 149 (b) 163
(a) Naked (b) Naïve (c) 123 (d) 213
(c) Narrate (d) Narrow
28. In a certain city there are five towers. Tower III is
21. From the given alternatives, according to dictionary, taller than tower II. Height of Tower V is the average of
which word will come at SECONG position? the height of towers I and IV. Height of Tower II is the
(a) Prodigy (b) Proceed average of the height of towers I and III. Height of tower
(c) Product (d) Producer I is the average of the height of towers II and V. Which
tower is the shortest?
22. A series is given with one term missing. Select the (a) IV (b) V
correct alternative from the given ones that will (c) I (d) II
complete the series.
RIF, KBY, DUR, WNK, ? 29. X said to Y that my sister’s father-in-law is your
(a) LFD (b) PGD husband’s father. How are X and Y related?
(c) PNC (d) NGB (a) Y is wife of X’s sister’s husband’s brother
(b) Y is sister of X’s sister’s husband
23. A series is given with one term missing. Select the (c) X is husband of Y’s husband’s sister
correct alternative from the given onces that will (d) X is wife of Y’s husband’s brother
complete the series.
PV, UA, ZF, ?, JP
(a) EK (b) DL
Engineer’s platform
30. From the given alternative words select the word 37. Which of the following terms follows the trend of
which cannot be formed using the letters of the given the given list?
word. OXXXXX, XXXXXO, XXXXOX, XXXOXX,
(a) vocal (b) ulcer XXOXXX,_____________.
(c) labor (d) burly (a) XXXXXO (b) XXXXOX
(c) XXXOXX (d) XOXXXX
31. If OBSCURE is coded as NARBTQD, then how will
THY be coded as ? 38. A bird stars from its nest and flies 3 km North, then
(a) GSB (b) VJA turns West and flies 4 km, then turns South and flies 11
(c) SGX (d) UIZ km, then turns to its left and flies 4 km. Where is it now
with respect to its nest?
32. In a certain code language, 1259 means ‘box in the (a) 8 km North (b) 8 km South
ring’, 6941 means ‘the square boxing ring’ and 5713 (c) 14 km South (d) 14 km North
means ‘put the box out’. Find the code for ‘ring’.
(a) 1 (b) 9 39. Two cars star from the same point on a highway. Car
(c) 2 (d) 5 X travels North 25 km and then turns right and travels
14 km. In the meanwhile, car Y travels 19 km South,
33. In a certain code language, ‘+’ represents ‘-‘, ‘-‘ then 3 km West, then 4 km South, then it turns left and
represents ‘x’, ‘x’ represents ‘+’ and ‘+’ represents ‘+’. travels 17 km. Where is car Y with respect to car X?
Find out the answer to the following question. (a) 48 km North (b) 10 km South
160 x 40 ÷ 20 + 10 − 2 = ? (c) 48 km South (d) 10 km North
(a) 328 (b) 85
(c) 52 (d) 4 40. In the question two statements are given, followed
by two conclusions, 𝐼 and II. You have to consider the
34. If 50 % 15 = 70 and 23 % 20 = 6 then find the value statements to be true even if it seems to be at variance
of 10 % 4 = ? from commonly known facts. You have to decide which
(a) 6 (b) 14 of the given conclusions, if any, follows from the given
(c) 12 (d) 7 statements.
Statement I: No lemon is sour
35. If A * B means A is mother of B, A + B means A is Statement II: All sour is yellow
sister of B and If A % B means A is daughter of B, then Conclusion I: Some yellow is lemon
what does C + D % E * F mean ? Conclusion II: All lemon is yellow
(a) C is daughter of F (a) Only conclusion I follows
(b) C is mother’s sister of F (b) Only conclusion II follows
(c) C is sister of F (c) Both conclusion I and II follow
(d) C is mother’s mother of F (d) Neither conclusion I nor conclusion II follows

36. Select the missing number from the given 41. In the question three statements are given, followed
responses. by three conclusions, 𝐼 and II and III. You have to
2 4 7 consider the statements to be true even if it seems to
2 ? 9 be at variance from commonly known facts. You have to
0 -6 -2 decide which of the given conclusions, if any, follows
from the given statements.
(a) 2 (b) 10 Statement I: All tanks are pools
(c) 7 (d) 11 Statement II: Some tanks are pools
Engineer’s platform
Statement III: Some seas are lakes 44. In the following figure, square represents Runners,
Conclusion I: Some seas are pools triangle represents Scientists, circle represents Indians
Conclusion II: All tanks are lakes and rectangle represents Mothers. Which set of letters
Conclusion III: No lakes are tanks represents Indian who are runners?
(a) Only conclusion I and II follow
(b) Only conclusion II and III follow
(c) Only conclusion I and III follow
(d) All conclusions I, II and III follow

(a) CEB (b) HFE


42. Which of the following cube in the answer figure
(c) HF (d) IG
cannot be made based on the unfolded cube in the
question figure?
45. Which of the following Venn diagrams represents
the relationship between Elephants, Giraffes and
Herbivores?

46. Which answer figure will complete the pattern in


the question figure?
43. Which of the following answer figure patterns can
be combined to make the question figure?

47. From the given answer figures, select the one in


which the question figure is hidden / embedded.

Engineer’s platform
can be represented by 78, 97 etc. Similarly. You have to
identify the set for the word ‘INKY’.

Matrix I Matrix II
0 1 2 3 4 5 6 7 8 9
0 F C K M C 5 Y S V N W
1 H A C A B 6 P X O Z Z
2 J F K G B 7 Y V Y Q Y
48. A piece of paper is folded and punched as shown
3 J B I F J 8 R Q U Q O
below in the question figures. From the given answer 4 E F B H B 9 W R Q T X
figures, indicate how it will appear when opened.
(a) 32, 58, 02, 75 (b) 34, 85, 98, 10
(c) 43, 89, 04, 55 (d) 11, 67, 03, 65

49. If a mirror is placed on the line MN, then which of


the answer figures is the right image of the given
figure?

ANSWERS:

50. A word is represented by only one set of numbers as 1. (a) 2. (a) 3. (b) 4. (c) 5. (a) 6. (c) 7. (b)
given in any one of the alternatives. The sets of 8. (a) 9. (a) 10. (a) 11. (a) 12. (d) 13. (b) 14. (c)
numbers given in the alternatives are represented by 15. (d) 16. (d) 17. (d) 18. (d) 19. (a) 20. (b) 21. (a)
two classes of alphabets as shown in the given two 22. (b) 23. (a) 24. (b) 25. (b) 26. (c) 27. (b) 28. (a)
matrices. The columns and rows of Matrix-I are 29. (a) 30. (b) 31. (c) 32. (b) 33. (d) 34. (c) 35. (c)
numbered from 0 to 4 that of Matrix-II are numbered 36. (b) 37. (d) 38. (b) 39. (c) 40. (d) 41. (c) 42. (d)
from 5 to 9. A letter from these matrices can be 43. (d) 44. (c) 45. (a) 46. (c) 47. (a) 48. (c) 49. (b)
represented first by its row and next by its column, for 50. (a)
example ‘J’ can be represented by 34, 20 etc. and ‘Q’

Engineer’s platform
EXPLANATIONS: - 10. Forest, the dwelling place of wild animals, is called
forest. Other options include the names of wild animals.
1. Badminton: Court → [Sport: Location of the game]
Similarly, Skating: Rinks? [Sports: Location of game] 12. Writing, reading and listening is physical action.
Whereas the color is different from them.
2. Teacher: School → [Employee: His work place]
Similarly, Artist: Theater → [ employee: his work place] 13.
CHM C(+5) = H H(+5)=M M
4. R(+6) = X JOS J(+5) = O O(+4)=S(Different) S
RATS A(+6) = G XGZY, SXC S(+5) = X X(+5)=C C
T(+6) = Z VAF V(+5) = A A(+5)=F F
S(+6) = Y

14.
R(+6) = X
Similarly, RIPE I(+6) = O XOYK, KQW K(+6)=Q Q(+6)=W W
BHN B(+6)=H H(+6)=N N
P(+6) = Y
RXF R(+6)=X X(+8)=F(Different) F
E(+6) = K
DJP D(+6)=J J(+6)=P P

5. FEAR:PCCH::VANS:QPYX 15.
P(+6) = V FBX F(-4)=B B(-4)=X X
PCCH C(-2) = A VANS, PLH P(-4)=L L(-4)=H H
C(+11)= N VRN V(-4)=R R(-4)=N N
H(+11)= S GCZ G(-4)=C C(-3)=Z(Different) Z

R(+6) = X 16. 17 – 289 = [17 x 17 = 289]


Similarly, FEAR A(-2) = Y QPYX, 26 – 676 = [26 x 26 = 676]
E(+11) = P 27 – 729 = [27 x 27 = 729]
F(+11) = Q 14 – 170 = [14 x 14 = 196]

6. L(+4) = P 17. 13 – 16 = [ 13 -16 = -3]


LAMP A(-3) = X PXQM, 19 – 22 = [ 19 - 22 = -3]
M(+4) = Q 21 – 24 = [ 21 - 24 = -3]
P(-3) = M 33 – 38 = [ 33 - 38 = -5] (Answers.)

21 3
C(+4) = G 18. 14 − 21 → [ = ]
14 2
Similarly, COKE O(-3) = L WMSW,
K(+4) = O 𝟐𝟖.5 3
19 − 𝟐𝟖. 5 → [ = 2]
E(-3) = B 19

𝟐𝟓.5 3
7. 43:7 = [4 + 3 = 7] 17 − 𝟐𝟓. 5 → [ = ]
17 2
Similarly, 61:7 = [6 + 1 = 7]
46 23
36 − 46 → [36 = 18] (𝐴𝑛𝑠𝑤𝑒𝑟)
8. 24:164 = [24 × 6 ÷ 20 = 164]
𝟑𝟐: 𝟐𝟏𝟐 = [𝟑𝟐 × 𝟔 ÷ 𝟐𝟎 = 𝟐𝟏𝟐](𝒂𝒏𝒔𝒘𝒆𝒓. )

Engineer’s platform
19. (5). Literate (2). Living (1). Loathe (4). Local 163 ↔ [163 + 91 = 254]
(3). Locate 254

20. Naïve, Naked, Narrate, Narrow, Nasty 29. X said to Y-


My sister’s father-in-law (X’s sister’s husband’s father)
21. Proceed, Prodigy, Producer, Product Your husband’s father. (Y’s husband’s father)
Because the father of X’s sister’s husband and fathr of
22. PGD (Answer.) Y’s husband is the same. Therefore Y, the wife of X’s
RIF R(-9)=I I(-3)=F sister’s husband’s brother.
KBY K(-9)=B B(-3)=Y
DUR D(-9)=U U(-3)=R 30. In the word ulcer, e is used, which is not in the given
WNK W(-9)=N N(-3)=W letter. So the answer will be ulcer.
PGD P(-9)=G G(-3)=D
31.
O B S C U R E
23. EK(Answer.)
(-1) (-1) (-1) (-1) (-1) (-1) (-1)
PV P(+6)=V N A R B T Q D
UA U(+6)=A Similarly,
ZF Z(+6)=F
T H Y
EK E(+6)=K (SGX) ans.
(-1) (-1) (-1)
JP J(+6)=P
S G X

24. [F(+4)=J]
32. 1259= box in the ring …(1)
[J(+5)=O]
6941= the square boxing ring …(2)
[O(+4)=S]
5713= put the box out …(3)
[S(+5)=X]
From equation (2) and (3)
[X(+4)=B]
1 → the
[B(+5)=G]↔G
From equation (1) and (2)
9 → ring (Answer)
25. 14, 16, 30, 45, 62, 132, 126, 393,
14, 30, 62, 126 →
33. (+) → (-), (-) → (X), (X) → (+), (+) → (+)
14 → [14 x 2 + 2 = 30]
On changing the, sign according to instenction.
30 → [30 x 2 + 2 = 62]
(160 × 40 ÷ 20 + 10 − 2)
62 → [62 x 2 + 2 = 126] 160
16,45,132,393? (160 ÷ 40 + 20 − 10 × 2) = ( ) + 20 − (10 × 2)
40
16 → [16 x 3 - 3 = 45]
45 → [45 x 3 - 3 = 132] = 4 + 20 – 20
132 → [132 x 3 - 3 = 393] =4

26. 6, 25, 62, 123, 214, 341 34. 50%15=70[(50-15)X2=70]


23%20=6[(23-20)X2=6]
27. 53 10%4=[(10-4)X2=12] (answer)
19 ↔ [19 + 53 =72]
72 ↔ [72 + 19 =91]
91 ↔ [91 + 72 = 163](answer.)

Engineer’s platform
35. Condition (1) → A * B = A is the mother of B. So E * F 39.
= E is the mother of F. E is mother.
Condition (2) → A % B = A is the daughter of B. So D % E
= D is the daughter of E, So D is the sister of F.
Condition (3) → A + B = A is the sister of B. So C + D = C
is the sister of D. So C is the sister of F.

36. 0 + 2 = 2,
9 + (-2) = 7
-6 + 10 = 4

37. The order of ‘O’ is moving forward from the back.

X and Y begin to move from point O. X move 25 km


from O to A, then 14 km from A to B. Y move 19 km
from O to D, than 3 km from D to E, than 4 km from E to
F, then 17 km from F to G.
→ BG = AD + EF
= AO + OD + EF
= 25 + 19 + 4
= 48 Km
38. Hence Y is 48 km south of X.

Bird moves 3 km from A to B, than 4 km from B to C,


then 11 km from C to D, then 4 km from D to E.
→ AE = BE - BA
= CD - BA (BE = CD)
= 11 - 3 = 8 km
Hence bird is 8 km south from the nest.

Engineer’s platform
Solved Paper 2018 (Set-5) 8. In the following question, select the related number
1. In the following question, select the related word from the given alternatives.
pair from the given alternatives. 63: 18: 42:?
Court: Judge: :?:? (a) 32 (b) 8
(a) School: Teacher (b) Factory: Wages (c) 16 (d) 12
(c) Company: Shares (d) Restaurant: Food
9. In the following question, select the related number
2. In the following question, select the related word from the given alternatives.
from the given alternatives. 7:346::9:?
AC: Cool:: Heater: ? (a)4 515 (b) 343
(a) Electricity (b) Ray (c) 732 (d) 636
(c) Heat (d) Shine
10. In the following question, select the odd word from
3. In the following question, select the related word the given alternatives.
from the given alternatives. (a) River (b) Sea
Milk: Drink: : Food:? (c) Lake (d) Land
(a) Drink (b) Hot
(c)Table (d) Eat 11. In the following question, select the odd word from
the given alternatives.
4. In the following question, select the related letters (a) Eagle (b) Sparrow
from the given alternatives. (c) Rabbit (d) Parrot
DRUM: JXAS: : CROP:?
(a) AZSN (b) IXUV 12. In the following question, select the odd word from
(c)UNSV (d) JYVX the given alternatives.
(a) Atlantic Ocean (b) Australian Ocean
5. In the following question, select the related letters (e) Pacific Ocean (d) Indian Ocean
from the given alternatives.
TOMB: 20MV:: LYMP:? 13. In the following question, select the odd letters
(a) PMSP (b) FNCF from the given alternatives.
(c) NOWN (d) NSHNN (a) DGJ (b) QTW
(c) RUY (d) XAD
6. In the following question, select the related letters
from the given alternatives. 14. In the following question, select the odd letters
STOP: WQSM: : MORK:? from the given alternatives
(a) QLVH (b) SLMH (a) WTQ (b) YVS
(c) QPVS (d) QLMT (c) TQN (d) FCX

7. In the following question, select the related number 15. In the following question, select the odd letters
from the given alternatives. from the given alternatives.
41: 62::37:? (a) SNI (b) PKE
(a) 60 (b) 58 (c) FAV (d) YTO
(c) 74 (d) 82
16. In the following question, select the odd number
pair from the given alternatives.
(a) 41-54 (b) 59-72
Engineer’s platform
(c) 54-77 (d) 66-79 (c) SV (d) RI

17. In the following question, select the odd number 24. A series is given with one term missing. Select the
from the given alternatives. correct alternative from the given ones that will
(a) 16-24 (b) 17-23.5 complete the series.
(c) 18-27 (d) 19-28.5 L, G, B,?, R, M
(a) A (b) V
18. In the following question, select the odd number (c) W (d) X
pair from the given alternatives.
(a) 22-55 (b) 26-65 25. In the following question, select the missing
(c) 30-85 (d) 38-95 number from the given series.
76, 23, 99, 122, 221,?
19. Arrange the given words in the sequence in which (a) 447 (b) 528
they occur in the dictionary. (c) 343 (d) 329
1. Consideration 2. Consist
3. Conquer 4. Connect 26. In the following question, select the missing
5. Confusion number from the given series.
a) 45312 (b) 54312 46, 235, 945, 2840, 5685,?
(c) 31542 (d) 13542 (a) 7218 (b) 6380
(c) 7445 (d) 5690
20. According to dictionary, which of the following
word will come at THIRD position? 27. In the following question, select the missing
1. Emergence 2. Eminent number from the given series
3. Eligible 4. Element 13, 13, 19.5, 39, 97.5,?
5. Elated (a) 393.5 (b) 191
a) Eligible (b) Eminent (c) 312 (d) 292.5
(c) Element (d) Emergence
28. Five cars are parked in a line. Honda is between
21. From the given alternatives, according to BMW and Suzuki, Toyota is ahead of Mercedes. Suzuki
dictionary, which word will come at LAST position? is ahead of Toyota. Which is the second car in the line?
(a) Hobble (b) Historic (a) BMW (b) Suzuki
(c) Hinder (d) Hold (c) Honda (d) Toyota

22. A series is given with one term missing. Select the 29. At a wedding A said to B that the groom was her
correct alternative from the given ones that will sister's son. B said that the bride was the daughter in-
complete the series. law of his brother. How are A and B related to each
MOB, UWJ, CER, KMZ, ? other?
(a) IMV (b) LKF (a) B is A's brother's wife's sister
(c) SUH (d) FIR (b) A is B's wife's brother's sister.
(e) A is B's brother's wife's sister.
23. A series is given with one term missing. Select the d) B is A's wife's brother's sister.
correct alternative from the given ones that will
complete the series. 30. From the given alternative words select the word
FQ, JW, NC, ? which cannot be formed using the letters of the given
(a) OP (b) QH word.
Engineer’s platform
FORFEITURE (c) -1 (d) 14
(a) forte (b) urate
(c) route (d) fruit 37. Which of the following terms follows the trend of
the given list?
31. If CONTEND is coded as XLMGVMW, then how will ABACBABAB, ABCABABAB, ACBABABAB,
SAY be coded as? CABABABAB, ABABABABC,__________
(a) HZB (b) UCA (a) ABABABCAB (b) ABABACBAB
(c) RZX (d) TBZ (c) ABABCABAB (d) ABABABACB

32. In a certain code language, 8193 means 'sow and 38. A auto picks a passenger and travels West 3.5 km,
you reap', 9862 means 'you and I go' and 8534 means then it turns to its left and rides for another 2.5 km,
pig and sow eat. Find the code for "sow'. then it turns East and rides for 6.5 km, then it turns to
(a) 8 (b) 1 its left and rides 2.5 km. Where is the auto now with
(c) 9 (d) 3 respect to its starting position?
(a) 3 km East (b) 3 km West
33. In a certain code language, '+’ represents ‘-’, ‘-’ (c) 10 km East (d) 10 km West
represents ’ x', ‘x' represents ’/' and ‘/’ represents ‘+’.
Find out the answer to the following question. 39. Two taxis start from the same stand. Taxi A travels
120 x 8 – 25 / 36 + 6 =? 10 km North, then turns to its right and travels a
(a) 965 (b) 405 further 7 km. Meanwhile Taxi B travels 6 km West,
(c) 25 (d) 3 then Turns South and travels 5 km, then it turns to its
left and travels 13 km. Where is Taxi B with respect to
34. If 19 $ 266 = 14 and 8 $ 160 = 20, then find the Taxi A?
value of 9 $ 54 =? (a) 15 km North (b) 15 km South
(a) 6 (b) 6 (c) 5 km South (d) 5 km North
(c) 45 (d) 3
40. In the question two statements are given, followed
35. If A $ B means A is son of B, A # B means A is by two conclusions, I and II. You have to consider the
brother of B and If A * B means A is father of B, then statements to be true even if it seems to be at
what does X * Y # Z $ W mean? variance from commonly known facts. You have to
(a) X is son's son of Z decide which of the given conclusions, if any, follows
(b) Z is father's father of Z from the given statements.
(c) X is son of Z Statement I: All sky is blue
(d) X is father of Z Statement II: Some red is sky
Conclusion I: Some red is blue
36. Select the missing number from the given Conclusion II: All red is blue
responses. (a) Only conclusion I follows
(b) Only conclusion II follows
(c) Both conclusions I and II follow
(d) Neither conclusion I nor conclusion II follows

41. In the question three statements are given,


followed by three conclusions, I, II and II. You have to
consider the statements to be true even if it seems to
(a) -6 (b) 1
be at variance from commonly known facts. You have
Engineer’s platform
to decide which of the given conclusions, if any, 44. In the following figure, square represents doctors,
follows from the given statements. triangle represents artists, circle represents circle
Statement I: All race is running represents weight-lifters and rectangle represents
Statement II: Some running is sports Russians. Which set of letters represents Russians who
Statement III: Some race is athletics are not doctors?
Conclusion I: Some running is athletics
Conclusion II: All athletics is sports
Conclusion III: Some running is race
(a) Only conclusions I and II follow
(b) Only conclusions II and III follow
(c) Only conclusions I and III follow
(d) All conclusions I, II and III follow

42. Which of the following cube in the answer figure (a) EF (b) HB
cannot be made based on the unfolded cube in the (c) GE (d) DG
question figure?
45. Which of the following Venn diagram represents
the relationship between Doctors, Men and Indians?

43. Which of the following answer figure patterns can 46. Which answer figure will complete the pattern in
be combined to make the question figure? the question figure?

Engineer’s platform
47. From the given answer figures, select the one in 50. A word is represented by only one set of numbers
which the question figure is hidden/embedded. as given in any one of the alternatives. The sets of
numbers given in the alternatives are represented by
two classes of alphabets as shown in the given two
matrices. The columns and rows of Matrix-I are
numbered from 0 to 4 and that of Matrix-II are
numbered from 5 to 9. A letter from these matrices
can be represented first by its row and next by its
column, for example 'G’ can be represented by 31, 13
etc. and ‘X’ can be represented by 89, 75 etc. Similarly,
you have to identify the set for the word 'AWRY’.

48. Apiece of paper is folded and punched as shown


below in the question figures. From the given answer
figures, indicate how it will appear when opened.

(a) 21, 78, 42, 56 (b) 32, 54, 98,31


(c) 33, 77, 21, 12 (d) 20, 96, 56,77

49. If a mirror is placed on the line MN, then which of


the answer figures is the right image of the given
figure?

Answers
1.(a) 2.(c) 3.(d) 4.(b) 5.(c) 6.(a) 7.(b)
8.(b) 9.(c) 10.(d) 11.(c) 12.(b) 13.(c) 14.(d)
15.(b) 16.(c) 17.(b) 18.(c) 19.(b) 20.(a) 21.(d)
22.(c) 23.(d) 24.(c) 25.(c) 26.(d) 27.(d) 28.(c)
29.(a) 30.(b) 31.(a) 32.(d) 33.(b) 34.(a) 35.(d)
36.(a) 37.(d) 38.(c) 39.(b) 40.(a) 41.(c) 42.(d)
43.(c) 44.(d) 45.(b) 46.(d) 47.(a) 48(d) 49.(c)
50.(d)
Engineer’s platform
Explanation 10. There is water in the river, sea and lake, While soil in
2. AC: Cool ⟹ AC is used to cool the room. Heater: the ground.
Heat? Heater is used to heat the room.
11. Eagles, birds and parrots, they can fly in the air,
3. Milk: Drink⟹ Milk is drunk. Food: Eat⟹ The food is While the rabbits do not fly.
eaten
12. Australian Ocean is not the ocean.
4. D → |D(+6)=J| → J
R → |R(+6)=X|→ X 13.
U → |U(+6)=A|→ A DGJ? D+3=G G+3=J J
M→ |M(+6)=S|→ S QTW? Q+3=T T+3=W W
RUY? R+3=U U + 4 = Y (Different) Y
5. TOMB: ZOMV XAD? X+3=A A+3=D D
LYMP: NOWN
14.
WTQ→ W (– 3) = T T (– 3) = Q I
YVS→ Y (– 3) = V V (– 3) = S E
TQN→ T (– 3) = Q Q (– 3) = N V
FCX→ F (– 3) = C C (– 5) = X |(Different) O

15.
SNI S(-5)=N N(-5)=I I
PKE P(-5)=K K(-6)=E (Different) E
FAV F(-5)=A A(-5)=V V
YTO Y(-5)=T T(-5)=O O
6.

16. 41-54 = [41-54 = -13]


59-72 = [59-72= -13]
66-69 = [66-69= -13]
54-77 = [54-77 = 23] (Answer)

17.
3
16 − 24 = [16 ∗ = 24]
2
3
18 − 27 = [18 ∗ = 27]
2
7. 41:62 = [41-62 = -21], 3
Similarly, 37:58 = [37-58 = -21] 19 − 𝟐𝟖. 5 = [19 ∗ = 𝟐𝟖. 5]
2
𝟑
𝟏𝟕 − 𝟐𝟑. 𝟓 = [𝟏𝟕 ∗ 𝟐 = 𝟐𝟓. 𝟓] (Different)
8. 63:18= [6*3 = 18],
Similarly, 42:8 = [4*2 = 8] 22 26 38
18. 55
= 65 = 95
5
9. 7:346 [ 73 +3 = 346] = ( ) (EQUAL ANSWER)
2
Similarly, 9:732 = [ 93 + 3 = 732 ] 30 6
=
85 17
(DIFFERENT ANSWER)

Engineer’s platform
19. (5).Confusion (4).Connect (3).Conquer 28.
(1).Consideration (2).Consist Toyota is ahead Suzuki is ahead Honda is between
of Mercodes of Toyota BMW & Suzuki
20. Elated, Element, Eligible, Emergence, Eminent
Mercodes→Toyota→Suzuki→ 𝐻𝑜𝑛𝑑𝑎 → 𝐵𝑀𝑊 → 𝐷𝐼𝑅𝐸𝐶𝑇𝐼𝑂𝑁
21. Hinder, Historic, Hobble, Hold. Honda

29. A, said to B- Groom was the son of his sister (A's


sister's son)
B said to A- The bride, the wife of his brother's daughter
(the brother of B's ??brother) = that is the wife of B's
brother's son. (son of A’s sister) Therefore, A is the
sister of B's ??brother's wife.

31. Alphabetical order-


(on further reading)
CONTEND = 3,15,14,20,5,14,4
24. L+5 = G,
G+5 = B, Similarly SAY = 19, 1, 25
B+5 = W,
(𝑟𝑒𝑎𝑑𝑖𝑛𝑔 𝑓𝑟𝑜𝑚 𝑏𝑎𝑐𝑘 𝑠𝑖𝑑𝑒
W+5 = R, XLMGVMW = 3,15,14,20,5,14,4
R+5 = M
Similarly HZB = 19, 1, 25
25. 23 = [23+76 = 99]
99 = [99 + 23 = 122] 32. 8193 = Sow and you rap …………. (1)
122 = [122+99=221] 9862 = you and I go .……………. (2)
221 = [221 + 122 = 343] 8534 = pig and sow eat ……………….(3)
Answer: 343 From (2) & (3), 8 => and
From (1) & (3), 3 = sow (answer).
26. 46 = [46 * 5+ 5 = 235]
235 = [235 * 4+5=945] 33. (+) = (-), (-)= (X), (X)= (÷), (÷)= (+)
945 = [1945 * 3+5 = 2840] Changing the sign according to instruction
2840 = [2840 * 2+5 5685] (120 X 8 – 25 ÷ 36 + 6)
5685 = [5685 * 1+5 = 5690] 120
(120 ÷ 8 X 25 + 36 - 6) = ( 8
)𝑋 25 + 36 − 6
Answer: 5690
= 15 X 25 + 30 = 405
3
27. 13 = [13 ∗ 2 = 𝟏𝟗. 5]
34. 19 $ 266 = 14 => |266/19= 14]
4 8 $ 160 = 20 => [160/8 = 20]
𝟏𝟗. 5 = [𝟏𝟗. 5 ∗ = 39]
2 9 $ 54 = 6 =>[54/9= 6](answer)
5
39 = [39 ∗ = 9𝟕. 5]
2
6 35. Condition (1) => A*B= A is the father of B. So X*Y = X
9𝟕. 5 = [9𝟕. 5 ∗ = 29𝟐. 5] is the father of Y. X is the father.
2
Answer: 292.5 Condition (2) => A#B =A is brother of B. So Y #Z = Y is Z's
brother. So X is also the father of Z.
Condition (3) => A$B =A is the son of B So Z$W

Engineer’s platform
= Z is the son of W. A and B begin to move from point O. A move 10 km
from O to P then 7 km from P to Q. B move 6 km from O
36. 7-4 =3 to R, then 5 km from R to S, than 13 km from S to T.
4-10 = -6 (answer)  QT = OP + RS
1-5 = -4 = 10 + 5 = 15 km
Hence Taxi B 15 km South of A.
37. The ordering of 'C' is progressing forward in each
term. 40. Option (1)

38. 41. Option (3)

Auto begin to move 3.5 km from A to B, then 2.5 km


from B to C, than 6.5 km from C to D, then 2.5 km from
D to E.
 AE = BE-BA 50. AWRY= 20, 96, 56, 77
= CD-BA (BE = CD)
=6.5-3.5
= 3km
Hence auto is 3 km east from the starting point.

39.

Engineer’s platform
Solved Paper 2017 (Set-1)
1. Select the related word/letters/number from the 8. Select the related word/letters/number from the
given alternatives. given alternatives.
Quarter : Half : : Single : ? 586 : 364 : : 824 : ?
(a) Multiple (b) Double (a) 520 (b) 740
(c) Set (d) Group (c) 442 (d) 602

2. Select the related word/letters/number from the 9. Select the related word/letters/number from the
given alternatives. given alternatives.
Ball : Sun : : Cigarette : ? 224 : 112 : : 326 : ?
(a) Pipe (b) Smoke (a) 189 (b) 146
(c) White (d) Fire (c) 163 (d) 176

3. Select the related word/letters/number from the 10. Select the odd word/letters/number/word
given alternatives. pair/number pair from the given alternatives.
Allot : a lot : : ? (a) Cheek (b) Toe
(a) Already : All ready (c) Ear (d) Nostril
(b) Sea : See
(c) Than : Then 11.Select the odd word/letters/number/word
(d) Peace : Piece pair/number pair from the given alternatives.
(a) Socks and Shoes
4. Select the related word/letters/number from the (b) Bow and Tie
given alternatives. (c) Vest and Shirt
HFDB : GEC : : YWUS : ? (d) Shirt and Coat
(a) XVT (b) TVX
(c) RTVX (d) XVTR 12.Select the odd word/letters/number/word
pair/number pair from the given alternatives.
5. Select the related word/letters/number from the (a) Wood and Chair (b) Cotton and Pillow
given alternatives. (c) Cotton and Wool (d) Fur and Pillow
Grace : Career : : Sitar : ?
(a) Saris (b) Artist 13.Select the odd word/letters/number/word
(c) Stair (c) Star pair/number pair from the given alternatives.
(a) RF (b) AH
6. Select the related word/letters/number from the (c) IM (d) OT
given alternatives.
Destine : Inertia : : Bison : ? 14.Select the odd word/letters/number/word
(a) Sonnets (b) Sonar pair/number pair from the given alternatives.
(c) Cosine (d) Brine (a) Busted (b) Blasted
(c) Disaster (d) Caste
7. Select the related word/letters/number from the
given alternatives. 15.Select the odd word/letters/number/word
99 : 18 : : 77 : ? pair/number pair from the given alternatives.
(a) 16 (b) 20 (a) ABB (b) EFF
(c) 12 (d) 14 (c) OPP (d) HII

Engineer’s platform
16. Select the odd word/letters/number/word 23. In the given series one word/one term/one number
pair/number pair from the given alternatives. is missing. Select the correct alternative from the given
(a) 428 (b) 802 ones that will complete the series.
(c) 640 (d) 537 Kpu, jot, ins, hmr, ?
(a) gnq (b)fns
17.Select the odd word/letters/number/word (c) fop (d) glq
pair/number pair from the given alternatives.
(a) 81 (b) 72 24. In the given series one word/one term/one number
(c) 44 (d) 63 is missing .Select the correct alternative from the given
ones that will complete the series.
18.Select the odd word/letters/number/word XXXXOXOX, XXXOXOX, XXOXOXX, XOXOXXX ?
pair/number pair from the given alternatives. (a) XOXOXXX (b) XXXXOXO
(a) 136,17 (b) 152,19 (c) XXXOXOX (d) OXOXXXX
(c) 160,20 (d) 148,18
25. In the given series one word/one term/one number
19. In the given series one word/one term/one number is missing .Select the correct alternative from the given
is missing. Select the correct alternative from the given ones that will complete the series.
ones that will complete the series. -7, -3, ?, 8, 15
DDRFTG,CVVBHT,DCERRT,? (a) 1 (b) 2
(a) REDCVT (b) DEEDCR (c) 3 (d) 5
(c) SDDAEW (d) SDERWW
26. In the given series one word/one term/one number
20. In the given series one word/one term/one number is missing .Select the correct alternative from the given
is missing. Select the correct alternative from the given ones that will complete the series.
ones that will complete the series. 2, 1.25, ?, -1/4, -1
Super, Erratic, Icebox, Oxen, ? (a) -1/2 (b) ½
(a) Lollipop (b) Popcorn (c) -0.75 (d) 0.75
(c) Papyrus (d) Envelop
27. In the given series one word/one term/one number
21. In the given series one word/one term/one number is missing. Select the correct alternative from the given
is missing. Select the correct alternative from the given ones that will complete the series.
ones that will complete the series. 13, ?, 19, 23, 29,
Ran, anon, clasp, rooted, ? (a) 14 (b) 15
(a) confuses (b) accolade (c) 16 (d) 17
(c) revamp (d) scoured
28. In m > l, m< n and n < 0, which of the following
22. In the given series one word/one term/one number Must be true ?
is missing. Select the correct alternative from the given l. l < 0 ll. n > 1
ones that will complete the series. (a) Only l (b) Both l and ll
Y, U, Q, M, ? (c) only ll (d) Neither l nor ll
(a) J (b) K
(c) L (d) I 29. The weight of five boxes are 20, 30, 40, 70 & 90
kilograms. Which of the following cannot be the total
weight in kilogram, of any combination of these boxes?
(a) 180 (b) 190
Engineer’s platform
(c) 200 (d) 170 FEEFEE, EFEEFE, EEFEEF, ?
(a) FFEEFF (b) EFEFEEF
30. If the letters E, D, X, S, P & O are numbered 1, 2, 3, (c) FEEFEE (d) FEFFEE
4, 5 & 6 respectively. Select that combination of
numbers so that letters arranged accordingly, from a 38. A postman cycles 4 km West, then turns South and
meaningful word. cycles 7 km, the turns East and cycles 4 km, the turns to
(a) 3532143 (b) 1356412 his left and cycles 10 km where is he now with
(c) 6121341 (d) 4321242 reference to his starting position.
(a) 17 km North from the starting position
31. IFDISCERNS is coded as WRHXVIMH, then how will (b) 3 km North from the starting position
AGO be coded as ? (c) 3 km South from the starting position
(a) TGB (b) BGT (d) 17 cm South from the starting position
(c) DFG (d)ZTL
39. Two persons A and B start from the same point A
32. In a certain code language, 639 means ‘wood is walks 4 m North, then turns to his right and walks 3m.
hard’, 657 means ‘fur is soft’ , 135 means ‘hard of soft’, At the same time, B has walked 6 m, Where is B now
find the code for ‘or’. with respect to the position of A?
(a) 4 (b)1 (a) B is the 6 m to the East of A
(c) 6 (d) 3 (b) B is 12 m to the East of A
(c) B is 6 m to the West of A
33. In a certain code language, ‘+’ represents ‘*’, ‘-‘ (d) B is 12 m to the West to A
represents ‘+’, ‘*’ represents ‘ ÷’ and ‘÷’ represent ‘-‘ .
Find out the answer to the following question. 40. In the question two statements are given following
(a) 11 (b) 89 by two conclusions I and II. You have to consider the
(c) 1.25 (d) 66 statements to be true even if it seems to be at
variance from commonly known facts. You have to
34. If A @ B means A is father of B, A # B means A is decide which of the given conclusions , if any, follows
brother of B and A! B means A is son of B then what from the given statements.
does L # M @ N ! O mean? Statement 1 : Some villagers are poor
(a) Mis husband of O (b) N is uncle of L Statement 2 : All poor are nice people
(c) L is brother of O (d) L is father of O Conclusion l : Some nice people are villagers.
Conclusion ll : No villagers are nice people.
35. If 62 # 51 =0, 82 # 54 = 5 then find the value of 72 # (a) Only conclusion l follows
66 = ? (b) Only conclusion ll follows
(a) 7 (b) 10 (c) Either I nor ll follows
(c) 12 (d) 5 (d) Neither l nor ll follows

36. Which of the following words follows the trend of 41. In the question a statement is given followed by two
the given list? arguments, l and ll. you have to consider the statement
Visual, Avarice, Savour, Salvage, Saliva, ? to be true even if it seems to be at variance from
(a) Drive (b) Reweave commonly known facts. You have to decide which of
(c) Eleven (d) Envelop the given arguments, if any, is a strong argument.
Statement: Should local train tickets be subsidized?
37. Which of the following terms follows the trends of Argument l: No, In an environment where local trains
the given list? are overcrowded, subsidy will worsen the situation.
Engineer’s platform
Argument ll: Yes, Local trains reduce number of vehicles players who do not play football and cricket players
on the road, thus reducing pollution reduce costs and who play kabaddi?
travel time.
(a) if neither argument l nor ll is strong
(b) if only argument ll is strong
(c) if only argument l is strong
(d) if both argument l and ll are strong (a) O, U and P (b) R, S and U
(C) O, S and U (d) Q, P and T
42. Which of the following cube in the answer figure
cannot be made based on the unfolded cube in the 46. Which answer figure will complete the pattern in
question figure? the question figure ?

43. Which of the following answer figure patterns can


be combined to make the question figure?
47. From the given answer figures, select the one in
which the question figure is hidden/embedded.

44. Which of the following diagrams represent the


relationship between Chinese, Indians and Asians?

48. A piece of paper is folded and punched as shown


below in the question figures. From the given answer
figures, indicate how it will appear when opened

45. In the following figure, square represents hockey


players, triangle represents cricket players, circle
represents kabaddi players and rectangle represents
football payers. Which set of letters represents hockey

Engineer’s platform
(a) 23, 95, 14, 20, 79
(b) 40, 31, 41, 68, 01
(c) 11, 41, 44, 85, 22
(d) 21, 40, 04, 69, 01

49. If a mirror is placed on the line MN, then which of


the answer figures is the right image of the given
figure?

50. A word is represented by only one set of numbers as


given in any one of the alternatives. The sets of
numbers given in the alternatives represented by two
classes of alphabets as shown in the given two matrices.
The columns and rows of Matrix-l are numbered from 0
to 4 and the of Matrix-ll are numbered from 5 to 9. A
letter from these matrices can be represented first by
its row and next by its column, for example ‘Z’ can be
represented by 20, 30 etc and ‘A’ can be represented by
65, 57, etc. Similarly, you have to identify the set for the
word ‘TOWER’.
MATRIX–l

Answer:

1.(b) 2.(a) 3.(a) 4.(a) 5.(b) 6.(a) 7.(d)


8.(d) 9.(c) 10.(b) 11.(b) 12.(a) 13.(a) 14.(a)
15.(d) 16.(d) 17.(c) 18.(d) 19.(d) 20(d) 21.(d)
22(d) 23(d) 24.(d) 25.(b) 26.(a) 27.(d) 28.(b)
29.(d) 30.(b) 31.(d) 32.(b) 33.(b) 34.(a) 35.(d)
36.(b) 37.(c) 38.(b) 39.(a) 40(a) 41.(d) 42.(a)
43.(b) 44.(b) 45.(c) 46.(c) 47.(a) 48.(c) 49.(d)
50.(b)

Engineer’s platform
EXPLANATIONS: 11. Socks and shoes are related. Shoes are worn after
socks. Socks and shoes are related. Shoes are worn
1. Quarter + Quarter = Half and single + single double. after socks. Similar relation made in vest shirt and shirt
Hence the right answer is Double. coat. This type of relationship is not made in bow and
2. Here the ball can be compared to the sun by its tie, so the correct answer is bow and tie
shape. The shape of the ball and the sun is round.
12. Cotton, pillow fur and wool, are all soft and
Cigarettes can be compared to pipes. The shape of lightweight. While both wood and chair are hard and
both of them is cylindrical. Thus the correct answer will heavy in weight. So the right answer would be wood
be pipe. and chair.
3. The last word in Allot and A lot is lot, which means 13. According to the alphabet, in Aand H, Acomes first,
very much. Similarly, the last word in Already and all then Hcomes. Somilarly in I and M, I comes first, then
Ready is ready which means keep or quick to give. Mcomes. In also O nd T, O comes first, then M comes. In
Combination is not made in other options. So the
also O and T, O comes first,then T comes. Whereas
correct answer is Already: All ready. before R and F, F comes, then R comes. So the correct
4. HFDB: GEC = A B C D E F GH I In this you can see that answer would be RF.
G, E, C is in the middle of HF,FD, and DB, respectively. 14.In Busted, Blasted, Disaster and Caste, Cast , Caste
Similarly, YWUS : XVT = S T U V W X Y. In this you can
will be different. Because caste represents a group of
see that X, V, T is located in the middle of YW, WU and humans. While busted, blasted, disaster the unpleasant
US, respectively: This type of relationship is not made in event. Therefore the correct answer would be Caste.
other options. So the correct answer would be XVT.
16. 428, 802 and 640. This is even numbers because it is
5. Grace : Career : = 5 : 6 (Ratio of the number of totally divided form 2. Whereas 537 is an odd number,
letters), similarly, Sitar : Artist = 5 : 6
because it is not completely divided by 2. So the correct
6. In this sequence the last 3 letters to the word are answer is 537
the starting letteres of the next word. For example 17. 81, 72,and 63. This number is totally divided from 9.
’ine’of ‘ Destine ‘ is the starting letter of ‘Ine’rtia. While 44, is not competely divided by 9. So the corret
Similarly ‘son’of ‘Bison ‘ is the starting letter of ‘Son’ar. answer would be 44.
7. 99 : 18 ⇒9+9=18 18.
Similarly 77 : 14, 7 + 7=14, So the correct answer is 14.

8. 586 : 364 ⇒ 586 − 364 =222 Similarly 824 :602⇒824-


602=222, so the carrect answer is 602 19. DDRFTG = 1st and 2nd position is the same.
9. 224 : 112 ⇒ 224 − 112 = 112 𝑜𝑟112 + 112 = 224 CVVBHT=2nd and 3rd position is the same.
𝑆𝑖𝑚𝑖𝑙𝑎𝑟𝑙𝑦 326 ∶ 163 ⇒ 326 − 163 DCEERT=3rd and 4th position is the same.
= 163 𝑜𝑟 163 + 163
= 326 𝑠𝑜 𝑡ℎ𝑒 𝑐𝑜𝑟𝑟𝑒𝑐𝑡 𝑎𝑛𝑠𝑤𝑒𝑟 𝑖𝑠 𝟏𝟔𝟑 DCFRRT=4th and 5th position is the same.

10. When we look at cheeks, ears and nostrils, we both Therefore, the final position would be that
the skin if the body, whereas the toe shows both the which would be similar to the 5th and 6th
bone and the skin. So the right answer would be Toe. positions. The correct answer is SDERWW.

Engineer’s platform
20. In this sequence, the last two letters of the word are 28. m > 1, m < n and n < 0 this equation can be written
the starting letters of the next word. For as follows.
example ‘er’of ‘Super’ is the starting letter of
‘Erratic’ similarly ‘en’ of ‘Oxen’ is the starting (1 < m < n < 0 ) = I are smaller than m, n and o. so both
letter of ‘Envelope’. the conclusion are correct.

29. 180 = 90 + 70 + 20
21. In this question the number of letters of the words
is increasing gradually. As such, the number of 190= 70 + 30 + 90
characters in ‘run’ is 3, 4 om ampm. Similarly
the final post will be ‘scoured’. The number of 200 = 40 +70 + 90
characters in this is 7.
170=with no combination of boxes, the total weight is
24. In the given sequence, the position of O is increasing not 170kg.
from the back to the front of each post.
30. Given, E = 1, D = 2, X = 3, S =4, P = 5, O=6,
th th
XXXOXO=O placed in 4 and 6 position
3532143 = XPXDESX = It does not make any sense.
rd th 1356412 = EXPPOSED this means that, to introduce
XXOXOX =O placed in 3 and 5 position
someone.
XOXOXX= O placed in 2nd and 4th position 6121341 = OEDEXSE = It does not make any sense
OXOXXX O placed in 1ST and 3rd position 4321242 = SXDEDSE = It does not make any sense

That is the correct answer. 31. On reading the englidh alphabet from the beginning
the lace of letters D, I, S, C, E, R, N, S, are 4, 9,
25. The correct answer is 2. 19, 3, 18, 14, 19,.
-7, -3, 2,8,15 ⇒This type of sequence can be On reading the English alphabet from the back the place
made known by the following method: of letters of W, R, H, X, V, I, M, H, is 4, 9, 19, 3, 5, 14, 19.
-7 + 4 (4) =- 3 Somilarly,

-3 + (5) = 2 On reading the English alphabet from the back the place
of lettersn A, G, O, are 1, 7, 15.
2 + (6) =8
On reading the English alphabet from the back position
8 + (7) = 15 of 1, 7, 15, is Z, T, O,

26. The correct answer is 1/2. 32. In’639’and ‘567’ there is 6 in common and’ is’ ‘is
common in ‘wood is hard’ and ‘fur is soft’ “soft”
⇒ 2, 1.25, 1//2, -1/4, -1
= 5.
This type of sequence can be made known by the
In ‘657’and 135’ there is in common and “hard” is
following nethod:-
common in ‘wood is hard’ and ‘fur is soft’ so “hard” = 3.

In 135, 3 = “hard” and 5 =- “soft” then 1 would mean


“or”.

Engineer’s platform
33. Condition (4) ⇒ FEEFEE ⇒ The location of F is 4th and 1th
That is the correct answer.

38.

34. Have given.

Condition (1) ⇒A @ B = A is the father of B. so M @ N =


M is the father of N. That is, Nis the son of M

Condition (2) ⇒ A # B = A is brother if B. So L # M = L is


brother or M. According to the diagram, the postman was in the ‘A’
place first. Then 4 km walks west and reaches ‘B’. then 7
Condition (3) ⇒ A ! B = A is the son of B,. So N ! O = N
km goes in the south direction and reaches ‘C’. then
is the son of O. That is, O will be the mother of N.
4km moves ion the east direction and reaches ‘D’ then
Because M is N’ s father. So M will be the husband of
10 km walks north and reaches ‘E’
O. Aad L, N’ uncle.
According to Diagram.
35. Condition (1)
62 # 51 = 0 ⇒ BC || AD or BC = AD
⇒ (6+1)-(2+5) = 7-7 = 0 AE = DE –AD
Condition (2) = DE – BC
82 # 54 = 5 = 10 – 7
⇒ (8+4) – (2+5) = 12 – 7 = 5 = 3 km
Similarly, Hence Postman, 3km. North from the starting position.
Condition (3)
⇒ 72 # 66 39.
⇒ (7+6) – (2+6) = 13 – 8 = 5
This will be the correct answer

36. In the given question the position of V is progressing


backwards in each post respectively. Such as in the
visual, the place of ‘V’ is the first, in Avarice, the final
position would be 6th. So the correct answer is
Reweave.
According to the diagram. A and B begin to move from
37. In the given question, the location of F is
point ‘O’ A reaches 4 m in the north ‘P’ east “R’ then 4
progressing gradually.
m in the north ‘S’.
1st and 4th According to diagram,
⇒ OR|| PS or OR = PS
Condition (2) ⇒ FEEFEE ⇒ The location of F is 2nd and 5th QR = PS – PQ
= OR –PQ
Condition (3) ⇒ EEFEEF ⇒ The location of F is 3nd and
= (9-3) m
6th Like this

Engineer’s platform
= 6 m. 44.
Hence, B is 6 m to the east of A

40.

Argument l = Indian and China are both placed in Asia.

Argument II = Indian and China are different countries.

According to diagram both arguments are true.


41. More people will travel through local trains as a
result of ticket exemption, certainly the number of 45. Have given:
vehicles in the streets will be reduced. But the local
railways of India are more congested increasing the Condition (1) ⇒ The group that shows the hockey
number of people can worsen the situation even worse. player, who does not play football.
Therefore, both arguments 1and ll are correct.
Condition (2) ⇒ Cricketers who play kabaddi.
42.
Condition (1) ⇒ According to diagram, ‘o’ refers to
those players who play football. And ‘S’ represents the
players who play football and kabaddi. This conclusion
is true for condition (1)

Conclusion (2) ⇒ According to diagram, ‘U’ refers to


those players who play cricket and kabaddi this
conclusion is true for condition (2).

Therefore, option (3) O, S and U are correct.

43.

This design perfectly matched on option (b) that is the


correct answer.

Engineer’s platform
Solved Paper 2017 (Set-2) 8. Select the related word / letters/ number from the
1. Select the related word letters/number from the given alternatives.
given alternatives. 364 : 697 : : 265 : ?
Coal : Black : : Chalk : ? (a) 598 (b) 573
(a) Porous (b) White (c) 863 (d) 475
(c)Brittle (d) Soft
9. Select the related word / letters/ number from the
2. Select the related word/ letters/ number from the given alternatives.
given alternatives. 861 : 123 : : 2352 : ?
Oasis : Water : : Island : ? (a) 336 (b) 461
(a) Land (b) Sea (c) 1548 (d) 2234
(c) Shore (d) Dry
10. Select the odd word /letters/number/word
3. Select the related word/ letters/ number from the pair/number pair from the given alternatives.
given alternatives. (a) Cycle (b) Motorcycle
Buy : Bye : : ? (c) Scooter (d) Car
(a) Brake : Break (b) pen : pencil
(c) Coffee : Toffee (d) Thorn: Horn 11. Select the odd word/letters/number/word
pair/number pair from the given alternatives.
4. Select the related word / letters/ number from the (a) Dead or alive (b) Life or Death
given alternatives. (c) Body and soul (d) Living or non-living
AEIOU : BFJPV : : JLNP : ?
(a) LKMO (b) KMOQ 12. Select the odd word /letters/number/word pair
(c)UOIE (d) MNOP number pair from the given alternatives.
(a) Television and Fan
5. Select the related word / letters/ number from the (b) Computer and Tube light
given alternatives. (c) Bed and Cupboard
Cause : Cease : : Poise : ? (d) Fan and Tube light
(a) Pipes (b) Poppies
(c) Possess (d) Sips 13. Select the odd word/letters/number/word
pair/number pair from the given alternatives.
6. Select the related word/ letters/ number from the (a) VD (b) ZA
given alternatives. (c) BY (d) XC
Gourmet : Methane : : Tomato : ?
(a) Atomize (b) Potato 14. Select the odd word/letters/number/word
(c) Atoll (d) Atomic pair/number pair from the given alternatives.
(a) Gender (b) Agenda
7. Select the related word / letters/ number from the (c) Spender (d) Amender
given alternatives.
26 : 65 : : 28 : ? 15. Select the odd word /letters/number/word pair
(a) 82 (b) 88 number pair from the given alternatives.
(c) 70 (d) 75 (a) DTF (b) UEF
(c) AIA (d) BHG
16. Select the odd word/letters/ number/word pair/
number pair from the given alternatives.
Engineer’s platform
(a) 100101 (b) 1100101 (a) pez (b) pdy
(c) 1110001 (d) 1010011 (c) qvd (d) qey

17. Select the odd word/letters/ number/word pair 24. In the given series one word/ one term/one
number pair from the given alternatives. number is missing. Select the correct alterative from
(a) 40 (b)25 the given ones that will complete the series.
(c) 64 (d) 100 OXOXXXX, XOXOXXX, XXOXOXX, XXXOXOX, ?
(a) XXXOXOX (b) OXOXXXX
18. Select the odd word/letters/number/word (c) XXXXOXO (d) XOXOXXX
pair/number pair from the given alternatives.
(a) 104, 17 (b) 114, 19 25. In the given series one word/ one term/one
(c) 144,24 (d) 132, 22 number is missing. Select the correct alterative from
the given ones that will complete the series.
19. In the given series one word/one term/one 11, 6, ?, -7, -15
number is missing. Select the correct alternative from (a) 1 (b) -1
the given ones that will complete the series. (c) 0 (d) -3
Educative, Probable, Synergy, Friday, ?
(a) Male (b) Rooted 26. In the given series one word/ one term/one
(c) Guilt (d) Beer number is missing. Select the correct alterative from
the given ones that will complete the series.
20. In the given series one word/one term/one 3, -4/3, 1/3, ?, 11/3
number is missing. Select the correct alternative from (a)-2 (b) 4/3
the given ones that will complete the series. (c) 2 (d) 4/3
Hello, Lofty, Tyre, Repair, ?
(a) Tonic (b) Ironic 27. In the given series one word/ one term/one
(c) Panic (d) Paired number is missing. Select the correct alterative from
the given ones that will complete the series.
21. In the given series one word/one term/one 17, 19, 23, ?, 31
number is missing. Select the correct alternative from (a) 25 (b) 30
the given ones that will complete the series. (c) 24 (d) 29
Ago, Veal, choke, Decree, ?
(a) Bristles (b) Deviance 28. If p < r, r < s and t > p, which of the following must
(c) Ordain (d) Ensures be true?
(I) p < s (II) s > t
22. In the given series one word/one term/one (a) Only I (b) Only II
number is missing. Select the correct alternative from (c) Neither I nor II (d) Both l and II
the given ones that will complete the series.
R, O, L, I, ? 29. The weights of five boxes are 10, 30, 40, 70 & 90
(a) H (b) G kilograms. Which of the following cannot be the total
(c) E (d) F weight, in kilograms, of any combination of these
boxes?
23. In the given series one word/one term/one (a) 180 (b) 170
number is missing. Select the correct alternative from (c) 160 (d)190
the given ones that will complete the series.
lav, mbw, nex, ody, ?
Engineer’s platform
30. If the letters B, N, O, A, & Z are numbered 1, 2, 3, 4 (c) WMWMWW (d) WWMWMM
& 5 respectively. Select that combination of numbers so
that letters arranged accordingly, form a meaningful 38. A hunter while chasing a prey walks 5 km North,
word. then turns East and walks 3 km. then turns North and
(a) 1214151 (b) 1324254 walks 10 km and walks 3 km. Where is, then turns to
(c) 5152243 (d) 4341652 his left he now with respect to the starting position?
(a) 5 km North from the starting position
31. If SCALDING is coded as HXZOWRMT, then how will (b) 15 km South from the starting position
PER be coded as ? (c) 5 km South from the starting position
(a) RFV (b) VFR (d) 15 km North from the starting position
(c) KVI (d) QWE
39. Two friends A and B start from the same point. A
32. In a certain code language, 648 means 'mat is wet’; walks 4 m North, then turns to his right and walks 6 m.
756 means ‘floor is dry’; 987 means 'wet or dry'; Find Meanwhile B has walked 6 M East, then he turns to his
the code for 'or; left and walks 12 m. Where is B now with respect to
(a) 5 (b) 1 the position of A?
(c) 9 (d) 3 (a) B is 16 m to the North of A
(b) B is 8 m to the South of A
33. In a certain code language, ‘+’ represents 'x’, ‘-’ (c) B is 16m to the South of A
represents ‘+’, 'x' represents ‘/' and ‘/' represents ‘-‘, (d) B is 8 m to the North of A
Find out the answer to the following question 0.02 +
400 - 123 x 3 =? 40. In the question two statements are given, followed
(a) -33 (b) 49 by two conclusions, I and II. You have to consider the
(c) 0.2 (d) 36 statements to be true even if it seems to be at
variance from commonly known facts. You have to
34. If A@B means A is mother of B, A#B means A is decide which of the given Conclusions if any follows
sister of B and A!B means A is daughter of B, then from the given statements.
what does X!Y#Z@W mean ? Statement 1: Some apples are green.
(a) Z is mother of X (b) Y is sister of W Statement 2: All green things are healthy.
(c) W is mother of X (d) Z is maternal aunt of X Conclusion I: Some apples are healthy.
35. If 62 @ 12 = 11, 23 @ 34 = 12, then find the value Conclusion II: No apples are healthy
of 73 @ 16 = ? (a) Only conclusion II follows
(a) 10 (b) 7 (b) Either I or II follows
(c) 13 (d) 17 (c) Only conclusion II follows
(d) Neither I nor II follows
36. Which of the following words follows the trend of
the given list? 41. In the question a statement is given, followed by
Reconquer, Tranquil, Unique, Tequilla, Squish,? two arguments, I and IL You have to consider the
(a) Queue (b) Plaque statement to be true even if it seems to be at variance
(c) Clique (d) Aqua from commonly known facts. You have to decide
which of the given arguments, if any, is a strong
37. Which of the following terms follows the trend of argument.
the given list? Statements: Should learning self-defense be made
WMMWWM, MMWWMW, MWWMWM compulsory for girl students?
(a) WMMWWM (b) MMWMWM
Engineer’s platform
Argument I: No, there is a risk that they will misuse
these skills to bully others.
Argument II: Yes, in an environment where cases of
assault on woman are increasing, self-defense training
becomes one essential part of woman safety.
(a) If only argument Il is strong
(b) If only argument I is strong 45. In the following figure, square represents coaches,
(c) If neither argument I nor II is strong triangle represents teachers, circle represents players
(d) If both arguments I and II are strong and rectangle represents boys. Which set of letters
represents boys who are not players and teacher who
42. Which of the following cube in the answer figure are not boys?
cannot be made based on the unfolded cube in the
question figure?

(a) K, M, N, and L (b) I, K, M and N


(c) J, K, N and L (d) K, N, L and I

46. Which answer figure will complete the pattern in


the question figure?

43. Which of the following answer figure patterns can


be combined to make the question figure?

47. From the given answer figures, select the one in


which the question figure is hidden /embedded.

44. Which of the following diagrams represent the


relationship between Doctors, Nurses and Surgeons?

Engineer’s platform
48. A piece of paper is folded and punched as shown
below in the question figures. From the given answer
figures, indicate how it will appear when opened. ANSWERS
1.(b) 2.(a) 3.(a) 4.(b) 5.(a) 6.(d) 7.(c) 8.(a) 9.(a) 10.(*)
11.(c) 12. (c) 13.(a) 14.(b) 15.(c) 16.(a) 17.(a) 18(a) 19.(c)
20.(b) 21. (d) 22.(d) 23.(a) 24.(c) 25.(c) 26.(c) 27.(d)
28.(a) 29.(a) 30.(b) 31.(c) 32.(c) 33.(b) 34.(d) 35.(d)
36.(a) 37.(d) 38.(d) 39.(d) 40.(c) 41.(a) 42.(a) 43.(d)
44.(c) 45.(d) 46.(d) 47.(b) 48.(b) 49.(a) 50.(c)

49. If a mirror is placed on the line MN, then which of


the answer figures is the right image of the given
figure?

50. A word is represented by only one set of numbers


as given in any one of the alternatives. The sets of
numbers given in the alternatives are represented by
two classes of alphabets as shown in the given two
matrices. The columns and rows of Matrix-I are
numbered from 0 to 4 and that of Matrix-II are
numbered from 5 to 9. A letter from these matrices
can be represented first by its rows and next by its
columns, for example 'Q’ can be represented by 10, 41
etc. and ‘I’ can be represented by 76, 97 etc. Similarly,
you have to identify the set for the word ULTRA".

(a) 11, 41, 44, 85, 22 (b) 21, 40, 04, 69, 01
(c) 23, 95, 14, 20, 79 (d) 12, 13, 41, 55, 69
Engineer’s platform
EXPLANATION 11. The words in the given options are the antonym of
1. The color of coal is black similarly color of chalk is each other. i.e., the antonym of the dead is alive but
white. Body is not the antonym words of soul. So this option is
3. The pronunciation of Buy' and Bye' words is the same different from other options.
and the number of letters is also equal. Similarly,
pronunciation is found in option (a). 12. Television, fan, computer and tube light, all these
devices work on help of electricity. While electricity is
4. AEIOU: BFJPV not used in bed and cupboards.
A => A + 1 = B
E => E + 1 = F 13. This question is based on the sequence of letters of
I => I + 1 = J the English alphabet.
O => O + 1 = P VD => ‘V’, 4th from the back and ‘D’, 4th from the front
U => U + 1 = V ZA => ‘Z’, 1st from the back and ‘D’, 1st from the front.
Similarly XC => ‘X’, 3rd from the back and 'C', 3rd from the front.
JLNP:? While in ‘By', ‘B’ 2nd from the front and ‘Y’, 2nd from
J => J + 1 = K the back. These options are different from other
L => L + 1 = M options.
N => N + 1 = O
P => P + 1 = Q 14. The 'ender' in Gender, Spender and Amender's
6. In this sequence, the last 3 letters of the word are the Spelling. Whereas in the Spelling of Agenda there are
starting letters of the next word. For example, 'met' of 'enda'. Which is different from other options.
'Gourmet’ is the starting letter of ‘Met'hane. Similarly,
‘ato’ of ‘Tomato' is the starting letter of 'Ato’mic. 15. 'AIA', the same letter has been used twice in this
option. While in the other options all the letters are the
𝟐𝟔 𝟐𝟖 different.
7. 𝟔𝟓
= 𝒙
𝟏𝟖𝟐𝟎
𝒙= = 𝟕𝟎
𝟐𝟔 16. ‘100101’, this number has six digits While the other
numbers have seven digits.
8. 364:697
3 => 3 + 3 = 6 17. Here, 25 = 52, 64 = 82 and 100 = 102, while 40 is not a
6 => 6 + 3 = 9 perfect square.
4 => 4 + 3 = 7
Similarly 114 144 132
18. 19
=24
= 22 = 6 (𝑇ℎ𝑒 𝑟𝑎𝑡𝑖𝑜 𝑖𝑠 𝑠𝑎𝑚𝑒. )
265:598
104
2 => 2 + 3 = 5 while in 17 = 𝟔. 11(𝑇ℎ𝑒 𝑟𝑎𝑡𝑖𝑜 𝑖𝑠 𝑑𝑖𝑓𝑓𝑒𝑟𝑒𝑛𝑡. )
6 => 6 + 3 = 9
5 => 5 + 3 = 8 19. The number of character in the word is gradually
decreasing. The first word has 9 characters, 8 in the
861 2352 second, and in the end the answer will be of 5
9. 123
= 𝑥
123 characters.
2352 ∗ = 336 Option (c) satisfies the above condition.
861

10. There are only two wheels in bicycles, motorcycles 20. In this sequence, the last 2 letters of the word are
and scooters, while the car has four wheels. the starting letters of the next word. For example, 'lo' of

Engineer’s platform
‘Hello’ is the starting letter of ‘Lo'fty. Similarly ‘ir' of 17, 19, 23, 29, 37 This type of sequence can be made
'Repair’ is the starting letter of ‘Ir'onic. known to the following method and all are prime
numbers.
21. In ago, veal, choke and decree, the numbers of 17 + (2) = 19
letters are increasing gradually. For example, the 19 + (4) = 23
number of characters in 'ago' is 3, 4 in veal', 5 in 'choke' 23 + (6) = 29
and 6 in 'decree'. Similarly, the next word will be 29 + (8) = 37
'ensures' in which these are 7 letters.
28. p < r, r < s and t > p.
22. This equation can be written as
t > p < r < s => p
Conclusion I is true.

29. 170 = 90 + 70 + 10
23.
160 = 90 + 70
190 = 70 + 30 + 90
While '180 kg' cannot be represented with the given
combination.
In the table, letters are moving according to the
Alphabetical correct. For example, l, m, n, o, p. The 30. Given, B = 1, N = 2, O = 3, A = 4, Z =5
answer is pez. 1214151 = BNABZB = It does not make any sense
1324254= BONANZA= this means Gift
24. In the given sequence, the position of O is increasing 5152243=BZNNAO = It does not make any sense.
in every position from the front to the back.
OXOXXXX => O's place 1st and 3rd 31. On reading the English alphabet from the beginning,
XOXOXXX => O's place 2nd and 4th the place of letters S, C, A, L, D, I, N, G are 19, 3, 1, 12, 4,
XXOXOXX => O's place 3rd and 5th 9, 14, 7 On reading the English alphabet from the back,
XXXOXOX => O's place 4th and 6th the place of letters of H, X, Z, O, W, R, M, T is 19, 3, 1,
XXXXOXO => O's place 5th and 7th 12, 4, 9, 14, 7.
Similarly,
25. On reading the English alphabet from the beginning, the
place of letters P, E, R are16, 5, 18. On reading the
English alphabet from the back, the position of 16, 5, 18
is for K, V, I letters.

32. In '648' and ‘756' there is 6 in common, and 'is' is


26. The given series, common in 'met is wet' and ‘floor is dry' so "is"=6.
4 1
−3, − 3 , 3 , 𝑥,
11
is in AP with common difference (3)
5 In '648' and '987 there is 8 in common, and "wet is
3
1 5
common in 'met is wet’ and ‘wet or dry’ so "wet"= 8.
∴ + =𝑥 In 756' and '987 there is 7 in common, and "dry" is
3 3
∴ 𝑥=2 common in ‘floor is dry’ and 'wet or dry' so "dry'=7.
∴ In 987, 8="wet" and 7 ="dry", then 9 would
27. The correct answer is 29. mean "or”

Engineer’s platform
33. (+) => (x), (-) => (+), (x) => (÷), (÷) => (-) 38.
Change the sign of given equation, with the condition
we get,
123 123
(0.02 x 400) + ( ) = (0.02 x 400) + ( )
3 3
= 8 + 41 = 49

34. Have given,


Condition (1) => A@B = A is the mother of B. So Z@W =
Z is the mother of W.
Condition (2) => A#B A is sister of B. So Y#Z = Y is sister
of Z. According to diagram, Hunter goes from A to 5 km, B
Condition (3) => A!B =A is the daughter of B, So X!Y = X then from B to 3 km towards C, then from C to 5 km
is the daughter of Y. That is, Y will be the mother of X. toward D, then from D to 5 km, towards E then from E
Because Y is sister of Z. So Z is maternal aunt of X. to 5 km, F.
𝐴𝐹 = 𝐴𝐵 + 𝐵𝐸 + 𝐸𝐹
35. = 𝐴𝐵 + 𝐶𝐷 + 𝐸𝐹 (∵ 𝐵𝐸 = 𝐶𝐷)
Condition (1) => 62@12 = 11 = 5 + 5 + 5 = 15
Hence the hunter 15 km North from the starting
 (6+2)+(1+2) = 8+3 = 11
Condition (2) => 23@34 = 12 position.
 (2+3) +(3+4) = 5+7 = 12
Similarly, 39.
Condition (3) => 73@16
 (7+3)+ (1+ 6) = 10+7= 17
This will be the correct answer.

36. In the given question, the position of qu is moving


forward in every word, respectively. As such, the
position of ‘qu' in Reconquer, is 6th and 7th, in Tranquil,
5th and 6th. Similarly, the position of ‘qu' in the final
post will be 1st and 2nd. So the correct answer is Queue.
A and B begin to move from point O. A goes towards 4m
37. In the given question, WW's position is progressing P, then goes from P to 6m Q, B goes toward 6m, then
backwards in each term, respectively. goes from R to 12m S.
Condition (1) => W MM WW M => WW's place is 4th and QS = RS - RQ
5th. = RS - OP (∵ RQ= OP)
Condition (2) => MM WW M W => WW’s place is 3rd and = 12 – 4 = 8
4th. Hence B is 8m to the North of A.
Condition (3) => M WW M W M => WW’s place is 2nd 40.
and 3rd.
Similarly,
Condition (4) => WW M W MM => WW's place is 1st and
2nd That is the correct answer.

According to diagram. Conclusion T is correct.


Engineer’s platform
41. Prevention of atrocities on women is difficult for any
particular person or organization at any given time. If
women learn to self-protect, then there will be a
shortage of atrocities on them. Therefore the argument
‘II' is correct.

42.

This type of combination not mode in option (a).

44.

All doctors can be surgeon. But the nurse is not doctor


or surgeon.
So option (c) is correct answer of the question.

50. U = 02, 23
L = 58, 87, 89, 95
T = 14
E = 20
R = 66, 79
Option (1) are perfectly satisfy the word ULTER
23, 95, 14, 20, 79.

Engineer’s platform
Solved Paper 2017 (Set-3) 8. Select the related word/letters/numbers
1. Select the related word/letters/numbers from the given alternatives.
from the given alternatives. 159 : 840 : : 345 : ?
Red : Colour : :French : ? (a) 654 (b) 765
(a) foreign (b) language (c) 876 (d)987
(c) European (d) Country
9. Select the related word/letters/numbers
2. Select the related word/letters/numbers from the given alternatives.
from the given alternatives. 761 : 928 : : 651 : ?
Chips : Potatoes : : Soda : ? (a) 765 (b) 753
(a) Fizz (b) Bottle (c) 807 (d) 951
(c) lemon (d) Water
10. Select the odd word/letters/numbers from
3. Select the related word/letters/numbers the given alternatives.
from the given alternatives. (a) kilometers (b) feet
Quadrilateral : Four : : ? (c) grams (d) micrometers
(a) Cylinder : Circle (b) Cube : Square
(c) Triangle : 180 (d) Hexagon : Six 11. Select the odd word/letters/numbers/word pair
from the given alternatives.
4. Select the related word/letters/numbers (a) pros and cons (b) dead or alive
from the given alternatives. (c) null and void (d) sooner or later
FGI : HIK : STV : : ?
(a) UVW (b) VWY 12. Select the odd word/letters/numbers/word pair
(c) XYZ (d) UVX /number pair from the given alternatives.
(a) Crayon and marker
5. Select the related word/letters/numbers (b) pen and eraser
from the given alternatives. (c) book and diary
Dream : Area : : Freer (d) pen and marker
(a)Farmer (b) Ear
(c) Fare (d) freer 13. Select the odd word/letters/numbers/word pair
/number pair from the given alternatives.
6. Select the related word/letters/numbers (a) OU (b)YC
from the given alternatives. (c) IA (d) EO
Brine : Inert : : Beware : ?
(a) Arenas (b) Declare 14. Select the odd word/letters/numbers/word pair
(c) Spare (d) Area /number pair from the given alternatives.
(a) Cuisine (b) Business
7. Select the related word/letters/numbers (c) Disinterested (d) Noisiness
from the given alternatives. 15. Select the odd word/letters/numbers/word pair
169 : 13 : : 225 : ? /number pair from the given alternatives.
(a) 22 (b) 25 (a) DRGK (b) WMUI
(c) 20 (d) 15 (c) OHAU (d) XHTV

Engineer’s platform
16. Select the odd word/letters/numbers/word alterntive from the given ones that will
pair /number pair from the given complete the series.
alternatives. L, J, H, F, ?
(a) 7531 (b)2468 (a) E (b) G
(c) 9753 (d) 8642 (c) D (d) I

17. Select the odd word/letters/numbers/word 23. If the given series one word/one term/one
pair /number pair from the given number is missing. Select the correct
alternatives. alterntive from the given ones that will
(a)15 (b)25 complete the series.
(c)53 (d)45 Eca, fdb, gec, hfd
(a) ige (b) ieg
18. Select the odd word/letters/numbers/word (c) gei (d) egi
pair /number pair from the given
alternatives. 24. If the given series one word/one term/one
(a) 108, 132 (b)114, 156 number is missing. Select the correct
(c) 136, 152 (d) 120, 138 alterntive from the given ones that will
complete the series.
19. In the given series one word/one term/one XXXXOXO,XXXXOOX, XXXXOOX, XXXOXOX
number is missing. Select the correct (a) XOXXOXX (b) XXXXOXO
alterntive from the given ones that will (c) XXXXOOX (d)XXOXXOX
complete the series.
Cremation, accolade, Maestro, Chrome? 25. If the given series one word/one term/one
(a) very (b) Zebra number is missing. Select the correct
(c) Virtual (d) Time alterntive from the given ones that will
complete the series.
20. If the given series one word/one term/one -2 , 1, 5, ?, 16
number is missing. Select the correct (a)9 (b) 10
alterntive from the given ones that will (c) 11 (d) 13
complete the series.
Toxic, Icon, Onto, Tomorrow 26. If the given series one word/one term/one
(a) Owl (b) Wet number is missing. Select the correct
(c) Rat (d) Borrow alternative from the given ones that will
complete the series.
21. If the given series one word/one term/one -10/3, ?, -2/3, 2/3, 2
number is missing. Select the correct (a) -2 (b) 2
alterntive from the given ones that will (c) -1/3 (c) 1/3
complete the series.
Age, dire, genre, stumpy, ? 27. If the given series one word/one term/one
(a) splayed (b) secretes number is missing. Select the correct
(c) preacher (d) shooed alternative from the given ones that will
complete the series.
22. If the given series one word/one term/one 23, 29, ?, 37, 41
number is missing. Select the correct (a) 30 (a)33
(c) 31 (d) 35
Engineer’s platform
28. If a < b, d > c and a < d, which of the following is 35. If 45 @ 323 = 14, 76 @ 22 = 17, then find the
true? value of 55 @ 10 = ?
I. b < c II. c > a (a) 8 (b) 11
(a)Only l (b) Neither l nor ll (c) 15 (d) 5
(c) Only ll (d) Both l and ll
36. Which of the following words follow the trend
29. The weights of five boxes are 10, 30, 40, 70, & of the given n list?
70 kilograms. Which of the following cannot be Zonal, Tzars, izen, Seize, Waltz
the total weight in kilograms if any combination (a) Unitize (b) Ablaze
of these boxes? (c) Azure (d) Sanza
(a) 190 (b)180
(c) 210 (d) 200 37. Which of the following words follow the trend
of the given n list?
30. If the letters Q, B, T, A, U, E, &, N are numbered ABC, BCAB, CABCA,
1, 2, 3, 4, 5, 6, & 7, respectively. Select that (a)BCABCA (b) CABCAB
combination of number so that letters arranged (c) ABCCBA (d) ABCABC
accordingly, from a meaningful word.
(a) 5617312 (b)5447134 38. A girl walks 3 km east starting from her home.
(c) 2471563 (c) 324237 She then turns south and walks 2 km then she
turns west and walks 7 km then she turns to
31. IF PROXIMAL is coded as KILCRNZO, then how her right and walks 2 km. where is she now
will WHY be coded as ? her starting position?
(a)DSB (b) EDC (a) 10km to the west from her home.
(c) CDE (d)BNM (b) 4 km to the east from her home.
(c) 10 km to the east from her home.
32. In a certain code language, 531 means ‘bay is (d) 4 km to the west from her home.
shy’,346 means ‘girl is bold’, 256 means ‘shy or
bold’, Find the code for ‘or’. 39. A is standing 6 m to the east B. A walks 9 m
(a) 1 (b)2 south the turns to his right and walks 7 m. at
(c) 3 (d) 4 the same time, B has walked 3 m west, then be
turned south and walked 9 m then he turned
33. In a certain code language, ‘+’ represent ‘X’, ‘-‘ to his left and walked 5m. where is B now with
Represent ‘+’, ‘X’ represents ‘+’ and ‘+’ and ‘+’ respect to the position of A?
Represent ‘-‘. Find out the answer to the following (a) B is 12 m to the east of A
question. (b) B is 3 cm to the west of A
(a) 90 (b) 10 (c) B IS 3 m to the east of A
(c) 1.25 (d) 108 (d) B is 12 m to the west of A

34. If A @ B means A is father of B, A # B means A 40. In the question two statements are given
is sister of B and A! B means A is A is son of followed by two conclusions, l and ll, You have
B, then what does E @ F ! G # mean, if H is to consider the statements to be be true even
a male? if it seems to be at variance from commonly
(a) H is brother of E (b) H is father of E known facts. You have to decide which of the
(c) H is son of E (d) H is E’ s wife’s brother
Engineer’s platform
given condusions, if any follows from the 43. Which of the following answer figure patterns
mentioned statements. can be combined to make the question figure ?
Statement l : A ll huts are made of mud.
Statement ll : Thing made of mud are not
strong
Conclusion l : All hutes are s strong.
Conclusion ll : Mud huts are not strng.
(a) Only conclusion l follows
(b) Either l or ll follows
(c) Neither I nor ll follows
(d) Only conclusion ll follows

41 In the question a statements is given followed


by two arguments, l and ll, You have to
consider the statements to be be true even
if it seems to be at variance from commonly
known facts. You have to decide which of
the given arguments, if any follows from the
44. Which of the following diagrams represent the
arguments.
relationship between engineers, software
Statement: Should teachers be permitted to
engineers and chemical engineers?
cane unruly children ?
Argument l: No, this will teach them that
physical violecce is an acceptable means
of social behavior.
Argument ll: Yes children taught in a strict
atmosphere are more successful.
(a) only argument ll is strong
(b) neither argument l nor ll is strong
45. In the following figure, square represents
(c) both argument l and ll are strong
engineers, trianglere present environmentalists,
(d) only argument l is strong
circle represents lawyers and rectangle
represent government officers. Which set of
42. Which of the following cube in the answer
letters represent lawyers who are not
figure cannot be made based on the unfolded
environmentalists and government officers who
cube in the question figure?
are engineers

Engineer’s platform
46. Which answer figure will complete the pattern
in the question figure?

50. A word is represented by only one set of numbers


47. From the given answer figures, select the one in as given in any one of the alternatives. The sets of
which the question figure is hidden/embedded. numbers given in the alternatives are represented by
two classes of alphabets as shown in the given two
matrices. The columns and rows of matrics-I are
numbered from 0 to 4 and that of matrices-II are
numbered from 5 to 9. A letter from these matrices can
be represent first by its row and next by its column, for
example ‘N’ can be represented by 00, 12 etc and ‘C’
can be represented by 95, 76 etc. Similarly, you have to
identify the set for the word ‘ TYRES’.

48. A piece of paper is folded punched as shown


below in the question figures. From the given answer
indicate how it will appear when opened

Answer:
1.(b) 2.(d) 3.(d) 4.(d) 5.(c) 6.(a) 7.(d)
49. If a mirror is placed on the line MN, then which of 8.(a) 9.(c) 10.(c) 11.(c) 12.(c) 13.(b) 14.(b)
the answer figures is the right image of the given 15.(a) 16.(b) 17.(c) 18.(c) 19.(b) 20(a) 21.(a)
figure? 22(c) 23(a) 24.(d) 25.(b) 26.(a) 27.(c) 28.(b)
29.(a) 30.(c) 31.(a) 32.(b) 33.(a) 34.(d) 35.(b)
36.(a) 37.(d) 38.(d) 39.(c) 40.(d) 41.(a) 42.(b)
43.(b) 44.(a) 45.(d) 46.(a) 47.(d) 48.(a) 49.(d)
50.(c)
Engineer’s platform
EXPLANATION: 12. Crayon, marker, pen and rubber are used in
1. Red is the color, similarly French is the name writing work. While the use of book and
of the language. diary is used in ‘reading ‘and to make
necessary information notes.
2. Chips : Potatoes ⇒ Chips are made from
13. OU, IA and EO, these are all vowels. While YC
potatoes.
is consonant. So the correct answer is YC.
Similarly, Soda : Water, ⇒ soda is mixed with
water and drink.
14. Except for the word “Business”, in all a
3. Quadrilateral : Four ⇒ There are four sides in
words have “isin”is used.
the Quadrilateral.
16. 7531 ⇒ (7-5) = (5-3) = (3-1) = 2 (The
4. FGI : HIK ⇒ F (+2) = H, G(+2) = L, L(+2) = answer is 2 when the numbers are reduced to
K, each other.) Similarly, it happens in 9753 and
Similarly, STV : UVX ⇒ S(+2) = U, T(+2) = V, 8642. 2468 ⇒ (2-4) = (4-6) = 6-8) = -2 (The
V(+2) = X answer is -2 when the number are reduced to
each other.) 2468 is different from the other
5. Dream : Area ⇒ 5 : 4 (Ratio of the number of options.
letters ) Similarly, Frame : Fare ⇒ 5 : 4
17. 15, 25 and 45, these number are completely
6. In this sequence, the last 3 letters of the word divisible by 5, while 53, is not divisible. So the
are the starting letters of the next word. The correct answer is 53.
‘ine of ‘Brine’ is the starting letter of ‘inert’.
Similarly, the ‘Arenas’. Also the number of 19. The number if characters in the word is
letters is also equal. gradually decreasing. The first word has
9 characters, 8 in the second. Similarly, in
7. 169 : :13 ⇒ 13 ⨯ 13 = 169, Similary 225 : 15 the end the answer will be of 5
⇒ 15 ⨯ 15 = 225. characters. The right answer will be Zebra

8. 159 : 840 ⇒ 159 + 840 = 999, Similarly, 345 : 20. In this sequence, the last 2 letters of the word
654. ⇒ 345 ÷ 654 = 999, the correct answer are the starting letters of the nest word. For
is 654. example, ‘ic’ of ‘Toxic’ is the starting letter of
‘icon’ similarly, ‘ow’ of ‘Tomorrow is the starting
9. 761 : 928 ⇒ 761 ÷ 167 (761’s opposite 167)
letter if ‘Owl’, The correct answer is Owl.
= 928 Similarly, 651 : 807 ⇒ 651 + 156
= 807 answer is 807.
21. The number of characters of words is increasing
gradually. The first word has 3 letters, 4 in the
10. Kilometers, feet and micrometers are the unit
second, in the same way the answer at the end
of length. While the grams is the unit of
weight. The correct answer is grams. will be of 7 characters. The correct answer
will be splayed.
11. The given options are the opposite words of
each other. Assuch, pros-cons, dead-alive, 22. In the given sequence, there is a difference of
sooner-later. The opposite words of null are one letter. For example, L, K, J, I, H, G, F, E,
not void. So the correct answer is null-void. D, So the correct answer is D.

Engineer’s platform
2471563 = BANQUET = This means, the
‘dinner’
5447134 = UAANQTA = It does not make
any sense.
3242637 = TBABETN = It does not make any
24. In the given sequence, the position of the last O sense.
is moving backward from every term.
XXXXOXO ⇒ O’s position 7th 31. On reading the English alphabet from the
XXXXOOX ⇒ O’s position 6th , back the place of letters of P, R, O, X, I, M, A,
XXXXOOX⇒ O’s position 5th L, is 11, 9, 12, 3, 18, 14, 26, 15, On reading the
XXXOXOX ⇒ O’s position 4th English alphabet from the beginning the place
XXOXXOX⇒ O’s position 3th of letters K, I, L, C, R, N, Z, O, are 11, 9, 12, 3,
18, 14, 26, 15.
25. -2, 1, 5, 10, 16, ⇒ This type of sequence can Similarly, On reading the English alphabet
made known to the following method:- from the back, the letters of letters W, H, Y,
-2 + (3) = 6 are 4, 19, 2.
1 + (4) = 5 Similarly, On reading the English alphabet
5 + (5) = 10 from the beginning the position of 4, 19, 2
10 + (6) = 16 is for D, S, B, letter

26. The given service is in AP with common 32. In ‘346’ and ‘256’ there is 6 in common, and
4 ‘bold’ is common in ‘girl is bold’ and ‘sky or
difference .
3 bold’ so “bold”=6.
4 4
−3 + ( 3) = −2 In ‘531’ and ‘256’ there is 5 in common, and
4 2 “sky” is common ‘boy is shy’ and ‘shy or bold’
−2 + ( ) = − so “shy” = 5. In 256, 6=”bold” and 5= “shy”,
3 3
2 4 2 then 2 would mean “or”.
− +( )=
3 3 3
2 4
+ ( 3) = 2 33. Have given, (+)⇒ (⨯),(-),⇒ (+),(⨯)⇒
3
(÷),(÷)⇒ (-)
Changing the sigh of given equation, we get
27. 23, 29, 31, 37, 41,
The given series is of prime numbers.

28. a < b, d > c and a > d ⇒ b > a > d > c ⇒ b. So


conclusions neither l nor ll are correct. 34. Have given,
Condition (1) ⇒ A@B = A is the father of B.
29 190 = 70 + 70 + 10 + 40 So E@F = E is the father of F.
180 = 70 + 70 + 30 + 40 Condition (2) ⇒A # B = A is sister of B. So
210 = 70 +70 + 70 + 30 G#H = G is sister of H. H is male so G is
While ‘200’ km’ cannot be represent by any female.
of the combination. Condition (3) ⇒ A ! B = A is the son of B, So
F!S = F is the son of G. Because, G is female,
30. Given, Q = 1, B = 2, Y = 3, A = 4, U = 5, E = So G is the mother of F and father is E. So
6, N = 7, H is E’ s wife’s brother
5617312 = UEQNTQB = It does not make any
sense.
Engineer’s platform
35. 39.

36. In the given question, the position of Z is moving


forward in every word, respectively. As such,
the position of ‘Z’ in Zonal ‘, is 1st , in ‘Tzars’, 2nd A begin to move from point P.A goes toward
similarly, the position of ‘Z’ in the final term will 9m, Q then from Q to 7m, R, B, begin to
be 6th So the correct answer is Unitize. move from point O. B goes toward 3 m S
37 then from S to 9m T, then from T to 5m U.
⇒ UR = QT – TB – UQ
= AS – TB – UQ (QT = AS)
= 9-2-4 = 3 M.
Hence, B is 3m to the east of A.

41. Children practice practical education with


their surroundings and behavior that
In table ABC increases gradually, For example, After accompanies them. As he teaches behaves in
B, after c. the same way in the society. Therefore,
argument l is much stronger.
38. 44.

Engineer can be softwere eng. And chemical


According to diagram the girl walks from A to 3km eng. but soft. Eng. And che.eng are
B, than from B to 2km C, then from C to 7km D, then different together.
from D to 2km E. So Option (a) is correct.
7km D, then from D to 2km E.
⇒ AE = BE – AB 50. TYRES = 11, 41, 44, 85, 22,
= CD – AB (BE = CD ) T = 11
=7–3 Y = 12, 41,
= 4 km R = 03, 42, 44
Hence the girl is 4 km to the west from her E = 58, 85, 99
home. S = 22

Engineer’s platform
Solved Paper 2017 (Set-4) 111 : 120 : : 222 : ?
1. Select the related word/letters/number from the (a) 211 (b) 312
given alternatives. (b) 130 (d) 231
Wallet : Cash : : Jac : ?
(a) Glass (b) Pickle 9. Select the related word/letters/number from the
(c) Cylinder (d) Tight given alternatives.
123 : 444 : : 431 : ?
2. Select the related word/letters/number from the (a) 865 (b) 565
given alternatives. (c) 479 (d) 361
Cat : Paws : : Goat : ?
(a) Hoof (b) Horns 10. Select the odd/word/letter/number/word
(c) Tail (d) Leather pair/number pair from the given alternatives.
(a) Radio (b) Television
3. Select the related word/letters/number from the (c) Laptop (d) Smart phone
given alternatives.
(a) Nuclei : Nucleus (b) theses : thesis 11. Select the odd/word/letter/number/word
(c) Crises : crisis (d) Focus : Foci pair/number pair from the given alternatives.
(a) Cloak and dagger (b) Hammer and Nail
4. Select the related word/letters/number from the (c) Suit and Tie (c) Knife and fork
given alternatives.
ZAYB : XCWD : : VEUF : ? 12. Select the odd/word/letter/number/word
(a) TSRQ (b) TGSH pair/number pair from the given alternatives.
(c) GHIJ (d) GSHR (a)Sword and Spear (b)Bow and Arrow
(c) Sword and Shield (d) Bow and shield
5. Select the related word/letters/number from the
given alternatives. 13. Select the odd/word/letter/number/word
Mouse : Museums : : Worst : ? pair/number pair from the given alternatives.
(a) Rotor (b) Sorrows (a) DA (b) GD
(c) Row (d) Root (c) IF (d)MO

6. Select the related word/letters/number from the 14. Select the odd/word/letter/number/word
given alternatives. pair/number pair from the given alternatives.
Treason : Sonnets : : Bribes : ? (a) Consultant (b) Insulate
(a) Best (b) Crimson (c) Sultry (d) insults
(c) Bestow (d) Sonnata
15. Select the odd/word/letter/number/word
7. Select the related word/letters/number from the pair/number pair from the given alternatives.
given alternatives. (a) YBDA (b) FBAT
78 : 15 : : 67 : ? (c) HOUY (d) PABI
(a) 13 (b) 12
(c) 11 (d) 10 16. Select the odd/word/letter/number/word
pair/number pair from the given alternatives.
8. Select the related word/letters/number from the (a) 666 (b) 676
given alternatives. (c) 558 (d) 144

Engineer’s platform
17. Select the odd/word/letter/number/word 24. In the given series one word/one term/one number
pair/number pair from the given alternatives. is missing. Select the correct alternative from the given
(a) 45 (b) 61 ones that will complete the series.
(c) 52 (d) 43 OOXXXXX, OXOXXXX, XOOXXXX, XOXOXXX, ?
(a) XOXOXXX (b) OOXXXXX
18. Select the odd/word/letter/number/word (c) XXOOXXX (c) OXOXXXX
pair/number pair from the given alternatives.
(a) 13,169 (b) 17,269 25. In the given series one word/one term/one number
(c) 14,196 (d) 16,256 is missing. Select the correct alternative from the given
ones that will complete the series.
19. In the given series one word/one term/one number 7, 5, 2, ?, -7
is missing. Select the correct alternative from the given (a) -2 (b) 0
ones that will complete the series. (c) -1 (d) 3
AnD EGT, FbuION, BDcsTY, DFGhjK,?
(a) ZXCAsd (b) CVvbTY 26. In the given series one word/one term/one number
(c) CVBfgR (d) YHijOO is missing. Select the correct alternative from the given
ones that will complete the series.
20. In the given series one word/one term/one number -2, -1/2, 1, ?, 4,
is missing. Select the correct alternative from the given (a) 1.5 (b)3.5
ones that will complete the series. (c) 2.5 (d) 2
Cream, Ampere, Revenge, Gene?
(a) Beat (c) Neat 27. In the given series one word/one term/one number
(c) Set (d) Heat is missing. Select the correct alternative from the given
ones that will complete the series.
21. In the given series one word/one term/one number 29, 31, 37, ?, 4
is missing. Select the correct alternative from the given (a) 38 (b) 40
ones that will complete the series. (c) 41 (d) 42
Ire, cow, moans, sheath, ?
(a) Aground (b) Splinted 28. If x < Y, W > x and w < z, which of the following must
(c) Thrashes (d) Flexed be true ?
I. Y < W ll. Z < X
22. In the given series one word/one term/one number (a) Neither l nor ll (b) Only
is missing. Select the correct alternative from the given (c) Only ll (d) Both l and ll
ones that will complete the series.
X, S, N, I, ? 29. The weights if five boxes are 20, 40, 70, & 90
(a) E (b) D kilograms. Which of the following cannot be the total
(c) F (d) G weight, in kilograms, of any combination of these
boxes?
23. In the given series one word/one term/one number (a) 190 (b) 180
is missing. Select the correct alternative from the given (c) 210 (d) 170
ones that will complete the series.
cau, dbv, ecw, fdx, ? 30. If the letters N, E, Y, G, S, & R are numbered 1, 2, 3,
(a) gey (b) gfz 4, 5, & 6 respectively. Select that Combination of
(c) fdc (d) fgh number of numbers so that letters arranged
accordingly, form a meaningful word.
Engineer’s platform
(a) 5312643 (b) 6425152 turns North and goes 6 km finally it turns left and goes 2
(c) 6345322 (d) 2614563 km to reach the end point. Where is the end point with
reference to the start point ?
31. If URBANITY is coded as FIYZMRGB, then how be (a) 4 km to the West of the starting point.
coded as ? (b) 8 km to the West of the starting point
(a)YHN (b) NHY (c) 8 km to the East of the starting point
(c) SLD (d) TYU (d) 4 km to the East of the starting point

32. In a certain code language, 528 means ‘leaf is fast’ 39. Two children A and B are playing. B is standing 5 m
513 means ‘glass is curved’ 923 means ‘flat or curved’, to the North of A, B then walks 10 m East then turns to
Find the code for ‘ir’. his right and walks 8 m. at the same time, A has walked
(a) 9 (b) 8 2 m to the west, then has turned to his left and walked
(c) 4 (d) 2 3 m Where is B now with respect to the position of A ?
(a) B is 8 m to the East of A
33. In a certain code language, ‘+’ represents ‘X’, ‘-‘ (b) B is 12 m to the West of A
represents ‘+’, ‘X’ represents ‘ ÷’ and ‘÷’ represent ‘-‘ . (c) B is 12 m to the East of A
Find out the answer to the following question. (d) B is 8 m to the West of A
0.002 + 8000 – 126 ⨯3 = ?
(a) 58 (b) 26 40. In the question two statements are given following
(c) 38 (d) 21 by two conclusions I and II. You have to consider the
statements to be true even if it seems to be at variance
34. If A % B means A is father of B, A & B means A is from commonly known facts. You have to decide which
sister of B and A $ B means A is daughter of B, then of the given conclusions, if any, follows from the given
what does P % Q & R $ S mean? statements.
(a) S is aunt of P (b) S is sister of P Statement 1 : All apples are red.
(c) S is husband of P (d) S is mother of P Statements 2 : Some red things are fruits.
Conclusion l : some fruits are apples.
35. If 91 # 32 =7, 83 # 54 = 4 then find the value of 62 # Conclusion ll : All red things are fruit.
96 = ? (a) Neither l nor ll follows
(a) 10 (b) 12 (b) Only conclusion l follows
(c) 17 (d)1 (c) Only conclusion ll follows
(d) Either l or ll follows
36. Which of the following words follows the trend of
the given list? Goodbye, Popeye, Butyne, Layman, 41. In the question a statement is given followed by
Lychee,.? two arguments, l and ll. you have to consider the
(a) Dingy (b) Lofty statement to be true even if it seems to be at variance
(c) Lying (d) Yatch from commonly known facts. You have to decide which
of the given arguments, if any, is a strong argument.
37. Which of the following words follows the trend of Statement: should children be allowed to use
the given list? smartphones?
(a) OQOQOQ (b) OQOOQO Argument l : No, Smartphones are addictive, they
(b) QOQOQO (d)OOQQOO reduce study time and play time.
Argument ll : Yes, The future is about technology. The
38. The marathon route goes 6 cm. south from the sooner the children learn how to use technology the
starting point. It then turns West and goes 6 kms, then better for their and society’ s future.
Engineer’s platform
(a) if neither argument l nor ll is strong 45. If the following figure, square represents Hindi
(b) if only argument ll is strong speakers, triangle represents Tamil speakers, circle
(c) if only argument l is strong represents Marathi speakers and rectangle represents
(d) if both argument l and ll are strong English speakers. Which set of letters represents English
speakers who do not speak Hindi and Tamil speakers
42. Which of the following cube in the answer figure who do not speak English ?
cannot be made based on the unfolded cube in the
question figure?

(a) L, K, M, N and O
(b) L, N, M, K, and O
(c) I, J, M, K, and O
(d) I, J, K, and O

46. Which answer figure will compete the pattern in the


43. Which of the following answer figure patterns can question figure?
be combined to make the question figure?

47. From the given answer figures, select the one in


which the question figure is hidden/embedded.

44. Which of the following diagrams represent the


relationship between Chinese, Indians and Asians?

48. A piece of paper is folded and punched as shown


below in the question figures. From the given answer
figures, indicate how it will appear when opened.

Engineer’s platform
(a) 40, 31, 41, 68, 01
(b) 23, 95, 14, 20, 79
(c) 11, 41, 44, 85, 22,
(d) 01, 34, 22, 10, 75
49. If a mirror is placed on the line MN, then which of
the answer figures is the right image of the given
figure? Answer:

1.(b) 2.(a) 3.(d) 4.(b) 5.(b) 6.(c) 7.(a)


8.(d) 9.(b) 10.(a) 11.(a) 12.(d) 13.(d) 14.(b)
15.(c) 16.(a) 17.(a) 18.(b) 19.(a) 20.(b) 21.(a)
22.(b) 23.(a) 24. (c) 25.(a) 26.(c) 27.(c) 28.(a)
29.(c) 30.(a) 31.(c) 32.(a) 33.(a) 34.(c) 35.(d)
36.(d) 37.(c) 38.(b) 39.(c) 40.(a) 41.(d) 42.(d)
43.(a) 44.(c) 45.(a) 46.(a) 47.(a) 48.(a) 49.(c)
50.(d)

EXPLANATIONS
50. A word is represented by only one set of numbers as
1. Cash is kept in wallet, similarly pickle is kept in jar.
given in any one of the alternatives. The sets of
numbers given in the alternatives represented by two
2. Cat: Paws. Here the cat’s relationship has been told
classes of alphabets as shown in the given two matrices.
from his claws (cat’s feet) Goat : Hoof. Here the goat is
The columns and rows of Matrix-l are numbered from 0
related to the hoof (goat’s feet).
to 4 and the of Matrix-ll are numbered from 5 to 9. A
letter from these matrices can be represented first by
3. Hare the relation of the singular and plural terms of
its row and next by its column, for example ‘U’ can be
the word has been given. Such as leaf : leaves similarly
represented by 30, 41 etc and ‘E’ can be represented by
focus: foci
85, 67, etc. Similarly, you have to identify the set for the
4.
word ‘SWORD’

Engineer’s platform
5. The ratio of the number of characters in the words is 19. In the given sequence, 1st and 2nd characters of the
5 : 7. first term are in Small letter, 2nd and 3rd characters in
Mouse (5) : Museums (7) :: Worst (5) : Sorrows (7) the second term are in Small letter Hence the correct
answer are ZXCA sd.
6. In this sequence, the last 3 letters of the word are the
starting letters of the next word. The ‘son’ of ‘Treason’ 20. In this sequence, the last 2 letters of the word are
is the starting letter of Sonnets. Similarly, ‘Bes’ of the starting letters of the next word. Like ‘am’ of
‘Bribes’ are the initial alters of ‘Bestow’ Also the ‘cream’ is the starting letter if ‘ampere’ Similarly, ‘ne’
number if letters is also equal. of ‘gene’ is the starting letter if ‘Neat’.

21. The number of character if the word is increasing,


7. 78 : 15 ⇒ (7 + 8) = 15
respectively. The first word has 3 letters, 4 in the
Similarly, 67 : 13 ⇒ (6 + 7) = 13.
second, and in the end the answer will be of 7
The correct answer is 13.
characters. The right answer will be aground.

8. 111 : 120 ⇒ (111 +9 ) = 120


22.
Similarly, 222 : 231 ⇒ (222 + 9) = 231
The correct answer is 231.

9. 123 : 444 ⇒ 123 ÷ 321(123’s reverse is


321) = 444
24. In the given sequence, the position of the last O is
Similarly, 431 : 565 ⇒ 431 +134 = 565.
moving forward from every term forward.
The correct answer is 565.
OOXXXXX ⇒ O’s position 1st and 2nd
OXOXXXX ⇒ O’s position 1st and 3nd
10. Only sounds are heard in the radio, while the
television laptops and smartphones also share the XOOXXXX ⇒ O’s position 2st and 3nd
voice along with the picture. XOXOXXX ⇒ O’s position 3st and 4nd
XXOOXXX ⇒ O’s position 3st and 4nd
12. Bow and shield ⇒ They were used as auxiliary
weapon. The objects shown in other options are used 25. The correct answer is 10.
to kill the enemy. 7, 5 2, -2, -7
The given sequence follows the given pattern
13. D-A, G-D, and I-F there is a difference of two 7-(2) = 5
letters between these letters. Whereas the difference 5 – (3) = 2
between M-O is one letter. The correct answer will be 2 – (4) = -2
MO. -2 – (5) = -7

14. Except ‘Sultry’, “nsul”’ is used in all other words. 26. The given series is in A.P. with common difference
‘3/2’
15. Except ‘Houy’ Except the word, “AB” is used in all
other words.

16. ‘666’ on any number, the digits are not repeated


thrice

18. 132 = 169, 142 = 196, 162 = 256 but 172 = 289.

Engineer’s platform
27. 29, 31, 37, 41, 43, The given series is of prime 36. In the given question, the position of Y is moving
number backward in every word respectively. As such, the
position of ‘Y’ in Goodbye, ‘is 6th , in ‘Popeye’, 5th
29. 190 = 20 + 40 +40 + 90 similarly, the position of ‘Y’ in the last term will be 1 so
180 = 20 + 70 + 90 the correct answer is Yatch.
170 = 90 + 40 +40 37.
Which ‘200 km’ cannot be represented by any of the
combination.

30. N = 1, E = 2, Y = 3, G = 4, S = 5, R = 6
5312643 = SYNERGY (meaningful)
64255152 = RGESNSE
6345321 = RYGSYEN 38.
2614563 = ERNGSRY

31. On reading the English alphabet from the beginning,


the place of letters of U, R, B, A, N, I, T, Y, are 21, 18, 1,
14, 9, 20, 25, respectively.
On reading the English alphabet from the beginning, the
place of letters of F, I, Y, Z, M, R, G, B, are 21, 18, 2, 1,
14, 9, 20, 25, respectively
Similarly,
On reading the English alphabet from the beginning, the Marathon race begin from point A to 6km B, than 6km
place of letters of H, O, W, is 8, 15, 23, respectively. from B to C than 6km from C to D, than 2km from D to
On reading the English alphabet from the back, the E.
position of 8, 15, 23 is of the letters S, L and D, ⇒ EA = ED + DA
respectively. = ED + CB (CB = DA)
33. = 2 + 6 = 8 KM
Hence marathon race is 8km to the west of starting
point.
39.

34. Condition (1) A%B = A is the mother of B. So P@Q =


P is the mother of Q.
Condition (2) A&B = A is sister of B. So Q#R = Q is sister
of R, so P is the mother of Q and R
Condition (3) A$B = A is the daughter of B, so R!S = R is
the daughter if S, So S is the father of R, so S is husband
of P
35. Given,
B, begin to move 10m from point P to Q than 8 m
from Q to R begin to move 2m from point V to U, than
3m from U to T.
⇒ TR = TS +SR
= UV + PQ (TS = UV , ST = PQ)
Engineer’s platform
=2+10 = 12M
Hence, 8km to the west of the starting point.
40.

41. Smartphones is am important tool for kids to


connect and learn technology. Learning to use
technology is impoetant for the future to become
good but more use of smartphones is also adversely
affecting children’s mental education and physical
cducation Therefore, argument l and ll, both strong.

44.

47.

50. SWORD = 01, 34, 22, 10, 75


S = 00, 01
W = 34
O = 22, 40
R = 10, 12, 13
D = 75

Engineer’s platform
Solved Paper 2017 (Set-5) 8. Select the related word/letters/number from the
1. Select the related word/letters/number from the given alternatives.
given alternatives. 111: 222 :: 444: ?
Branch: Plant : Limb: ? (a) 888 (b) 666
(a) Dog (b) Walk (c) 777 (d) 333
(c) Hand (d) Animal
9. Select the related word/letters/number from the
2. Select the related word/letters/number form the given alternatives.
given alternatives. 453: 99 : : 642: ?
Cactus: Thorns:: Sheep:? (a) 765 (b) 346
(a) Meat (b) Wool (c) 867 (d) 396
(c) Milk (d) Graze
10. Select the odd word/letters/number/word pair/
number pair from the given alternatives
3. Select the related word/letters/number from the
(a) Violet (b) Indigo
given alternatives.
(c) White (d) Blue
Dawn: Dusk::?
(a) Then: Now (b) Late: Never
11. Select the odd word/letters/number/word pair/
(c) Summer: Winter (d) North: South
number pair from the given alternatives.
(a) Tea and Coffee (b) Bread and Butter
4. Select the related word/letters/number from the (c) Cup and Saucer (d) Wine and Cheese
given alternatives.
YB : : DW : : CX : ? 12. Select the odd word/letters/number/word
(a) AZ (b) UF pair/number pair from the given alternatives.
(c) AB (d) ZA (a) Bungalow and Villa
(b) Office and Shop
5. Select the related word/letters/number from the (c) House and Apartment
given alternatives. (d) House and Villa
Thin: Ninth :: Hole: ?
(a) Hell (b) Hello 13. Select the odd word/letters/number/word pair/
(c) Loo (d) Heel number pair from the given alternatives.
(a) OP (b) IJ
6. Select the related word/letters/number from the (c) SU (d) DE
given alternatives.
Care: Area :: Amine:? 14. Select the odd word/letters/number/word pair/
(a) Define (b) Dine number pair from the given alternatives.
(c) Inertia (d) Inept (a) fluffiness (b) finely
(c) definite (d) define
7. Select the related word/letters/number from the
given alternatives. 15. Select the odd word/letters/number/word
1000 : 3 : : 100000: ? pair/number pair from the given alternative
(a) AXUI (b) ELAM
(a) 6 (b) 4
(c) ASIV (d) YREV
(c) 5 (d) 3

16. Select the odd word/letters/number/word pair/


number pair from the given alternatives.
Engineer’s platform
(a) 4567 (b) 2345 (a) dfe (b) Cva
(c) 8765 (d) 6789 (c) fil (d) frs

17. Select the odd word/letters/number/word pair/ 24. In the given series one word/one term/one
number pair from the given alternatives. number is missing. Select the correct alternative from
(a) 101 (b) 37 the given ones that will complete the series.
(c) 225 (d) 65 XOXOXXX, XXOOXXX, XXOOXXX, XXOXOXX,?
(a) XOXXOXX (b) XOXOXXX
18. Select the odd word/letters/number/word pair/ (c) XXOOXXX (d) XXOXXOX
number pair from the given alternatives
(a) 112, 147 (b) 130, 152
25. In the given series one word/ one term/one
(c) 126, 161 (d) 119, 133
number is missing. Select the correct alternatives from
the given ones that will complete the series.
19. In the given series one word/one term/one
13, 11, 8, ?, -1
number is missing. Select the correct alternative from
(a) 5 (b) 3
the given ones that will complete the series.
(c) 4 (d) 1
Buzzwords, Preacher, Bonanza, Credit?
(a) Slam (b) Honey
26. In the given series one word/one term/one
(c) Mole (d) Pace
number is missing. Select the correct alternative from
the given ones that will complete the series
20. In the given series one word/one term/one
-11/4, -2, 5/4, ?, 1/4
number is missing. Select the correct alternative from
(a) 1/2 (b) -0.75
the given ones that will complete the series.
(c) 0.75 (d)-1/2
Crazy, Zygote, Teapot, Other, ?
(a) Time pass (b) Errand
27. In the given series one word/one term/one
(c) Roads (d) Elephant
number is missing. Select the correct alternative from
the given ones that will complete the series.
21. In the given series one word/one term/one
19, 7, 29, 31, 37
number is missing. Select the correct alternative from
(a) 21 (b) 23
the given ones that will complete the series.
(c) 22 (d) 26
pal, jowl, spine, griped, ?
(a) scurries (b) sentries
28. If v < y, x < y, w < z and z > y, which of the following
(c) ancient (d) queasy
is true?
(I) z > v (II) w > v (III) x < z
22. In the given one word/one term/one number is
(a) II only (b) III only
missing. Select the correct alternative from the given
(c) II and III only (d) I and III only
ones that will complete the series.
G, I, K, M,? 29. The weights of five boxes are 10, 30, 50, 70, & 80
(a) P (b) O kilograms. Which of the following cannot be the total
(c) N (d) Q weight, in kilograms, of any combination of these
boxes?
23. In the given series one word/one term/one (a) 160 (b) 220
Number is missing. Select the correct alternative from (c) 180 (d) 150
the given ones that will complete the series.
beh, efi dgi ehk, ?
Engineer’s platform
30. If the letters, I, N, A, T, E, & C are numbered 1, 2, 3, (c) IIIIVIII (d) IIIVVIII
4, 5, & 6 respectively. Select that combination of
numbers so that letters arranged accordingly, form a 38. A fishing boat sails 5 km South in still water, then
meaningful word. turns East and sails 8 km, then turns North and sails 5
(a) 3261524 (b) 6543421 km, then turns to its right and sails 10 km. Where is
(c) 6214523 (d) 2541562 the boat now with reference to its starting position?
(a) 2 km to the West of the starting position.
31. If CONFUSED is coded as XLMUFHVW, then (b) 18 km to the West of the starting position.
Similarly, how will BAT be coded? (c) 2 km to the East of the starting position.
(a) YZG (b) RGD (d) 18 km to the East of the starting position.
(c) TGO (d) APV
39. Two people A and B start walking from the same
32. In a certain code language, 462 means 'lion is wild' point. A walks 15 m East, then turns south and walks5
543 means 'cat is hungry'. Find the code for 'and'. m. At the same time, B walks 5 m South, then he turns
(a) 5 (b) 3 to his left and walks 25 m. Where is B now with
(c) 1 (d) 9 respect to the position of A?
(a) B is 35m to the East of A
33. In a certain code language'+' represents 'x’, ‘-’ (b) B is 10m to the East of A
represents '+', 'x' represents '/' and ‘/’ represents ‘-’. (c) B is 10 m to the West of A
Find out the answer to the following question. (d) B is the 35 m to the West of A
0.25 + 240 - 300 x 5 = ?
(a) 0 (b) 120 40. In the question two statements are given, followed
(c) 1 (d) 2.5 by two conclusions. I and IL. You have to consider the
statement to be true even if seems to be at variance
34. If A@B means A is mother of B, A#B means A is from commonly known facts.
sister of B and A!B means A is granddaughter of B, You have to decide which of the given conclusions, if
then what does P#Q!R@S mean, if R has only one any, follows from the given statements.
child? Statement 1: All babies are toys.
(a) R is mother of P (b) Q is sister of S Statement 2: All children are toys
(c) S is mother of P (d) P is sister of s Conclusion I: Some children are babies.
Conclusion II: No child is a baby.
35. If 67 x 25 = 6, 24 x 12 = 3, then find the value of 90 (a) Only conclusion I follows
x 44=?
(b) Only conclusion II follows
(a) 2 (b) 1
(c) Either I or II follows
(c) 3 (d) 4
(d) Neither I nor II follows

36. Which of the following words follow the trend of


41. In the question statement is given, followed by two
the given list? arguments, I and II, you have to consider the
Vertex, Affix, Unix, Waxy, Extra,? statement to be true even if it seems to be at variance
(a) Annex (b) Xray from commonly known facts. You have to decide
(c) Apex (d) Axial which of the given arguments, if any, is a strong
argument.
37. Which of the following terms follow the trend of Statement: Should prison term of those criminals who
the given list? have shown good behavior be reduced?
IVII, IIVII, IIIVII, ……… Argument I: No, if they are freed they will again commit
(a) IIIVIII (b) IIIIVII crimes.
Engineer’s platform
Argument II: Yes, this will incentivize others to show 45. In the following figure, square represents boys,
good behavior triangle represents girls, circle represents cricket
(a) if only argument I is strong players and rectangle represents hockey players.
(b) if neither argument I nor II is strong Which set of letters represent girls who not play
(c) if both arguments I and II are strong hockey and boys who do not play cricket?
(d) if only argument II is strong

42. Which of the following cube in the answer figure


cannot be made based on the unfolded cube in the
question figure?
(a) U, R, S and T (b) U. Q, T and S
(c) R, V, Q and U (d) V, S, R and T

46. Which answer figure will complete the pattern in


the question figure?

43. Which of the following answer figure patterns can


be combined to make the question figure?

47. From the given answer figure, select the one in


which the question figure, is hidden/embedded.

44. Which of the following diagrams represent the


relationship between hockey players, football players
and those who have won medals in both hockey and
football? 48. A plece of paper is folder and punched as shown
below in the question figures. From the given answer
figures, indicate how it will appear when opened.

Engineer’s platform
(a) 12, 13, 41, 55, 69
(b) 31, 01, 65, 58, 41
(c)24, 20, 40, 31, 76
(d) 21, 40, 04, 69, 01

ANSWERS:
1.(d) 2.(b) 3.(c) 4.(d) 5.(b) 6.(d) 7.(c)
8.(a) 9.(d) 10.(c) 11.(c) 12.(b) 13.(c) 14.(c)
15.(a) 16.(c) 17.(c) 18.(b) 19.(b) 20.(b) 21.(c)
22.(b) 23.(c) 24.(d) 25.(c) 26.(d) 27.(b) 28.(d)
49. If a mirror is placed on the line MN, then which of 29.(b) 30.(a) 31.(a) 32.(c) 33.(b) 34.(c) 35.(b)
the answer figures is the right image of the given 36.(b) 37.(b) 38.(a) 39.(b) 40.(c) 41.(d) 42.(c)
figure?
43.(c) 44.(c) 45.(b) 46.(b) 47.(b) 48.(d) 49.(b)
50.(d)

EXPLANATIONS

2. Cactus: Thorns => Thorns are present cactus leaves.


Similarly, wool can be obtained from sheep's body.

4. YB: DW:: CX:?


50. A word is represented by only one set of numbers
as given any one of the alternatives. The sets of
numbers given in the alternatives are represented by
two classes of alphabets as shown in the given tw
matrices. The columns and rows of Matrix-I are
numbered from 0 to 4, and that of Matrix-I are
numbered from 5 to 9. A letter from these matrices
can be represented first by its row and next by its
column, for example T can be represented by 10, 32
etc. and 'A' can be represented by 65, 56 etc. Similarly,
you have to identity the set for the word "WOVEN"
5. Thin: Ninth ↔ 4:5 (Ratio of the number of letters),
Similarly, Hole: Hello ↔ 4:5

7. 1000:3 ↔ [1000:3 (Number of zeros), 100000:5 ↔


[100000:5 (Number of zeros)]

8. 111:222 ↔ [111 x 2 = 222], Similarly, 444:888 ↔


[444 x 2 = 888]

9. 453:99 ↔ [4 + 5 + 3): (9 + 9) = 12:18], Similarly,


642:396 ↔ [(6 + 4 + 2): (3 + 9 +6) = 12:18]

Engineer’s platform
11. Cup: saucer This option only comes in a group of 11 => (11-3 = 8)
utensils. While the other options are related foodstuffs. 8 => (8 - 4 = 4)
4 => (4 - 5 = 1) => -1
12. Office: Shop: Only this option comes in the business
area. While the other options is related to the resident 3
26. The given series is in AP with common difference .
area. 4

11 3
14. In fluffiness, finely and define, fine' is commonly − + ( ) = −2
4 4
used.
3 5
15. YREV, only this word starts with consonant, while −2 + ( ) = −
4 4
the other word begins with vowel.
5 3 1
16. 8765 => Only these numbers are written in − +( )=−
4 4 2
descending order, whereas all the other numbers are
1 3 1
written in descending order. − +( )=
2 4 4
17. 225 => [152= 225], is complete square number
27. 19, (23), 29, 31, 37. The given series is of prime
18. 130, 152 Except this option, all the other options are number.
completely divided by 7.
28. v < y and z > y => z > y > v …(1)
19. The number of letters of each word is gradually x < y and z > y => x < y < z ...(2)
decreasing. From eq. (1) (z > v) is correct.
and from eq. (2) (x < z) is correct.
20. The last two letters of the word are the starting
words of the next word. 29. 160 = 30+50+80
180 = 30+70+80
[Crazy => Zygote], [Zygote => Teapot], [Teapot => 150 = 30+50+70
Other], [Other => Errand] While ‘220 kg' cannot be represented by any of the
combination.
21. The number of letters of each word is increasing
gradually. 30. 3261524 = ANCIENT (meaningful)
65434221 = CETATNI
22. G (+2) = I
6214523= CNITENA
I (+2) = K
2541562= NETIECN
K (+2) = M
M (+2) = O.
31. CONFUSED = 3, 15, 14, 6, 21, 19, 5, 4 (On the
23. reading of Numeric position of letters from A to
Z)XLMUFHVW =3, 15, 14, 6, 21, 19, 5, 4 (On the reading
of Numeric position of letters, from Z to A)
Similarly,
BAT => 2, 1, 20 (YZG)

24. The ‘O' in the second position is progressing in each 32. 462 (Lion is wild) ...(1)
term, respectively. 543 (cat is hungry) ...(2)
XOXOXXX, XXOOXXX, XXOOXXX, XXOXOXX, XXOXXOXX 165 (wild and hungry) ...(3)
25. 13 => (13-2=11) From (1) and (3)
Engineer’s platform
6 = (wild) ...(4) Fishing boat move from point A to 5km B, then 8 km
From (2) and (3) on removing common from B to C, then 5km from C to D, then 10 km from D
5 = (hungry) ...(5) to E.
By comparing (4) and (5) => 1 = (and) AE = AD+DE
= BC+DE (AD = BC)
33. Given, = 8 + 10 = 18 km
‘+’ → ‘x’, ‘-’ → ‘+’, ‘x’ → ‘÷’ ‘÷’ → ‘-’ Hence, the fishing boat is 18km to the east of the
On changing the sign of eq. we get starting position.

300 300 43.


(0.25 ∗ 240 + ) = (0.25 ∗ 240) + ( )
5 5
= 60 + 60 = 𝟏𝟐𝟎

34. Given,

Condition (1) A@B = A is the mother of B. So R@S, R is


the mother of S. R has only one child (given.) That is S.

Condition (2) A!B = A is the granddaughter of B. So QIR


A and B, begin to move from point 'O'A move 15m from
= Q is the granddaughter of R. So R, Q's grandmother.
O to P, than 5m from P to Q. B move 5m from O to R,
So S became the mother of Q.
than 25m from R to S.
Condition (3) A#B = A is the sister of B. So P#Q= P is the
QS = RS - RQ
sister of Q. So S is the mother of P.
= RS-OP (RQ = OP)
35. 67 x 25 = 6 => [(6+7) - (2+5) = 13-7=6], = 25-15 = 10m
24 x 12 = 3 => [(2+ 4) - (1+2) = 6 – 6 = 3] Hence, B is 10m to the east of A.
Similarly,
44.
90 x 44 = 1 => [(9 +0) – (4+4) = 9 - 8 =1].

36. The X's position is moving forward from the back to


the next, respectively. As such, in Vertex, the location of
X is 6th, 5th in affix, similarly, in the last term, the
position of X will be 1st (Xray).

37. Extra ‘I’ is increasing at the first position in each


post. E.g., IVII, IIVII, IIIVII, IIIIVI Condition (I) = Some players who play both hockey and
football.
42. Condition (II) = Players who win some medals, both play
hockey and football. Options (3) is correct.

50. WOVEN = 21, 40, 04, 69, 01


W = 21,
O = 03, 40
V = 04, 30, 42
E = 69
N = 01

Engineer’s platform
Solved Paper 2016 Ans. B

1. Select the relate word/letters/number/from the ven


alternatives. 7. Select the related word/letters/number from the
given alternatives.
Quarter : Half : : Single :?
99 : 18 : : 77 : ?
A . Multiple B. Double
A. 16 B. 20
C. Set D. Group
C. 12 D. 14
Ans B
Ans. D

2. Select the relate word/letters/number/from the


given alternatives. 8. Select the related word/letters/number from the
given alternatives.
Ball : Sun : : Cigarette : ?
586 : 364 : : 824 : ?
A. Pipe B. Smoke
A. 520 B. 740
C. White D. Fire
C. 442 D. 602
Ans A
Ans. D

3. Select the relate word/letters/number/from the


given alternatives. 9. Select the related word/letters/number from the
given alternatives.
Allot : A lot : : ?
224 : 112 : : 326 : ?
A. Already: All ready B. Sea : See
A. 189 B. 146
C. Than : Then D. Peace : Piece
C. 163 D. 176
Ans. A
Ans. C

4. Select the related word/letters/number from the


given alternatives. 10. Select the odd word/letters/number/word
pair/number pair from the given alternatives.
HFDB : GEC : : YWUS : ?
A. Cheek B. Toe
A. XVT B. TVX
C. Ear D. Nostril
C. RTVX D. XVTR
Ans. B
Ans. A

11. Select the odd word/letters/number/word


5. Select the related word/letters/number from the
pair/number pair from the given alternatives.
given alternatives.
A. Socks and Shoes B. Bow and Tie
Grace : Career : : Sitar : ?
C. Vest and Shirt D. Shirt and Coat
A. Saris B. Artist
Ans. B
C. Stair D. Star
Ans B
12. Select the odd word/letters/number/word
pair/number pair from the given alternatives.
6. Select the related word/letters/number from the
A. Wood and Chair B. Cotton and Pillow
given alternatives.
C. Cotton and Wool D. Fur and Pillow
Destine : Inertia : : Bison : ?
Ans. A
A. Sonnets B. Sonar
C. Cosine D. Brine

Engineer’s platform
13. Select the odd word/letters/number/word Ans. D
pair/number pair from the given alternatives.
A. RF B. AH 20. In the given series one word/one term/one number
C. IM D. OT is missing. Select the correct alternative from the given
Ans. A ones that will complete the series.
Super, Erratic, Icebox, Oxen, ?
14. Select the odd word/letters/number/word A. Lollipop B. Popcorn
pair/number pair from the given alternatives. C. Papyrus D. Envelop
A. Busted B. Blasted Ans. D
C. Disaster D. Caste
Ans. A 21. In the given series one word/one term/one number
is missing. Select the correct alternative from the given
ones that will complete the series.
15. Select the odd word/letters/number/word
pair/number pair from the given alternatives. run, anon, clasp, rotted, ?
A. ABB B. EFF A. confuses B. accolade
C. OPP D. HII C. revamp D. scoured
Ans. D Ans. D

16. Select the odd word/letters/number/word 22. In the given series one word/one term/one number
pair/number pair from the given alternatives. is missing. Select the correct alternative from the given
ones that will complete the series.
A. 428 B. 802
Y, U, Q, M, ?
C. 640 D. 537
A. J B. K
Ans. D
C. L D. I
Ans. D
17. Select the odd word/letters/number/word
pair/number pair from the given alternatives.
A. 81 B. 72 23. In the given series one word/one term/one number
is missing. Select the correct alternative from the given
C. 44 D. 63
ones that will complete the series.
Ans. C
kpu, jot, ins, hmr, ?
A. gnq B. fns
18. Select the odd word/letters/number/word
C. fop D. glq
pair/number pair from the given alternatives.
Ans. D
A. 136, 17 B. 152, 19
C. 160, 20 D. 148, 18
24. In the given series one word/one term/one number
Ans. D
is missing. Select the correct alternative from the given
ones that will complete the series.
19. In the given series one word/one term/one number XXXXOXO, XXXOXOX, XXOXOXX, XOXOXXX, ?
is missing. Select the correct alternative from the given
A. XOXOXXX B. XXXXOXO
ones that will complete the series.
C. XXXOXOX D. OXOXXXX
DDRFTG, CVVBHT, DCEERT, DCFRRT, ?
Ans. D
A. REDCVT B. DEEDCR
C. SDDAEW D. SDERWW
Engineer’s platform
25. In the given series one word/one term/one number A. 3532143 B. 1356412
is missing. Select the correct alternative from the given C. 6121341 D. 4321242
ones that will complete the series.
Ans. B
-7, -3, ?, 8, 15
A. 1 B. 2
31. If DISCERNS is coded as WRHXVIMH, then how will
C. 3 D. 5 AGO be coded as?
Ans. B A. TGB B. BGT
C. DFG D. ZTL
26. In the given series one word/one term/one number Ans. D
is missing. Select the correct alternative from the given
ones that will complete the series.
32. In a certain code language, 639 means 'wood is
2, 1.25, ?, -1/4, -1
hard', 657 means 'fur is soft', 135 means 'hard or soft'.
A. -1/2 B. 1/2 Find the code for 'or'.
C. -0.75 D. 0.75 A. 4 B. 1
Ans. B C. 6 D. 3
Ans. B

27. In the given series one word/one term/one number 33. In a certain code language, '+' represents 'x', '-'
is missing. Select the correct alternative from the given represents '+', 'x' represents '÷' and '÷' represents '-'.
ones that will complete the series. Find out the answer to the following question.
13, ?, 19, 23, 29 0.0625 + 800 - 234 x 6 = ?
A. 14 B. 15 A. 11 B. 89
C. 16 D. 17 C. 1.25 D. 66
Ans. D Ans. B
34. If A @ B means A is father of B, A # B means A is
28. If m > l, m < n and n < o, which of the following must brother of B and A ! B means A is son of B, then what
be true? does L # M @ N ! O mean?
I. l < o II. n > l A. M is husband of O B. N is uncle of L
A. Only I B. Both I and II C. L is brother of O D. L is father of O
C. Only II D. Neither I nor II Ans. B Ans. A

29. The weights of five boxes are 20, 30, 40, 70 & 90 35. If 62#51 = 0, 82#54 = 5 then find the value of 72#66
kilograms. Which of the following cannot be the total =?
weight, in kilograms, of any combination of these A. 7 B. 10
boxes? C. 12 D. 5
A. 180 B. 190 Ans. D
C. 200 D. 170
Ans. D 36. Which of the following words follows the trend of
the given list? Visual, Avarice, Savour, Salvage,
30. If the letters E, D, X, S, P & O are numbered 1, 2, 3, Saliva, ?
4, 5 & 6 respectively. Select that combination of A. Drive B. Reweave
numbers so that letters arranged accordingly, form a C. Eleven D. Envelop
meaningful word.
Ans. B
Engineer’s platform
37. Which of the following terms follow the trend of the 41. In the question a statement is given, followed by
given list? FEEFEE, EFEEFE, EEFEEF, ? two arguments, I and II. You have to consider the
A. FFEEFF B. EFEFEEF statement to be true even if it seems to be at variance
from commonly known facts. You have to decide which
C. FEEFEE D. FEFFEE
of the given arguments, if any, is a strong argument.
Ans. C
Statement : Should local train tickets be subsidized?
Argument I : No, In an environment where locals trains
38. A postman cycles 4 km West, then turns South and are overcrowded, subsidy will worsen the situation.
cycles 7 km, then turns East and cycles 4 km, then turns
Argument II : Yes, Local trains reduce number of
to his left and cycles 10 km. Where is he now with
vehicles on the road, thus reducing pollution, reduce
reference to his starting position?
costs and travel time.
A. 17 km North from the starting position
A. if neither argument I nor II is strong
B. 3 km North from the starting position
B. if only argument II is strong.
C. 3 km South from the starting position
C. if only argument I is strong.
D. 17 km South from the starting position
D. If both argument I and II are strong
Ans. B
Ans. D

39. Two persons A and B start from the same point. A


42. Which of the following cube in the answer figure
walks 4 m North, then turns to his right and walks 3 m.
cannot be made based on the unfolded cube in the
At the same time, B has walked 9 m East , then he
question figure?
turned to his left and walked 4 m. Where is B now with
respect to the position of A?
A. B is 6 m to the East of A
B. B is 12 m to the East of A
C. B is 6 m to the West of A
D. B is 12 m to the West of A
Ans. A

40. In the question two statements are given, followed


by two conclusions, I and II. You have to consider the
statements to be true even if it seems to be at variance
from commonly known facts. You have to decide which Ans. A
of the given conclusions, if any, follows from the given
statements. 43. Which of the following answer figure patterns can
Statement 1 : Some villagers are poor. be combined to make the question figure
Statement 2 : All poor are nice people.
Conclusion I: So me nice people are villagers.
Conclusion II: No villagers are nice people.
A. Only conclusion I follows
B. Only conclusion II follows
C. Either I nor II follows
D. Neither I nor II follows
Ans. A

Engineer’s platform
44. Which of the following diagrams represent the 47. From the given answer figures, select the one in
relationship between Chinese, Indians and Asians? which the question figure is hidden/embedded.

Ans. B

45. In the following figure square represents hockey Ans. C


players, triangle represents cricket players, circle
represents kabaddi players and rectangle represents
football players. Which set of letters represents hockey 48. A piece of paper is folded and punched as shown
players who do not play football and cricket players below in the question figures. From the given answer
who play kabaddi? figures, indicate how it will appear when opened

A. O, U And P B. R, S and U
C. O, S and U D. Q, P and T
Ans. C
Ans. C
46. Which answer figure will complete the pattern in
the question figure? 49. If a mirror is placed on the line MN, then which of
the answer figures is the right image of the given
figure?

Ans. C

Engineer’s platform
Ans. D

50. A word is represented by only one set of numbers as


given in any one of the alternatives. The sets of
numbers given in the alternatives are represented by
two classes of alphabets as shown in the given two
matrices. The columns and rows of Matrix-I are
numbered from 0 to 4 and that of Matrix-II are
numbered from 5 to 9. A letter from these matrices can
be represented first by its row and next by its column,
for example, ‘Z’ can be represented by 20, 23 etc., and
‘A’ can be represented by 65, 57 etc. Similarly, you have
to identify the set for the word “TOWER”.

A. 23,95,14,20,79
B. 40,31,41,68,01
C. 11,41,44,85,22
D. 21,40,04,69,01
Ans. C

Engineer’s platform
Solved Paper 2015
7. R I A T N I E ?
Directions (Q. 1-2): Select the related number from the
(a) A (b) B
given responses.
(c) C (d) D
1.
𝟏 𝟏 𝟏
8. (𝟖) , (𝟒) , (𝟐) , 𝟏, ? , 𝟒

3 2
(a) (8) (b) (8)
(c) 2 (d) 6

9. 11, 12, 16, 25, ?


(a) 45 (b) 41
(a) 64 (b) 69 (c) 43 (d) 49
(c) )65 (d) 68
10. 3, 9, 21, 45,?
2. 20 30 12 (a) 54 (b) 78
3 4 8 (c) 87 (d) )93
80 ? 116
(a) 120 (b) 60 Directions (Q. 11-19): Select the related
(c) 100 (d) 140 word/letters/number from the given alternatives.

3. Hospital is 12 km towards east of Rupin's house. His 11. CHAIR: FURNITURE :: FORK: ?
school is 5 km towards south of Hospital. What is the (a) SPOON (b) CUTLERY
shortest distance between Rupin's house and school? (c) CROCKERY (d) FOOD
(a) 16 km (b) 17 km
(c) 12 km (d) 13 km 12. Compass : Ship:: Vaastu: ?
(a) Building (b) Flat
4. Two cars started from a particular spot. The car A (c) Home (c) Land
ran straight at the speed of 30 km/h for 2 hours north
and then took a right turn. It ran 40 km and again 13. BOOK: LIBRARY :: ? : FILE
(a) COMPUTER (b) DATA
turned right. It stopped after 30 km. The car B ran
(c) FOLDER (d) BYTES
straight towards east at the speed of 20 km/h for 2
hours and turned left. It ran for 10 km and then
14. q:d :: b:?
stopped. How far were these two cars from each other (a) p (b) d
when both of them stopped at last? (c) q (d) b
(a) 17 km (b) 18 km
(c) 19 km (d) 20 km 15. ABB: EGJ:: FHL:?
(a) BDH (b) JMT
Directions (Q. 5-10): A series is given with one term (c) FHH (d) JJL
missing. Choose the correct alternative from the given
ones that will complete the series. 16. EV: KP:: TG:?
(a) ZA (b) AZ
(c) ZZ (d) AA
5.CDDP DEER EEFT FGGV GHHX?
(a) ZIIH (b) HIIZ 17. 21: 65:: 31:?
(c) HJJY (d) HILZ (a) 78 (b) 80
(c) 85 (d) 95
6. l m n m n o p n o p q r ?
(a) pqrst (b) lmnop
(c) opqrs (d) hpqrs
Engineer’s platform
18. 17: 102:: 23: ? Directions (Q. 29-31): Which one of the given responses
(a) 112 (b) 138 would be a meaningful order of the following?
(c) 216 (d) 413
29. 1. Village 2. State
19. 25: 36:: ? 3. Nation 4. District
(a) 9: 25 (b) 16: 25
(a) 1,2, 4,3 (b) 1,4, 2, 3
(c) 25:49 (d) 81: 121
(c) 2,3, 1, 4 (d) 4,2, 3, 11
Directions (Q. 20-27): Select the related word letters/n
umber from the given alternatives. 30. 1. Branches 2. Root
3. Trunk 4. Leaf
20. 5. Flower
(a) stare (b) glance (a) 4, 1, 3, 2, 5 (b) 2,3, 1, 4, 5
(c) look (d) hug
(c) 1,2,3, 4, 5 (d) 4, 3, 1, 2,5
21.
(a) Analogy (b) Reasoning 31. 1. Adulthood 2. Babyhood
(c) Decoding (d) Cycling 3. Childhood 4. Infancy
(a) 4, 3, 2, 1 (b) 4, 2, 3, 1
22. (c) 4, 1, 2, 3 (d) 4, 3, 1, 2
(a) Nephrology (b) Astrology
(c) Pathology (d) Entomology
32. Ramu's mother has three sons. The eldest one is
called onekari, the second one is called twokari. Then
23. the third son's name is
(a) accdff (b) prrsuu (a) Teenkari (b) Sandu
(c) mnnoqq (d) egghjj (c) Ramu (d) Nokari

24. 33. Ashok is heavier than Gopal. Mahesh is lighter than


(a) OQTX (b) JMNQ Jayesh. Prashant is heavier than Jayesh but lighter
(c) EGJN (d) XZCG than Gopal. Who among them is heaviest?
(a) Gopal (b) Ashok
25. (c) Prashant (d) Mahesh
(a) NMOK (b) PKQJ
(c) RLSK (d) TGUF 34. From the given alternative words, select the word
which cannot be formed using the letters of the given
26. word:
(a) 997 (b) 976 KILOMETERS
(c) 778 (d) 895 (a) OIL (b) MEET
(c) TREES (d) STREET
27.
(a) 8 (b) 8 35. In a certain code language, if the word RHOMBUS'
(c) 111 (d)9 is coded as TJQODWU, then how is the word
‘RECTANGLE' in that language?
28. Pick the odd number from the sequence below: (a) TGEVCPIMG (b) TGEVCPING
2, 3, 6, 7, 11, 15, 30 (c) TGEWDPING (d) TGFWEPING
(a) 7 (b) 11
(c) 6 (d) 30

Engineer’s platform
36. If in a certain code 'Education' is written as 3 6 5 7
9 8 2 1 4 then how ‘Conduct' can be written?
(a) 7 1 4 6 5 7 8 (b) 6 5 4 7 8 7 1
(c) 1 4 5 8 7 7 6 (d) 6 4 8 5 7 6 7

37. If 7x = 8k and 5y = 6k, then the value of ratio x is to


y is
(a) 20:21 (b) 21:20
(c) 35:48 (d) 48:35
Directions (Q. 42-43) : In questions one/ two
38.If 44+12=30, 77+14=61, 84+16=66 then what should
statement(s) are given followed by two conclusion
be for 44+22=?
assumption, I and II. You have to consider the
(a) 28 (b) 20
statements to be true even if they seem to be at
(c) 32 (d) 24
variance from commonly known facts. You have to
decide which of the given conclusion /assumptions, if
39. Select the set of symbol which can be fitted
any, follows from the given statements
correctly in the equation.
8__4__2__6__3 = 32
42. Statements:
(a) x, -, +, / (b) +, x, /, -
1. Tigers do not fly
(c) +, /, x, - (d) -, x, /, +
2. Hens do not fly
Conclusions:
40. A word is represented by only one set of numbers
I. Tigers are birds
as given in any one of the alternatives. The sets of
II. All birds cannot fly
numbers given in the alternatives are represented by
two classes of alphabets as in two matrices given (a) Only I follows
below. The columns and rows of Matrix-I are (b) Only II follows
numbered from 0 to 4 and that of Matrix-II are (c) Either I or II follows
numbered from 5 to 9. A letter from these matrices (d) Neither I nor II follows
can be represented first by its two and next by its
column, e.g., M' can be represented by 01, 14 etc., and 43. Statements:
S' can be represented by 58, 77 etc. Similarly, you have All students are girls.
to identify the set for the word ROHAN". Some students are not talented.
Conclusions:
I. No student is talented.
II. Some girls are talented.
(a) Only I follows
(b) Only II follows
(c) Both I and II follow
(d) Neither I nor II follows

(a) 11, 57, 00, 55, 12 (b) 11, 75, 00, 55, 10 44. In the given figure, the circle stands for intelligent,
(c) 32, 75, 21, 55, 10 (d) 32, 67, 41, 55, 12 square the hardworking, triangle for Post graduate and
the rectangle for loyal employees. Study the figure and
41. I fa mirror is placed on the line MN, then which of answer the following questions.
the answer figures is the right image of the given
figure?
Engineer’s platform
48. Write the number of space enclosed by rectangle
and circle but not by triangle.

Employees who are intelligent, hardworking and loyal


but not Post graduate are represented by (a) 3 (b) 2
(a) 11 (b) 5 (c) 1 (d) 4
(c) 4 (d) 3
49. Find the number of minimum straight lines
45. A piece of paper is folded and cut as shown below required to make figure.
the question figures. From the given figures indicate
how it will appear when opened.

(a) 13 (b) 17
(c) 15 (d) 19

50. How many triangles are there in the figure?


46. From the given answer figures, select the one in
which the question figure is hidden/embedded.

(a) 7 (b) 10
(c) 16 (d) 20

47. Which answer figure will complete the pattern in


ANSWERS
the question figure.
1.(d) 2.(d) 3.(d) 4.(d) 5.(b) 6.(c) 7.(b)
8.(c) 9.(b) 10.(d) 11.(b) 12.(d) 13.(b) 14.(a)
15.(b) 16.(d) 17.(d) 18.(b) 19.(b) 20.(d) 21.(d)
22.(d) 23.(c) 24.(b) 25.(a) 26.(a) 27.(a) 28.(*)
29.(b) 30.(b) 31.(b) 32.(c) 33.(b) 34.(d) 35.(b)
36.(a) 37.(a) 38.(b) 39.(a) 40.(b) 41.(a) 42.(d)
43.(d) 44.(c) 45.(b) 46.(a) 47.(b) 48.(d) 49.(a)
50.(c)
Engineer’s platform
EXPLANATIONS 7. R I A T N I E ?
1. From column 1 13x2+4 = 30 I+9=R
From column 2 15x4+5 = 65 A+8=I
From column 3 12x5+8 = 68 T–7=A
N+6=T
2. 20x3+20 = 80
I+5=N
12x8+20 = 116
E+4=I
Similarly,
C+3=E
30x4+20 = 140
B+3=E
3. Option (6) is correct.
1 1 1 1 1
8. ∗ 2 = 4 , 4 ∗ 2 = 2 , 2 ∗ 2 = 1, 1 ∗ 2 = 2, 2 ∗
8
2=4

Hence the required number is 2.


9.

4. From the given fig. distance travelled by A in 2


hours= 2 * 30 = 60 km 10.

11. Chair belongs to Furniture similarly; Fork


belongs to Cutlery.

12. Compass is related to ship similarly, Vaastu is


related to Land.

And distance travelled by B in 2 hours 13. A Collection of books is called library similarly; a
= 2 * 20 = 40 km collection of data is called file.
Now total distance between their end point
= 60 - (30+ 10) 14. q: d :: b:?
= 20 km Water image of d is 'b’
Similarly, water image of 'q’ will be ‘p'
5. Option (a) is correct.

15.

Engineer’s platform
25.

16.

Option (a) is correct.


17. 21 * 3 + 2 = 65
26. 997 = 9+9+7 = 25
Similarly, 31 * 3 + 2 = 95
976 = 9+7 +6 = 22
778 = 7+7+8 = 22
18. 17 * 6 = 102
895 = 8+9+5 = 22
Similarly, 23 * 6 = 138
Hence from the given options, option (a) is
different from others, because in the given number
19. (5)2 = 25
the sum of digit is 25.
(5+1)2 = 36
Similarly, (4)2 = 16 27. Given number 8 is the cube of the number 2.
(4+1)2 = 25
28.
20. From the given options the word hug is
different from other words.

21. Analogy, Reasoning and decoding all are the


Hence, from given numbers, 11 is wrong number
part of general intelligence and reasoning, while
because instead of 11, it will be 14.
cycling is different from others.
29.
22. All the conclusion regarding the statement are
untrue. Option (d) is correct.

23.

24. So, from the above venn diagram it is clear that,


the meaningful order will be 1, 4, 2, 3.

Engineer’s platform
30. Leaf 36.

Branches

Trunk

Root
So, the meaningful order will be 2, 3, 1, 4, 5. 37. Given
7𝑥 = 8𝑘 …(i)
31. The first stage of life is infancy, second stage 5𝑦 = 6𝑘 …(ii)
Babyhood, third stage childhood and fourth stage Divide equation (i) by (ii), we get
is adulthood. 7𝑥 8𝑘 4
= =
5𝑦 6𝑘 3
32. Ramu's mother has three sons. The eldest one
is called onekari, the second one is called twokari, 𝑥 4 ∗ 5 20
= =
then the third son's name is Ramu himself. 𝑦 3 ∗ 7 21

So, the value of ratio x is to y is 20: 21.


33.
38. 44-12-2 =30
77-14-2 =61
84-16-2 = 66
44-22-2= 20
So, the required number is 20.

39. From option (a)


LHS =8 x 4 – 2 + 6 + 3 = 32 – 2 + 2
32 = 32
LHS = RHS
So, from the above diagram Ashok is the heaviest.
40. From option (a)
34. From the given word KILOMETERS the word
STREET cannot be formed, because the letter T is
not in the given word.

35.

41. From the given question, answer fig. (a) is the


mirror image.

Engineer’s platform
43. Statements: Step 10: IJ
Step 11: JL
Step 12: LM
Step 13: NK
Hence minimum 13 lines are required to make the
given figure.

50. In the given question fig. there are 16 triangles.

Conclusions:
I–x
II - √

44. The number of employees who are intelligent,


hardworking and loyal but not post graduate are
represented by 4.

47. Answer figure (6), will complete the given


question figure.

48. The number of spaces enclosed by rectangle


and circle but not by triangle is 4.

49.

Draw with following stop


Step 1: AB
Step 2: BC
Step 3: CD
Step 4: DA
Step 5: EF
Step 6: FG
Step 7: GH
Step 8: HE
Step 9: IM
Engineer’s platform
Solved Paper 2014 (Set – 1) 12. Carrot, Cabbage, Potato, Ginger, Beetroot
(a) Cabbage (b)Carrot
1. Uttarakhand: Dehradun: : Mizoram : ?
(c) Potato (d) Beetroot
(a) Aizawl (b) Kohima
(c) Shillong (d) Darjeeling
13. (a)HGFE (b) PONM
(c) DCBA (d) MSTU
2. Crime : Court : : Disease : ?
(a) Doctor (b) Medicine
14. (a)GFI (b) VUX
(c) Hospital (d) Treatment
(c) POR (d)LKM
3. YQXP : JBIA : : OVNU : ?
15. (a) vwqp (b)yxmn
(a) FAGZ (b) HRIS
(c) gfkl (d)cbrs
(c) DKCJ (d) DNEO
16. (a) (324, 18) (b) (441, 72)
4. ADGJ : BEHK : : DGJM : ?
(c) (117, 81) (d) (186, 14)
(a) KPUB (b) GJMP
(c) KNQT (d) PSVY
17. (a) (11, 121) (b) (25, 625)
(c) (12, 141) (d) (15, 225)
5. ACE : BDF : : GIK : ?
(a) HJL (b) AXP
18. Find the smallest number which when divided by 25,
(c) CFG (d) GFC
40 or 56 has in each case 13 as remainder.
(a) 1413 (b) 1400
6. CAT : BIG : : DDY : ?
(c) 1439 (d) 1426
(a) CLL (b) CLM
(c) CML (d) CEP
19. Arrange the following words as per order in the
dictionary:
7. 1 : 1 : : 10 : ?
1. Emplane 2. Empower
(a) 12 (b)110
3.Embrace 4. Elocution
(c) 210 (d) 1000
5. Equable
(a) 5, 1, 3, 2, 4 (b) 4, 2, 1, 3, 5
8. 7 : 56 : : 5 : ?
(c) 4, 3, 1, 2, 5 (d) 4, 5, 2, 3, 1
(a) 25 (b) 26
(c) 30 (d) 35
20. Which one of the given responses would be a
meaningful order of the following words?
9. The following numbers fall in a group. Which one
1. Sowing
does not belong to the group?
2. Tilling
53, 63, 83, 73.
3. Reaping
(a) 53 (b)63
4. Weeding
(c) 83 (d) 73
(a) 3, 1, 2, 4
(b) 2, 1, 4, 3
10. Which one is the same as Mumbai, Kolkata and
(c) 1, 2, 4, 3
Cochin
(d) 1, 3, 2, 4
(a) Delhi
(b)Kanpur
21. Arrange the Colors of the rainbow (in the reverse
(c) Chennai
order) (from the top edge) :
(d) Sholapur
Red, Orange, …………..
1. Blue 2. Indigo
Directions: (Q.11 – 17) Find the odd
3. Yellow 4.Green
word/letters/number pair from the given alternatives.
5.Violet
(a) 3, 4, 1, 2, 5 (b) 4, 3, 2, 5, 1
11. (a) Kolkata (b) Vishakhapatnam
(c) 5, 3, 4, 2, 1 (d) 2, 4, 3, 1, 5
(c) Bengaluru (d) Haldia
Engineer’s platform
Directions (Q. 22 – 24): A series is given, with one term 28.
missing. Choose the correct alternative from the given
ones that will complete the series.

22. CEG, JLN, QSU ? (a) 176 (b) 115


(a) QQS (b) TVX (c) 157 (d) 131
(c) HJL (d) UVW
29.
23. B-1, D-2, F-4, H-8, J-16, ?
(a) K-64 (b) L-32
(c) M-32 (d) L-64

24. CCJ, KOR, TXA, ? (a) 3 (b) 9


(a)ACE (b) JDP (c) 5 (d) 2
(c)FJM (d) UWY
Directions (Q. 25 -29): Find the missing number from 30. Arrange the letters to form a word and suggest what
the given responses. is it.
NGDEALN
25. (a) State (b) Country
(c) River (d) Ocean

31. If A = 1, B = 2, and N = 14, then BEADING = ?


(a) 2154(14)97 (b) 2514 (14) 79
(c) 25149 (14) 7 (d) 2154(14)79

32. If A = 1, AGE = 13, then CAR ?


(a) 19 (b) 20
(c) 21 (d) 22
(a) 422 (b)374 33. If an electric train runs in the direction from North
(c) 256 (d) 342 to South with a speed of 150 km/hr covering 2000 km,
then in which direction will the smoker or its engine go?
26. (a) N⇥ S (b) S ⇥ N
(c) E ⇥ W (d) No direction

34. If 1 = 1, 2 = 3, 3 = 5 and 4 = 7 , then 5 = ?


(a) 9 (b) 7
(c) 5 (d) 8
(a) 40 (b) 38
(c) 39 (d) 44 35. Find the answer of the following:
7+3 = 421
27. 11+7 = 477
9+5 = 445
6+2 = ?
(a) 444 (b) 412
(c) 475 (d) 487

36. Find the odd number out:


18, 34, 36, 54,
(a) 34 (b) 54
(a) 56 (b)57 (c)18 (d)36
(c) 58 (d) 69

Engineer’s platform
37. Introducing a girl, Ram said to his son –in law, 42. Which of the following numbers is present only in
“Her brother is the only son of my brother –in –low,” the square and the corcle?
Who is the girl of Ram?
(a) Sister –in-law (b) Niece
(c) Daughter (d) Sister

38. Which of the following are the lines of symmetry?

(a) 5 (b) 4
(c) 3 (d)2

43. Which figure represents the relation among


Computer, Internet and Information Communication
Technology?
(a) AB and CD
(b) EF and GH
(c) All of the above
(d) None of the above

39. Murthy drove from town A to town B. In the first


1
hour, he travelled 𝑜𝑓 the journey. In the next one
4 Directions : (Q. 44-45): One or two statements are
1
hour,he travelled of the journey. In the last 30 given following by three four Conclusions Arguments, l,
2
minutes, he travelled 80 km. Find the distance of the ll, lll and lV. You have to consider the statements to be
whole journey. true, even if they seem to be at variance from
(a) 240 km (b) 300 km commonly known facts. You are to decide which of the
(c) 320 km (d) 360 km given Conclusion | Arguments can definitely be drawn
from given statement (s). indicate your answer.
40. Identify the answer figure from which the pieces
given in question figure have been cut. 44. Statement:
Question figure: 1. SAGE is a reputed publisher of both journals and
books.
2. All publishing of SAGE is highly qualitative.
Conclusions:
l. SAGE publishes qualitative articles.
ll. SAGE did not publish lowest quality articles.
lll. SAGE enriches its publications by high scrutinization.
(a) Only conclusion lll
(b) All conclusions
(c) Only conclusions l and ll
(d) Only conclusions ll and lll
41. Which of the answer figures is not mad up only by
the components of the question figure? 45. Statement :
Should little children be loaded with such heavy school
bags ?
Arguments :
l. Yes, a heavy bag means more knowledge.
ll. No, heavy school bags spoil the posture of the
children.
lll. Yes, Children need to be adapted for earning
knowledge

Engineer’s platform
lV. No, a heavy bag never ensures knowledge gathering 50. In the following question, a matrix of certain
(a) l and lll appear to be strong arguments characters is given. These characters follow a certain
(b) l and lll are poor arguments trend, row –wise or column –wise. Find out this trend
(c) ll and lV are strong arguments and choose the missing character accordingly.
(d) l and lV are strong arguments

Directions (Q. 46- 47): Which answer figure will


complete the pattern in the question figure?
46.

(a) 450 (b) 550

(c) 320 (d) 420

47.

48. A piece of paper is folded and cut as shown below in


the question figures. From the given answer figures,
indicate how it will a appear when opened.

49. If a mirror is placed on the line MN, then which of


the answer figures is the correct image of the question
figure?
Answer:
1.(a) 2.(c) 3.(c) 4.(b) 5.(a) 6.(a) 7.(d)
8.(c) 9.(b) 10.(c) 11.(c) 12.(a) 13.(d) 14.(d)
15.(a) 16.(d) 17.(c) 18.(a) 19.(c) 20.(b) 21.(a)
22.(c) 23.(b) 24. (c) 25.(d) 26.(b) 27.(b) 28.(a)
29.(a) 30.(b) 31.(c) 32.(d) 33.(d) 34.(a) 35.(b)
36.(a) 37.(b) 38.(c) 39.(c) 40.(c) 41.(c) 42.(b)
43.(b) 44.(b) 45.(c) 46.(c) 47.(d) 48.(a) 49.(d)
50.(d)
Engineer’s platform
EXPLANATIONS: 10. Mumbai, Kolkata and Cochin are the capital of
1. The capital of Uttarakhand is Dehradun, Similarly the Maharashtra, west Bengal and Kerala, similarly
capital of Mizoram is Aizawl. Chennai is the capital of Tamilnadu.

2.Crime is related to court similarly Disease is related to 11. Kolkata, Vishakhapatnam and Haldia are coastal
Hospital areas except Bengaluru.
3.
12. Corrot, potato, Ginger and Beetroot all are ground
vegetables except cabbage.
13.

4.
14.

5. 15.

16. All pairs are divisible by 9, except opcept option (d).


6.

17. Given pairs of numbers are the square of first


number to get second number except option (c) is not
the square of any number.

18. LCM of 25, 40, 56 = 1413


Required number = 1400 + 13 = 1413

19. When the given words are arranged in dicionary


7. 3
(1) = 1 order we get following arrangement.
(10)3 = 1000 Elocuton, Embrace, emplane, Empower,Educate
Option(c) is correct.
8. 7 ⨯ 8 = 56
(7 – 2) ⨯ (8-2) = 30 20. First land is filled then socoing is done. After that
process of welding is done and findly reaping take
9. All numbers are prime, except 63. place. Option (b) is correct
Engineer’s platform
21. 31. A = 1, B = 2, and N = 14

R ⤍ Red
Position of letters of english alphabet starts
O ⤍ Orange
from A.
Y ⤍ Yreen
32. A=1
G ⤍ Green
AGE = 1 + 7 + 5 = 13
B ⤍ Blue
CAR = 3 + 1 +18 =22
I ⤍ Indigo
V ⤍ Violet 33. From the given question, it is clear that train is
22. moving only in the direction from north to South so
no direction will the smoke of its engine go.
34.

23.

35. 7 + 3 = (7 – 3 ) (7 ⨯ 3) = 421
11 + 7 = (11 – 7) (11 ⨯ 7 ) = 477
9 + 5 = (9 – 5) (9 ⨯ 5) = 445
6 + 2 = (6 – 2)(6 ⨯ 2) = 412
25. 2⨯2+=6
6 ⨯ 2 – 2 = 10
36. All numbers are multiple of 18, except 34.
10⨯2 +2 = 22
22 ⨯ 2- 2 = 42
37. The girl is neice of Ram.
42 ⨯2 +2 = 86
Option (b) is correct.
86 ⨯2- 2 = 170
38. In the given figure AB, CD EF and GH are the lines
170 ⨯ 2 + 2 = 342
of symmetry.

26. 9 + (2)2 = 13
39. Let the total distance of whole journey be x km.
13 + (3)2 = 22
Accoording to question
22 + (4)2 = 38

27. 8 ⨯ 3 + 1 = 25
3 ⨯ 4 +1= 13
4 ⨯ 5 + 1 = 21
5 ⨯ 6 +1 = 31
6 ⨯ 7+ 1 = 43
7 ⨯ 8 + 1 = 57 42. The number is present only in the square and the
circle is’4’.
29. 7 ⨯ 9 ⨯1 = 63 43.
3 ⨯ 1 ⨯ 7⨯ ? = 63
? =3

30. ENGLAND : It is a country.

Engineer’s platform
44. From the given statements, it is clear that all the
conclusion follow.

45. From the given statenments, it is clear that


arguments ll and lV are strong arguments.

46. Answer figure (c) will complete the pattern in the


given question figure.

47. In the given question figure answer figure (d) will


complete the pattern.

48. Answer figure (a) will be appeared when it opened

49. If a mirror is placed in the line MN, then answer


figure (d) is the miror image.

50.

Engineer’s platform
Solved Paper 2014 (Set – 2) 10.
Directions (Q. 1-8): Select the related word/ (a) 21-27 (b) 9-27
letters/number from the given alternatives. (c) 9-12 (d) 15-19
1. 5: 26 :: 8: _____ ?_____
11.
(a) 67 (b) 64
(a) 38- 76 (b) 28-84
(c) 65 (d) 66
(c) 34-76 (d) 23-64

2. Pyorrhea : Teeth: : Eczema: ?


12.
(a) Skin (b)Heart
(a) 5-7 (b) 3-8
(c) Lungs (d) Eye
(c) 6-8 (d) 4-5

3. N * O : 14 * 15 :: G * S :_?
13.
(a) 5 * 17 (b) 15 * 16
(a) Sphere (b) Triangle
(c) 6 * 18 (d) 7 * 19
(c) Circle (d) Oval

4. Writer: Book :: __?__


14.
(a) Composer: Song (b) Building: Architect
(a) Rosemary (b) Mint
(c) Poem: Poet (d) Chair: Carpenter
(c) Peepal (d) Coriander

5. BMCX: CNDY :: __?: EXFW


15.
(a) DWEV (b) DUGT
(a) ZXUR (b) ZXWU
(c) FGUT (d) DTGU
(c) YWVT (d) WUTR

6. 24: 288 : : 22: ?


16.
(a) 248 (b) 238 (a) Gold (b) Iron
(c) 240 (d) 242 (c) Brass (d) Copper

7. Car: Garage: : Aircraft: ? 17.


(a) Airdrome (b) Shelter (a) Thrive (b) Excite
(c) Hangar (d) Jetty (c) Flourish (d) Prosper

𝟑 𝟏𝟐 𝟒
8. : ∷ 𝟓:? 18.
𝟖 𝟑𝟐
16 4 (a) Krishna (b) Vaigai
(a) 20 (b) 6
(c) Kaveri (d) Narmada
5 10
(c) 6 (d) 23
19. Which one of the given response would be a
9. Which one of the following is always associated with meaningful order of the following?
JUSTICE? 1. Tissue 2. Cell
(a) Autocracy (b) Hypocracy 3. Organ
(c) Democracy (d) Legitimacy (a) (2), (3), (1) (b) (1), (2), (3)
(c) (3), (1), (2) (d) (2), (1), (3)
Directions (Q. 10-18): Find the odd number letters
figure/number pair from the given alternatives. 20. Which item will appear third in the dictionary?
(a) pair (b) pain
(c) page (d) pall
Engineer’s platform
Directions (Q.21-26): A series is given, with one term EXPLANATION
missing. Choose the correct alter-native from the given 1. (5)2 + 1 = 26
ones that will complete the series Similarly, (8)2 + 1 = 65

21. 1, 2, 8, _?, 148, 765 2. Pyorrhea is a disease related to Teeth. Similarly,


(a) 74 (b) 32 Eczema is related to skin.
(c) 40 (d) 33
3. N * O
22. BC, FGH, KLMN, _?, XYZABC ↓ ↓
(a) QRSTU (b) RSTUV 14 * 15
(c) PQRST (d) QRST (Position of alphabets start from 'A')
Similarly,
23. DE, _?, JL, MO G * S
(a) LN (b) CE ↓ * ↓
(c) GI (d) AC 7 * 17

24. 7, 12, 19, 28, 39, _? 4. Book is written by writer Similarly song is composed
(a) 51 (b) 49 by composer.
(c) 57 (d) 52 Option (a) is correct.

25. DMP, FLN, HKL, JJJ, _? 5.


(a) MIH (b) MII
(c) LIH (d) MIF

6. 24 * 12 = 288
22 * 11 = 242

7. Hangar is a large building, typically for housing


aircraft. Option (c) is correct.

ANSWERS 8.
1.(c) 2.(a) 3.(d) 4.(a) 5.(a) 6.(d) 7.(c) 𝟑 ∗ 𝟒 𝟏𝟐
=
8.(a) 9.(d) 10.(d) 11.(d) 12.(c) 13.(b) 14.(c) 𝟖 ∗ 𝟒 𝟑𝟐
15.(a) 16.(c) 17.(b) 18.(d) 19.(d) 20.(d) Similarly,
4 ∗ 4 16
21.(d) 22.(a) 23.(c) 24. (d) 25.(c) =
5 ∗ 4 20
Engineer’s platform
9. Legitimacy is always associated with Justice because 23.
justice can only be given according to law. Option (d) is
correct.

24.
10. From the given number all are divisible by 3, except
option (d).

11. From the given options, there is a pair of even 25.


number while in option (d) there is odd and even
number.

12. From the given option (c), a pair of number is


divisible by 2.

13. Sphere, circle and oval comprises volume but


triangle do not have volume. Option (b) is correct.

14. Peepal is tree while other are herbs. Option (c) is


correct.

15.

16. Brass is an alloy, while Gold, Iron and Copper are


elements which are found in free state.

19. Organ

Tissue

Cell

21. 1 * 1 + (1)2 = 2
2 * 2 + (2)2 = 8
8 * 3 + (3)2 = 33
33 * 4 + (4)2 =148
148 * 5 + (5)2 = 765

22.

Engineer’s platform
𝟓 𝟕 𝟏𝟎
Solved Paper 2013 8. : : :?
𝟗 𝟏𝟑 𝟗
14 14
GENERAL INTELLIGENCE AND REASONING (a) (b)
26 27
Directions: In questions no. I to 9. select the related 14 14
(c) 23 (d) 25
figure /letters/ number from the given alternatives.
1. Question figures
9. 3: 9:: 6: ?
(a) 1 (b) 18
(c) 17 (d) 16

Answer figures Directions: In questions no. 10 to 18. select the one


which is different from the other three responses.
10.
(a) 7-145 (b) 6-108
(c) 5-75 (d) 4-48
(a) (b) (c) (d)
11.
2. Question figures (a) Mars (b) Jupiter
(c) Earth (d) Comet

12.
Answer figures (a) Geeta (b) Quran
(c) Bible (d) Mahabharata

13.
(a) Message (b) Information
(a) (b) (c) (d) (c) Matter (d) Material

3. Question figures 14.


(a) Guitar (b) Veena
(c) Flute (d) Sitar

15.
Answer figures (a) 17-142 (b)71-34
(c) 41-28 (d) 14-28

16.
(a) 3, 5, 7,9 (b) 5, 7,9, 11
(a) (b) (c) (d)
(c) 4,6, 8, 10 (d) 2, 5,9, 10
4. 23 8:: 32: ?
17.
(a) 6 (b) 9
(a) 8662 (b) 5731
(c) 17 (d) 27
(c) 4628 (d) 2864
5. MLKJ: NOPQ:: IHGF:?
(a) UTSR (b) RSTU
18.
(c) SRUT (d) UTRS
(a) Tagore (b) Raman
6. ACEG: ZXVT:: BDFH:?
(c) Bhaskara (d) Khurana
(a) YWUS (b) YXWV
(c) YWVT (d) YXVW
19. Arrange the following words in a meaningful order:
7. BADC: XWZY:: FEHG:?
1. Grapes 2. Vineyard
(a) VXRT (b) TSVU
3. Whisky 4. Brewing
(c) YXCV (d) VSXW
5. Distillation
Engineer’s platform
(a) 2, 1, 5, 4, 3 (b) 3, 5, 4,2, 1 29. A car covers the first half of the distance between
(c) 2,1,4, 3, 5 (d) 2, 1,4, 5, 3 two places at 40 km/h and the second half of the
distance at 60 km/h. So what is the average speed of
20. Which will appear fourth in the dictionary? the car?
(a) Xylophilous (b) Xylophagus (a) 45 km/h (b) 48 km/h
(c) Xylopyrography (d) Xylophagan (c) 50 km/h (d) 60 km/h

21. Number of letters skipped in between adjacent 30. In a certain code language, TOGETHER is written as
letters in the series increases by one. Which of the EGTORETH. How is CONGRATULATE written in that
following series observes the rule given below? language?
(a) BEIN (b) CDJO (a) GRTULTEANOC (b) GNCOUTRAETLA
(c) GJLS (d) QUNZ (c) GNOCUTARETAL (d) GLNAOTCEURTA

22. In the following words, the group of letters should 31. In certain code language, REQUEST is written as
not contain more than three vowels. Which of the S2R52TU. How is RETEST written in that language?
following words does not conform to the rule? (a) S2V2RV (b) S2U2RU
(a) SCARCITY (b) PROGNOSIS (c) S2U2TU (d) S2V2TV
(c) COMPLEXITY (d) CONVULSION
32. Some equations are solved on the basis of a certain
Directions: In questions no. 23 to 26, choose system. On the same basis, find out the correct answer
correct alternative from the given responses that for the unsolved equation. If 42 = 7, 52 = 7, 62 = 9, then
will complete the series. 72 = ?
23. ?, PSV, EHK, TWZ, ILO (a) 14 (b) 13
(a) BEH (b) IMP (c) 10 (d) 8
(c) ACG (d) ADG
33. Find out the number which belongs to the given
24. 78, 86, ?, 88, 82, 90 group of numbers from the alternatives. 246, 579, 135,
(a) 76 (b) 84 35, 68
(c) 83 (d) 80 (a) 55 (b) 468
(c) 123 (d) 31
25. 3 7 13 ? 31 43 57
(a) 51 (b) 81 34. If P stands for ÷, Q stands for x, R stands for +,
(c) 41 (d) 21 then 18 Q 12 P 4 R 5 = ?
(a) 59 (b) 26
26. EJOT, INSX, AFKP, ? (c) 11.7 (d) 2.33
(a) CHMS (b)XTOJ
(c) BGLQ (d) E.JOT 35. From the given alternatives, select the word which
con not be formed using the letters of the given word.
27. My father has two brothers. The youngest has two ACCOMPANIED
sons and one daughter. The elder one has one son and (a) PANIC (b) COME
two daughters and the remaining one has three sons. If (c) COMB (d) PAIN
my father has four nephews, how many cousins
(brothers) have I got? Directions: In questions no. 36 and 37, find the missing
(a) 6 (b) 4 number from the given responses.
(c) 7 (d) 5 36.

28. Find the wrong number in the given series.


3, 7, 15, 31, 64, 127
(a) 127 (b) 6
(c) 31 (d) 3

Engineer’s platform
(a) 43 (b) 17 (a) 5 (b) 4
(c) 23 (d) 37 (c) 6 (d) 7

37. 6 11 25 42. In the following Venn diagram, identify the letter


8 6 16 which denotes Film Actors who are Singers but not
? 5 16 Directors.
(a) 10 (b)14
(c) 12 (d) 16

38. Ram travelled 6ft towards West, he turned left and


walked 8ft, then turned left and walked 4ft, then turned
left and walked 8ft again. How far is he now from the
starting point? (a) D (b) C
(a) 8ft (b) 6ft (c) E (d) F
(c) 4ft (d) 2ft
43. Identify the answer figure from which the pieces
39. How many black-faced cubes are there in the given given in the question figure have been cut.
structure? Question figure

Answer figures

(a) (b) (c) (d)

(a) 75 (b) 55 Directions: In questions no. 44 and 45, one or two


(c) 25 (d) 15 statements are given, followed by two conclusions I and
II. You have to consider the statements to be true even if
40. The door of Priya's house faces East. From the back they seem to be at variance from commonly known
side of the house, she walks straight 50 meters, then facts. You have to decide which of the given conclusions,
turns to the right and walks 50 meters again. Finally, she if any, follows from the given statements.
turns towards the left and stops after walking 25
meters. Now Priya is facing which direction? 44. Statement:
(a) North (b) West A social movement is an interaction in of people with a
(c) East (d) South common motivational base in frustration.
Conclusions:
41. In the following diagram, rectangle represents I. In a social movement, people who are satisfied
males, circle represents urban and square represents interact with frustrated people.
educated. Which region represents educated urban II. Frustrated people interact with each other in a social
males? movement.
(a) Only conclusion I follows
(b) Only conclusion II follows
(c) Neither conclusion I nor II follows
(d) Both conclusions I and II follow

45. Statement:

Engineer’s platform
All scientists are hard-working. No hard-working man is 48. Components of which of the answer figures will
poor. exactly make up the question figure given below.
Conclusions: Question figure
I. No scientist is poor
II. No poor man is a scientist
(a) Only conclusion I follows
(b) Only conclusion II follows
(c) Neither conclusion I and II follow Answer figures
(d) None of the conclusions I or II follows

46. Which of the answer figures is exactly the mirror


image of the given figure, when the mirror is held on
the line AB?
(a) (b) (c) (d)
Question figure
49. Select the answer figure in which the question
figure is hidden/embedded.
Question figure

Answer figures

Answer figures

(a) (b) (c) (d)

47. A word is represented by only one set of numbers (a) (b) (c) (d)
as given in any one of the alternatives. The sets of
numbers given in the alternatives are represented by 50. A piece of paper is folded and punched as shown
two classes of alphabets as in the two matrices given below in the question figures. From the given answer
below. The columns and rows of Matrix I are figures, indicate how it will appear when opened?
numbered from 0 to 4 and that of Matrix II are Question figures:
numbered from 5 to 9. A letter from these matrices
can be represented first by its row and next by its
column, e.g., 'A can be represented by 13, 76, etc. and
G can be represented by 22, 65 etc. Similarly, you have
to identify the set for the word PUBLIC". Answer figures:

(a) (b) (c) (d)

(a) 12, 30, 87, 41, 66, 33


(b) 99, 30, 87, 77, 23, 44
(c) 55, 01, 87, 98, 34, 87
(d) 40, 30, 87, 89, 24, 43

Engineer’s platform
ANSWERS 7.
1.(c) 2.(c) 3.(a) 4.(c) 5.(b) 6.(a) 7.(b)
8.(b) 9.(b) 10.(a) 11.(d) 12.(d) 13.(d) 14.(c)
15.(d) 16.(d) 17.(b) 18.(c) 19.(d) 20.(c) 21.(a)
22.(d) 23.(a) 24.(d) 25.(d) 26.(c) 27.(b) 28.(b)
29.(b) 30.(b) 31.(c) 32.(b) 33.(b) 34.(a) 35.(c)
36.(c) 37.(b) 38.(d) 39.(b) 40.(b) 41.(b) 42.(a)
43.(d) 44.(b) 45.(c) 46.(c) 47.(a) 48.(d) 49.(b) 8.
50.(a)

EXPLANATIONS
1.

9. 3:9::6:?
3*3=9
6 * 3 = 18
2.
10. All others are divisible by first numbers.

11. All others are planet.

12. All others are books.


3.
13. All others are non-visible.

14. All others have strings.

15. All others are not divisible by first number


4. 23 – 15 = 8
16.
32 - 15 = 17

5. MLKJ => Reverse Sequence


NOPQ => Actual Sequence
IHGF => Reverse Sequence
by matching option
RSTU => Actual Sequence
6.

17. All others are even numbers.

18. All others got Nobel prize.

19. Sequence of making whisky


Vineyard => Grapes => Brewing => Distillation =>
Whisky.

Engineer’s platform
20. Correct sequence in dictionary 29.
(i) X y l o p h a g n
(ii) X y l o p h a g u s
(iii) X y l o p h i l o u s
(iv) X y l o p y r o g r a p h y (40+50)
Avg. Speed = 2
= 50𝑘𝑚/ℎ
21.
30.

22. Number of vowel


SCARCITY => 2
31.
PROGNQSIS => 3
COMPLEXITY => 3
CONVULSION => 4

23.

32. 42 = 16 => 1 + 6 = 7,
52 = 25 => 2 + 7 = 7,
62 = 36 => 3 + 6 = 9,
72 = 49 => 4 + 9 = 13

34. P=÷
(+8) (-6) (+8) (-6) (+8)
24. 78 → 86 → 80 → 88 → 82 → 90 Q=*
R=+
25. 18 Q 12 P 4 R 5
3 →(+4) 7→(+6) 13→(+8) 21 →(+10) 31 →(+12) 43 →(+14) 57 18 * 12 ÷ 4 + 5
According to BODMAS rule
26. E→(+5) J →(+5) O →(+5) T 18 * 3 + 5 = 54 + 5 = 59
I →(+5) N →(+5) S →(+5) X
A →(+5) F →(+5) K →(+5) P 35. COMB cannot be formed because there is no B in
B →(+5) G →(+5) L →(+5) Q ACCOMPANIED.

27. 36.

(15 * 12) - (2 * 12) = 156


28. 3 →(+4) 7 →(+8) 15 →(+16) 31 →(+33)
64 →(+64) 127

Engineer’s platform
44. From given statement conclusion II can be follow

45. From given statement both conclusion I and II can


be follow.

(21 * 17) - (2 * 17) = 323 46.

(x * 25) - (2 * 25) = 252 47.


x = 23 P → 12, U → 30, B → 87
L → 41, I → 66, C → 33
37.
6 11 25 => (11 X 2) + (6 + 2) = 25
8 6 1 6 => (6 X 2) + (8 + 2) = 16
? 5 1 6 =>
Similarly, 5 X 2 + x ÷ 2 = 16
𝑥
∴ 10 + = 16
2
∴ 𝑥 = 12
Option (c) is correct.

38.

39. Total black faced cubes are 55

40.

Priya is facing towards west

41. 4 represents educated Urban males

42. D represents Actors who are Singers but not


Directors

43. When all the pieces are joined together we will get
figure (d).

Engineer’s platform
Solved Paper 2012 Conclusions:
I: Persons without empathy cannot become good
GENERAL INTELLIGENCE AND REASONING
teachers.
1. How many white cubes are there in the given
II: Good teachers understand the problems of their
students.
(a) Neither conclusion I nor II follows
(b) Both conclusion I and II follow.
(c) Only conclusion I follows.
(d) Only conclusion II follows

structure? 6. Which answer figure will complete the pattern in


(a) 40 (b) 65 the question figure?
(c) 16 (d) 24 Question Figure:

2. In the following Venn diagram, identify the number


which denotes Doctors who know both Swimming and
Dancing.
Answer Figures:

(a) 6 (b) 5 (a) (b) (c) (d)


(c) 3 (d) 4
7. Select the answer figure in which the question figure
3. Which one of the following diagrams best depicts is hidden/embedded.
the relationship among College Graduates, Question Figure:
Professional Athletes and Great Scientists?

Answer Figure:

Directions (Q. 4-5): Two statements are given followed


by two conclusions I and II. You have to consider the (a) (b) (c) (d)
statements to be true even if they seem to be at
variance from commonly known facts. You are to decide 8. A piece of paper is folded and punched as shown
which of the given conclusions, if any, follow from the below in the question figures. From the given answer
given statements. figures, indicate how it will appear when opened?
4. Statements: Mind is a stream of thoughts. Mind is Question Figure:
working all the time.
Conclusions:
I: If there is no thought, there is no mind.
II: Thoughtless people will not succeed. Answer Figure:
(a) Neither conclusions I nor II follows.
(b) Both conclusion I and II follow
(c) Only conclusion I follows
(d) Only conclusion II follows.
(a) (b) (c) (d)
5. Statements: Teachers should have empathy.
Students need empathetic approach from their
teachers.
Engineer’s platform
9. Which of the answer figure is exactly the mirror Answer Figures:
image of the given figure, when the mirror is held on
the line AB?
Question Figure:
(a) (b) (c) (d)

13. Question Figures:


Answer Figures:

Answer Figures:
(a) (b) (c) (d)

10. A word is represented by only one set of numbers


as given in any one of the alternatives. The sets of (a) (b) (c) (d)
numbers given in the alternatives are represented by
two classes of alphabets as in two matrices given 14. ?: JHKI:: TRUS: OMPN
below. The columns and rows of matrix I and II are (a) GEHF (b) GEFH
numbered from 0 to 4. A letter from these matrices (c) LOMP (d) QMPN
can be represented first by its row and next by its
column, e.g., 'A' can be represented by 24, 31 etc. and 15. AEJO: ZVQL: : DINS: ?
P" can be represented by 11,32, etc. Identify the set (a) WRMH (b) WSOJ
for the letters AELO. (c) WRNJ (d) WSNI

16. IRTH: HQSG:: ?: RQPO


(a) QPON (b) PQRO
(c) OPQR (d) SRQP

17. 16: 64:: 25: ?


(a) 83 (b) 125
(a) 31, 00, 23, 22 (b) 43, 01, 12, 42 (c) 55 (d) 110
(c) 12, 34, 30, 02 (d) 12, 30, 42, 14
18. 5: 15:: 40: ?
Directions (Q. 11-19): Select the related word/ (a) 60 (b) 45
letter/number/figure from the given alternatives. (c) 120 (d) 55

11. Question Figures: 19.

(a) 81 (b) 196


Answer Figures: (c) 169 (d) 324
Directions (Q. 20-28): Select the one which is different
from the other three responses
20.
(a) (b) (c) (d) (a) Aluminium (b) Tungsten
(c) Copper (d) Diamond
12. Question Figures:
21.
(a) Customer (b) Hawker
(c) Broker (d) Salesman
Engineer’s platform
22. (a) xy (b) rs
(a) Weaver (b) Spinner (c) yx (d) sr
(c) Engineer (d) Potter
33. PQR, HLJ, DEF, ?
23. (a) ABC (b) BCD
(a) Champaka (b) Hibiscus (c) DEF (d) CDE
(c) Rose (d) Jasmine
34. NDB, LED, JGG, ?
24. (a) LNP (b) HED
(a) Cholera (b) Jaundice (c) HJJ (d) HJI
(c) AIDS (d) Typhoid
35. 18, 54, 162, 486, 1458, ?
25. (a) 39366 (b) 4374
(a) RQFJ (b) ODHR (c) 2187 (d) 13122
(c) SRBH (d) RHSN
36. 20, 30, 42, 56, 72, ?
26. (a) 87 (b) 95
(a) 114 57 28 (b) 120 60 30 (c) 85 (d) 90
(c) 144 72 36 (d) 124 62 31
37. Find the wrong number in the given series
27. 7, 15, 32, 65, 138
(a) 8987 (b) 6354 (a) 6 (b) 138
(c) 7832 (d) 2398 (c) 7 (d) 15

28. 38. A party consisted of a man, his wife, his three sons
(a) 49-7 (b) 36- 6 and their wives and three children in each son's family.
(c) 64-8 (d) 80-9 How many were there in the party?
(a) 17 (b) 24
29. Arrange the given words in the order in which they (c) 22 (d) 13
occur in the dictionary and find the last but one word:
(a) Faubourg (b) Fatiscent 39. 6,500 were divided equally among a certain
(c) Fauxbourdon (d) Favonian number of persons. Had there been 15 more persons,
each would have got 30 less. Find the original number
30. Arrange the following words in the order in which of persons.
they occur in the dictionary. Which will appear fourth (a) 50 (b) 55
in the dictionary? (c) 40 (d) 45
(a) Nucleosynthesis (b) Nucleoprotein
(c) Nucleonic (d) Nuclearize 40. From the following alternatives, select the word
which cannot be formed using the letters of the given
31. Arrange the following words in the order in which word:
they occur in the Dictionary: UNIVERSITY
(1) Interview (2) Inventory (a) NEVER (b) REST
(3) Invention (4) Interval (c) INVERT (d) UNITE
(5) Investment
(a) 3, 5, 4, 1, 2 (b) 2, 3, 5, 4, 1 41. In a certain code MEN is written as MIN and
(c) 2, 5, 3, 1, 4 (d) 4, 1, 3, 2, 5 WOMEN is written as WUMIN, then how will CHILD be
written in the same code?
Directions (Q.32-36): Choose the correct alternative (a) CHOLD (b) CHULD
from the given responses that will complete the series: (c) CHELD (d) CHALD

32. xy, wy, xy, ut, xy, __?


Engineer’s platform
42. If Y = 2, PEN = 11 - 12 - 13, 48. Going 60 metres to the South of his house, Kiran
Then 10 - 6 - 18 - 24 - 16 =? turns left and goes another 20 metres then turning to
(a) QUICK (b) QUITE the North, he goes 40 metres and then starts walking
(c) JFRXP (d) QUACK to his house which direction is his house. In which
direction is his house from there?
43. Find out the number which belongs to the given (a) East (b) North West
group of number from the four alternatives. (c) North (d) South East
5, 25, 90, 35, 60
(a) 21 (b) 83 49. Ram started walking towards East. After 1 km, he
(c) 15 (d) 24 turned South and walked 5 km. Again, he turned East
and walked 2 km. Finally, he turns to the North and
44. If + stands for division walked 9 km. How far is he from the starting point?
- stands for equal to (a) 5 km (b) 7 km
* stands for addition (c) 3 km (d) 4 km
÷ stands for greater than
= stands for less than 50. Four positions of a cube are shown below. If
> stands for multiplication symbol Sun is at the top, what symbol will be at the
< stands for subtraction bottom?
then of the given alternatives which one is correct? Question figures:
(a) 5 * 3 < 7 ÷ 8 + 4 < 2
(b) 5 + 3 > 7 - 8 * 4 + 2
(c) 5 > 3 * 7 = 8 > 4 + 2
(d) 5 < 3 > 7 – 8 > 4 + 2 Answer figures:

Direction (Q. 45-47): Select the missing number from


the given responses.
(a) (b) (c) (d)
45.

(a) 120 (b) 195


(c) 61 (d) 89

46.

(a) 33 (b) 123


(c) 121 (d) 63 ANSWERS:
1.(a) 2.(d) 3.(c) 4.(a) 5.(c) 6.(b) 7.(a)
47. 2 5 7 8.(d) 9.(d) 10. (c) 11.(b) 12.(c) 13.(d) 14.(a)
6 15 211 15.(a) 16.(d) 17.(b) 18.(c) 19.(b) 20.(d) 21.(a)
10 19 ? 22.(c) 23.(b) 24.(c) 25.(a) 26.(a) 27.(b) 28.(d)
(a) 28 (b) 52 29.(c) 30.(a) 31.(d) 32.(d) 33.(b) 34.(c) 35.(b)
(c) 29 (d) 25 36.(d) 37.(a) 38.(a) 39.(a) 40.(a) 41.(a) 42.(a)
43.(c) 44.(a) 45.(c) 46.(b) 47.(b) 48.(b) 49.(a)
50.(c)

Engineer’s platform
EXPLANATIONS
2. Swimmer (S) = (2, 3, 4, 5)
Doctors (DO) = (1, 3, 4, 6)
Dancers (DA) = (4, 5, 6, 7)
∴ S ⋂ DO ⋂ DA = {4}
Option (d) is correct. Option (a) is correct

3. Relationship among college graduates, professional 16. I R T H : H Q S G : : ? : R Q P O


athletics and great scientist is depicted through diagram
below.

Similarly,

10. From option (c),


12 => A
34 => E
30 => I
Option (d) is correct.
02 => O
Option (c) is correct.
17. 16 : 64 : : 25 : ?
16 : 64
11. By symmetry concept, option (b) is correct.
(4)2 : (4)3
Similarly, 25 : ?
14. ?: J H K I : : T R U S : O M P N
(5)2 : (5)3
? = 125
Option (b) is correct.

18. 5 : 15 : : 40 : ?
5 *3 = 15
Such relation is found in option (a) Similarly, 40 * 3 = 120
i.e. Option (c) is correct.

19.

Option (a) is correct.


5 + 2 + 1 = 8 => (8)2
15. A E J O : Z V Q L : : D I N S : ?
Similarly, 4 + 3 + 7 = 14 => (14)2
142 = 196
Option (b) is correct.
20. Among the given options, diamond has highest
hardness.
Option (d) is correct.
Similarly,
21. Hawker, broker and salesman advertise the product
to sell it to customer.
Option (a) is correct.
Engineer’s platform
24. Typhoid, cholera and jaundice affect a particular Option (b) is correct.
part of body while AIDs affect the immune system.
Option (c) is correct. 34.

26.
From option (a), 114 57 28 => 57 + 28 * 2 = 113
From option (b), 120 60 30 => 60 + 30 * 2 = 120
From option (c), 144 72 36 => 72 + 36 * 2 = 144
From option (d), 124 62 31 => 62 + 31 * 2 = 124
Option (a) is correct. Option (c) is correct.

27. Among the given option only 6354 is divisible by 3. 35. 18, 54, 162, 486, 1458, ____?
Option (b) is correct.

28. From option,


(7)2 = 49
(6)2 = 36
(8)2 = 64
But (9)2 = 81 ≠ 80
Option (d) is correct.

29. Arranging the given words in dictionary order we


get following sequence.
Fatiscent, Faubourg, Fauxbourdon, Favonian. Hence,
second last word is "Fauxbourdon"
Option (c) is correct.

30. Arranging the given words in dictionary order we


get the following sequence.
Nuclearize, Nucleonic, Nucleoprotein, Nucleosynthesis
Hence last word is Nucleosynthesis
Option (a) is correct.

31. Arranging the given words in dictionary order we


get the following sequence.
Interval, Interview, Invention, Inventory,
Investment
The order of sequence is 4, 1, 3, 2, 5
Option (d) is correct.

32.

Option (d) is correct.


33. PQR, HIJ, DEF, ?

Engineer’s platform
Solved Paper 2011 10.
(a) Apple (b) Mango
Directions (Q 1-8): Select the related
(c) Potato (d) Orange
word/letters/number from the given alternatives.
11.
1. 25: 37:: 49: ? (a) Father- Daughter (b) Husband- Wife
(a) 12 (b) 56 (c) Son-Mother (d) Brother - Sister
(c) 4 (d) 65 12.
(a) BCD (b) FGH
2. 9: 39:: 45: ? (c) NOP (d) IKL
(a) 175 (b) 185 13.
(c) 195 (d) 205 (a) DHKM (b) JNPR
(c) KORT (d) MQTV
3. ONM: JIH:: TSR: ? 14.
(a) TSU (b) LMN (a) 4-9 (b) 36-49
(c) XYZ (d) ONM (c) 16-25 (d) 64-80

4. ZYXW: DCBA:: SRQP: ? 15.


(a) ZXYP (b)KJIH (a) 5698 (b) 7894
(c) JHIL (d) ABCD (c) 9865 (d) 8793
5. Question figures 16. Which one of the given responses would be the
English dictionary order of the following words?
1. Protein 2. Problem
3. Proverb 4. Property
5. Project
Answer figures: (a) 2, 1, 4, 3, 5 (b) 1,2, 3, 4,5
(c) 3,4, 5, 2, 1 (d) 2, 5, 4, 1,3

17. A series of figures is given which can be grouped


into classes. Select the group into which the figures
(a) (b) (c) (d) can be classified from the given responses.
Question figures:
6. Tired: Work:: Happy: ?
(a) Rest (b) Success
(c) Eating (d) Exercise
Answer:
7. Music: Sitar:: ? (a) 1, 2, 3; 4, 5, 6; 7, 8, 9
(a) Author: Book (b) Literature: Novel (b) 1, 4, 6; 2, 7, 9; 3, 5, 8
(c) Guitar: Sound (d) Painter: Picture (c) 1, 3, 7; 2, 6, 9; 4, 5, 8
(d) 1, 2, 4; 3, 6, 5; 7, 8, 9
8. DEA: 10:: ACE: ?
(a) 8 (b) 10 18. Which one of the given responses would be a
(c) 9 (d) 11 meaningful order of the following words?
1. Nation 2. Village
Directions (Q.9-15): Find the odd word/ number letters/ 3. Taluk 4. District
word or number pair from the given alternatives. 5. State
9. (a) 2, 3, 4, 1, 5
(a) 36 (b) 64 (b) 1, 3, 5, 4, 2
(c) 81 (d) 46 (c) 2, 3, 4, 5, 1
(d) 1, 2, 3, 4, 5
Engineer’s platform
Directions (Q. 19): Which one set of letters when 27. If in a certain code, TOWER is written as OTEWR,
sequentially placed at the gaps in the given letter series then in that code, which word will be written as
shall complete it? RTAIL?
(a) TRIAL
19. bcabbc_bbca_bcabb_ab (b) TRAIL
(a) acaa (b) cabe (c) TRLAI
(c) acba (d) cacc (d) TAILR
Directions (Q. 20-24): A series is given with one two
term(s) missing. Choose the correct alternative from the 28. If HARD is coded as 1357 and SOFT is coded as
given ones that will complete the series. 2468, then 21448 stands for
(a) HSOOT (b) SHOQS
20. 4, 14, 34, ?, 104, 154 (c) SHOOT (d) RGOQT
(a) 64 (b) 54
(c) 74 (d) 84 29. Choose one word out of the given alternatives,
which can be formed from the letters of the word
21. 75, 77, 72, 80, 69, 83, ?, ? ‘DISOBEDIENCE'.
(a) 65,87 (b) 66, 80 (a) DISTANCE (b) OBEDIENT
(c) 66, 86 (d) 72, 86 (c) BESIDE (d) DEFENCE

22. Question figures: 30. Sachin walked 10 kms to north, turned left and
walked 5 kms. Again, he turned south and walked 4
kms. Finally, again he turned left and walked 5 kms. In
which direction is Sachin from the starting point?
(a) East (b) West
Answer figures: (c) North (d) South

31. A boy starts from home, which faces south. He


walks a distance of 10 m, turns right and walks 10 m.
(a) (b) (c) (d) Then he turns right and walks 10 m, then turns left and
walks 10 m and walks back 5 m. What is the distance
23. A, E, I, M, Q, U, ? between the home and the boy?
(a) V (b) W (a) 35 m (b) 45 m
(c) Z (d) Y (c) 15 m (d) 20 m

24. CDF, DEC, EFH, ? 32. Five students M, N, 0, P and Q are standing in a
(a) FGI (b) FGH row.
(c) FHG (d) EFI O, who is the third to the left of P, is to the immediate
right of M and second to the left of Q. Q. who is not at
25. Which number is wrong in the given series? any of the ends, is the third to the right of M. P is at
7, 9, 15, 27, 51, 99 one of the ends.
(a) 7 (b) 9 Then who is standing in the middle?
(c) 15 (d) 27 (a) M (b) N
(c) O (d) Q
26. The average age of 10 children in a class is 12
years. If two children aged 14 and 16 years join the 33. Among 5 sons of Raghav, Anand is the elder
class, it will raise the average age by how much? brother of Bhima, and Chandra is the elder brother of
(a) 2 months (b) 4 months Anand. Bhima has younger brothers Dhanu and
(c) 6 months (d) 8 months Ishwar. Who is the eldest son of Raghav?
(a) Bhima (b) Anand
(c) Chandra (d) Dhanu

Engineer’s platform
34. Amul is Atul's brother. Darly is Atul's mother. Lali is 40. The Manager
Darly's mother. Prakash is Lali's father. How is Amul R&C Finance Limited
related to Prakash? The Metropolitan, 10th Floor, B Block, Bandra(W)
(a) Son-in-law (b) Grandson Mumbai-400051
(c) Great-grandson (d) Son (a) The Manager
R&C Finance Limited
35. A father's age is 3 times of his son's age at present. The Metropolitan, 10th Floor B Block, Bandra
10 years back, the father's age was 5 times of his son's (W)
age. What is the age of the son at present? Mumbai-500051
(a) 20 years (b) 30 years (b) The Manager
(c) 25 years (d) 15 years R&C Finance Limited
The Metropolitan, 10th Floor, B Block, Bandra
36. I am 12th in the queue from either end. How many (W)
persons are there in the queue? Mumbai-400051
(a) 13 (b) 23 (c) The Manager,
(c) 14 (d) 24 R&C Finance Limited
The Metropolitan, 11th Floor, D Block, Bandra
37. Choose one word out of the given alternatives, (W)
which cannot be formed from the letters of the word Mumbai-400051
'COMMENTATOR'. (d) The Manager
(a) MENTOR (b) COMMON R&C Finance Limited
(c) ROTATE (d) MEMBER The Metropolitan, 10th Floor, B Block, Bandra
(W)
38. Statements: Mumbai-400050
1. Some teachers are students. Directions (Q. 41- 42): Select the missing number from
2. All students are girls. the given responses.
Conclusions:
I. All teachers are girls. 41. 8 3 9
II. Some girls are teachers. 6 4 2
III. Some girls are students. 4 8 ?
IV. All students are teachers. 192 96 324
(a) Only I follow (a) 48 (b) 28
(b) Only I and III follow (c) 18 (d) 38
(c) Only II and III follows
(d) All follow 42.

39. Statements:
1. All young ladies are modern.
2. No modern men are religious minded.
Conclusions: (a) 66 (b) 68
I. No young ladies are religious minded. (c) 70 (d) 62
II. No young men are religious minded.
(a) Only I follow (b) Only II follows 43. If ÷ stands for *, - for +, + for ÷, then which of the
(c) Both I and II follow following equations is correct?
(d) Neither I nor II follows (a) 21 - 3 + 9 ÷ 12 = 51
(b) 21 + 3 ÷ 9 - 12 = 75
Directions (Q. 40): There is an address which has been (c) 21 ÷ 9 + 3 - 12 = 51
reproduced against (a), (b), (c) and (d). Three of these (d) 21 - 3 ÷ 9 + 12 = 75
have some mistake or the other. The one without any
mistake is your answer 44. Some equations are solved on the basis of a certain
system. On the same basis, find out the correct answer
for the unsolved equation.
Engineer’s platform
If 4 + 3 = 25 and 5 + 2 = 29, then 6 + 4 =? 48. Which answer figure will complete the question
(a) 52 (b) 100 figure?
(c) 50 (d) 48 Question figure:

45. Postal PIN codes of 25 letters are given below. The


first digit from the left indicates the zone and the last
three digits the delivery Post Office. How many
Answer figures:
maximum letters are meant for the same delivery Post
Office under Zone 2?
101012 221718 303051 221104 421015
221054 310032 101113 221108 230221
(a) (b) (c) (d)
308015 308012 221054 101012 221054

111118 221054 210502 221054 101114


49. From the given answer figures, select the one in
which the question figure is hidden/embedded.
210512 182114 128107 183115 610014 Question figure:

(a) 3 (b) 4
(c) 5 (d) 6
Answer figures:
Directions (Q. 46): A piece of paper is folded and cut as
shown below in the question figures. From the given
answer figures, indicate, how it will appear when
opened.
(a) (b) (c) (d)
46. Question figures
50. Which of the given diagrams correctly represents
the relationship among
Mammals, Reptiles and Birds?
(a)
Answer figures:

(b)

(a) (b) (c) (d)


(c)
47. If a mirror is placed on the line AB, then which of
the answer figures is the correct image of the given
question figure? (d)
Question figure:

ANSWERS:
1.(d) 2.(c) 3.(d) 4.(b) 5.(a) 6.(b) 7.(b)
8.(c) 9.(d) 10.(a) 11.(b) 12.(d) 13.(b) 14.(d)
Answer figures: 15.(d) 16.(d) 17.(b) 18.(c) 19.(b) 20.(a) 21.(c)
22.(a) 23.(d) 24.(a) 25.(a) 26.(c) 27.(a) 28.(c)
29.(c) 30.(c) 31.(c) 32.(b) 33.(c) 34.(c) 35.(a)
36.(b) 37.(d) 38.(c) 39.(d) 40.(b) 41.(c) 42.(d)
43.(b) 44.(a) 45.(d) 46.(b) 47.(c) 48.(b) 49.(d)
(a) (b) (c) (d) 50. (c)
Engineer’s platform
EXPLANATION 15. (a) 5698 → 5+6 +9+8= 28
1. 25: 37: :49: ? (b) 7894 → 7+8+9+4 = 28
i.e. 52 :62 + 1 : : 72 : 82 + 1 (c) 865 → 9+8+6+5 = 28
Next number will be 65. (d) 8793 → 8+7+9+ 3 = 27

3. 16. Problem→Project→Property→Protein→Proverb
2, 5, 4, 1, 3

18. Village→Taluk→District→State→Nation
2, 3, 4, 5, 1
4. ZYXW: DCBA: : SRQP : KJIH
19. b c a b b c a b b c a b b c a b b c a b
8. DEA: 10 : : ACE: ?
The correct square of latter is bcab
D → 4 Similarly, A→1
E→5 C→3
20.
A→1 E→5
10 9

9. 36, 64 and 81 are perfect square, where as 46 is not a


21.
square of any number.

10. Apple is the false fruit.

11. All relations, other than Husband-wife are blood 22. Here, cross is rotating by 45° whereas point is
relation. rotating by 90° in anticlockwise direction so the next
figure will be (a)
12.
(a) B(+1) C (+1) D (b) F+1)G (+1)H 23.
(c) N(+1) O(+1) P (d) I(+2) K(+1) E

13.
(a) 24.

(b)
25.

(c)
The difference between first two number should be 3

(d) 26. Total age to 10 children = 120


after adding two more children = 150
Now total children = 12
In option (b) difference between N and P is 2. 150
average age = 12 = 12.5
Raised by .5 years = 6 months
14. (a) 4 - 9 (b) 36 - 49
22 – 32 62 – 72
27. O T E W R–T O W E R
(c) 16 -25 (d) 64 - 80
1 2 3 4 5- 2 1 4 3 5
42 – 52 82 - 8.94
R T A I L -T R I A L
Second term of option(d) is not perfect square.
1 2 3 4 5

Engineer’s platform
28. H A R D S O F T So, there are 23 persons
1 3 5 7 2 4 6 8
Then 214 4 8 37. In option (d), there is an alphabet ‘B’ which is not in
SHOOT word "COMMENTATOR"

30. 38.

(i) Some girls are teacher


(i) Some girls are student
From starting point he is in north direction.
41. 8 3 9
* * *
31.
6 4 2
* * *
4 8 ? (18)

32. MONQP
N is standing in the middle.

33. Chanda

Anand

Bhuma
↓ ↓
Dhanu Ishwar

34.

35. Let son's age = 𝑥 year


Father's age = 3𝑥 year at present
Now according to question
5(𝑥 − 10) = (3𝑥 − 10)
5𝑥 − 50 = 3𝑥 − 10
2𝑥 = 40
𝑥 = 20 𝑦𝑒𝑎𝑟𝑠

36. 1 2 3 4 5 6 7 8 9 10 11 12
23 22 21 20 19 18 17 16 15 14 13 12
Engineer’s platform

You might also like